You are on page 1of 154

INSURANCE G01 CASE DIGESTS

POLICY
67.

RCBC V. CA (FULL CASE POSTED IN THE FB GROUP - ADDITIONAL


CASE - PLEASE READ ON YOUR OWN- FIRST CASE TO BE DISCUSSED)

68.

PACIFIC BANKING V . CA, 168 SCRA 1 (1988)

Facts:
An open fire insurance policy, was issued to Paramount Shirt
Manufacturing by Oriental Assurance Corporation to indemnify
P61,000.00, caused by fire to the factorys stocks, materials and
supplies.
The insured was a debtor of Pacific Banking in the amount of
(P800,000.00) and the goods described in the policy were held in
trust by the insured for Pacific Banking under trust receipts.
The policy was endorsed to Pacific Banking as mortgagee/ trustor of
the properties insured, with the knowledge and consent of private
respondent to the effect that "loss if any under this policy is payable
to the Pacific Banking Corporation".
A fire broke out on the premises destroying the goods contained in
the building.
The bank sent a letter of demand to Oriental for indemnity.
The company wasnt ready to give since it was awaiting the
adjusters report.
The company then made an excuse that the insured had not filed
any claim with it, nor submitted proof of loss which is a clear
violation of Policy Condition No.11, as a result, determination of the
liability of private respondent could not be made.
Pacific Banking filed in the trial court an action for a sum of money
for P61,000.00 against Oriental Assurance.

At the trial, petitioner presented communications of the insurance


adjuster to Asian Surety revealing undeclared co-insurances with
the following: P30,000 with Wellington Insurance; P25,000 with
Empire Surety and P250,000 with Asian Surety undertaken by
insured Paramount on the same property covered by its policy with
Oriental whereas the only co-insurances declared in the subject
policy are those of P30,000.00 with Malayan P50,000.00 with South
Sea and P25.000.00 with Victory.
The defense of fraud, in the form of non-declaration of coinsurances which was not pleaded in the answer, was also not
pleaded in the Motion to Dismiss.
The trial court denied the respondents motion. Oriental filed
another motion to include additional evidence of the co-insurance
which could amount to fraud.
The trial court still made Oriental liable for P 61,000. The CA
reversed the trial court decision. Pacific Banking filed a motion for
reconsideration of the said decision of the respondent Court of
Appeals, but this was denied for lack of merit.
Issues:
1. WON unrevealed co-insurances Violated policy conditions No. 3
2. WON the insured failed to file the required proof of loss prior to
court action.
Held: Yes. Petition dismissed.
Ratio:
1. Policy Condition No. 3 explicitly provides:
3. The Insured shall give notice to the Company of any insurance
already effected, or which may subsequently be effected, covering
any of the property hereby insured, and unless such notice be given
Page 1 of 154

INSURANCE G01 CASE DIGESTS


and the particulars of such insurance or insurances be stated in or
endorsed on this Policy by or on behalf of the Company before the
occurrence of any loss or damage, all benefit under this policy shall
be forfeited.
The insured failed to reveal before the loss three other insurances.
Had the insurer known that there were many co-insurances, it could
have hesitated or plainly desisted from entering into such contract.
Hence, the insured was guilty of clear fraud.
Concrete evidence of fraud or false declaration by the insured was
furnished by the petitioner itself when the facts alleged in the policy
under clauses "Co-Insurances Declared" and "Other Insurance
Clause" are materially different from the actual number of coinsurances taken over the subject property.
As the insurance policy against fire expressly required that notice
should be given by the insured of other insurance upon the same
property, the total absence of such notice nullifies the policy.
Petitioner points out that Condition No. 3 in the policy in relation to
the "other insurance clause" supposedly to have been violated,
cannot certainly defeat the right of the petitioner to recover the
insurance as mortgagee/assignee. Hence, they claimed that the
purpose for which the endorsement or assignment was made was
to protect the mortgagee/assignee against any untoward act or
omission of the insured. It would be absurd to hold that petitioner is
barred from recovering the insurance on account of the alleged
violation committed by the insured.
It is obvious that petitioner has missed all together the import of
subject mortgage clause which specifically provides:
Loss, if any, under this policy, shall be payable to the PACIFIC
BANKING CORPORATION Manila mortgagee/trustor as its interest
may appear, it being hereby understood and agreed that this
insurance as to the interest of the mortgagee/trustor only herein,

shall not be invalidated by any act or neglectexcept fraud or


misrepresentation, or arsonof the mortgagor or owner/trustee of
the property insured; provided, that in case the mortgagor or
owner/ trustee neglects or refuses to pay any premium, the
mortgagee/ trustor shall, on demand pay the same.
The paragraph clearly states the exceptions to the general rule that
insurance as to the interest of the mortgagee, cannot be
invalidated; namely: fraud, or misrepresentation or arson.
Concealment of the aforecited co-insurances can easily be fraud, or
in the very least, misrepresentation.
Undoubtedly, it is but fair and just that where the insured who is
primarily entitled to receive the proceeds of the policy has by its
fraud and/or misrepresentation, forfeited said right.
Petitioner further stressed that fraud which was not pleaded as a
defense in private respondent's answer or motion to dismiss, should
be deemed to have been waived. It will be noted that the fact of
fraud was tried by express or at least implied consent of the parties.
Petitioner did not only object to the introduction of evidence but on
the contrary, presented the very evidence that proved its existence.
2. Generally, the cause of action on the policy accrues when the loss
occurs, But when the policy provides that no action shall be brought
unless the claim is first presented extrajudicially in the manner
provided in the policy, the cause of action will accrue from the time
the insurer finally rejects the claim for payment
In the case at bar, policy condition No. 11 specifically provides that
the insured shall on the happening of any loss or damage give
notice to the company and shall within fifteen (15) days after such
loss or damage deliver to the private respondent (a) a claim in
writing giving particular account as to the articles or goods
destroyed and the amount of the loss or damage and (b) particulars
of all other insurances, if any.
Page 2 of 154

INSURANCE G01 CASE DIGESTS


Twenty-four days after the fire did petitioner merely wrote letters
to private respondent to serve as a notice of loss. It didnt even
furnish other documents. Instead, petitioner shifted upon private
respondent the burden of fishing out the necessary information to
ascertain the particular account of the articles destroyed by fire as
well as the amount of loss. Since the required claim by insured,
together with the preliminary submittal of relevant documents had
not been complied with, it follows that private respondent could
not be deemed to have finally rejected petitioner's claim and
therefore there was no cause of action.
It appearing that insured has violated or failed to perform the
conditions under No. 3 and 11 of the contract, and such violation or
want of performance has not been waived by the insurer, the
insured cannot recover, much less the herein petitioner.

69.

ORIENTAL ASSURANCE V . COURT OF APPEALS, 200 SCRA 459


(1991)

ORIENTAL ASSURANCE v. CA (PANAMA SAW MILL)


200 SCRA 459
MELENCIO-HERRERA; August 9, 1991
NATURE
Petition for review on certiorari
FACTS
- Sometime in January 1986, private respondent Panama Sawmill
Co., Inc. (Panama) bought, in Palawan, 1,208 pieces of apitong logs,
with a total volume of 2,000 cubic meters. It hired Transpacific
Towage, Inc., to transport the logs by sea to Manila and insured it
against loss for P1-M with petitioner Oriental Assurance
Corporation (Oriental Assurance).
- While the logs were being transported, rough seas and strong
winds caused damage to one of the two barges resulting in the loss
of 497 pieces of logs out of the 598 pieces loaded thereon.
- Panama demanded payment for the loss but Oriental Assurance
refuse on the ground that its contracted liability was for "TOTAL
LOSS ONLY."
- Unable to convince Oriental Assurance to pay its claim, Panama
filed a Complaint for Damages against Oriental Assurance before
the Regional Trial Court.
Page 3 of 154

INSURANCE G01 CASE DIGESTS


- RTC ordered Oriental Assurance to pay Panama with the view that
the insurance contract should be liberally construed in order to
avoid a denial of substantial justice; and that the logs loaded in the
two barges should be treated separately such that the loss
sustained by the shipment in one of them may be considered as
"constructive total loss" and correspondingly compensable. CA
affirmed in toto.
ISSUE
WON Oriental Assurance can be held liable under its marine
insurance policy based on the theory of a divisible contract of
insurance and, consequently, a constructive total loss
HELD
NO
- The terms of the contract constitute the measure of the insurer
liability and compliance therewith is a condition precedent to the
insured's right to recovery from the insurer. Whether a contract is
entire or severable is a question of intention to be determined by
the language employed by the parties. The policy in question shows
that the subject matter insured was the entire shipment of 2,000
cubic meters of apitong logs. The fact that the logs were loaded on
two different barges did not make the contract several and divisible
as to the items insured. The logs on the two barges were not
separately valued or separately insured. Only one premium was
paid for the entire shipment, making for only one cause or
consideration. The insurance contract must, therefore, be
considered indivisible.

- More importantly, the insurer's liability was for "total loss only." A
total loss may be either actual or constructive (Sec. 129, Insurance
Code). An actual total loss is caused by:
(a) A total destruction of the thing insured;
(b) The irretrievable loss of the thing by sinking, or by being
broken up;
(c) Any damage to the thing which renders it valueless to the
owner for the purpose for which he held it; or
(d) Any other event which effectively deprives the owner of the
possession, at the port of destination, of the thing insured.
(Section 130, Insurance Code).
- A constructive total loss is one which gives to a person insured a
right to abandon, under Section 139 of the Insurance Code. This
provision reads:
SECTION 139. A person insured by a contract of marine insurance
may abandon the thing insured, or any particular portion thereof
separately valued by the policy, or otherwise separately insured,
and recover for a total loss thereof, when the cause of the loss is a
peril injured against,
(a) If more than three-fourths thereof in value is actually lost, or
would have to be expended to recover it from the peril;
(b) If it is injured to such an extent as to reduce its value more
than three-fourths;
xxx xxx xxx
Page 4 of 154

INSURANCE G01 CASE DIGESTS


- The requirements for the application of Section 139 of the
Insurance Code, quoted above, have not been met. The logs
involved, although placed in two barges, were not separately valued
by the policy, nor separately insured. Resultantly, the logs lost in the
damaged barge in relation to the total number of logs loaded on the
same barge cannot be made the basis for determining constructive
total loss. The logs having been insured as one inseparable unit, the
correct basis for determining the existence of constructive total loss
is the totality of the shipment of logs. Of the entirety of 1,208,
pieces of logs, only 497 pieces thereof were lost or 41.45% of the
entire shipment. Since the cost of those 497 pieces does not exceed
75% of the value of all 1,208 pieces of logs, the shipment cannot be
said to have sustained a constructive total loss under Section 139(a)
of the Insurance Code.
Disposition judgment under review is SET ASIDE

70.

FORTUNE ASSURANCE V . COURT OF APPEALS, 244 SCRA 308


(1995)

G.R. No. 115278 May 23, 1995


Petitioner: FORTUNE INSURANCE AND SURETY CO., INC. (Fortune)
Respondent: PRODUCERS BANK OF THE PHILIPPINES (PBP)
FACTS:
> PBP filed against Fortune a complaint for recovery of the sum of
P725,000.00 under the policy issued by Fortune. The money was
allegedly lost during a robbery of Producer's armored vehicle while
it was in transit to transfer the money from its Pasay City Branch to
its head office in Makati along Taft Avenue.
>The armored car was driven by Benjamin Magalong escorted by
Security Guard Saturnino Atig.
>Driver Magalong was assigned by PRC Management Systems with
the PBP by virtue of an Agreement and Atiga was assigned by
Unicorn Security Services, Inc. by virtue of a contract of Security
Service.
>After an investigation conducted by the Pasay police authorities,
the driver Magalong and guard Atiga were charged, together with
Edelmer Bantigue, Reynaldo Aquino and John Doe, with violation of
P.D. 532 (Anti-Highway Robbery Law) before the Fiscal of Pasay City.
>Demands were made by PBP but Fortune refused to pay as the loss
is excluded from the coverage of the insurance policy which is
stipulated under "General Exceptions" Section (b) which reads as
follows:
Page 5 of 154

INSURANCE G01 CASE DIGESTS


GENERAL EXCEPTIONS
The company shall not be liable under this policy in
report of
xxx xxx xxx
(b) any loss caused by any dishonest, fraudulent or
criminal act of the insured or any officer, employee,
partner, director, trustee or authorized
representative of the Insured whether acting alone
or in conjunction with others. . . .
8. The plaintiff opposes the contention of the
defendant and contends that Atiga and Magalong
are not its "officer, employee, . . . trustee or
authorized representative . . . at the time of the
robbery.
>RTC & CA: held that there should be recovery. The trial court ruled
that Magalong and Atiga were not employees or representatives of
Producers. The wages and salaries of both Magalong and Atiga are
presumably paid by their respective firms, which alone wields the
power to dismiss them. Neither is the Court prepared to accept the
proposition that driver Magalong and guard Atiga were the
"authorized representatives" of plaintiff.
ISSUE:
W/N the recovery in the policy is precluded under the general
exceptions clause?
HELD:

Fortune is exempt from liability under the general exceptions clause


of the insurance policy.
>It should be noted that the insurance policy entered into by the
parties is a theft or robbery insurance policy which is a form of
casualty insurance (Section 174 of the Insurance Code). Other than
what is mentioned in the provision, the rights and obligations of the
parties must be determined by the terms of their contract, taking
into consideration its purpose and always in accordance with the
general principles of insurance law.
>The purpose of the exception is to guard against liability should the
theft be committed by one having unrestricted access to the
property. In such cases, the terms specifying the excluded classes
are to be given their meaning as understood in common
speech. The terms "service" and "employment" are generally
associated with the idea of selection, control, and compensation.
>A contract of insurance is a contract of adhesion, thus any
ambiguity therein should be resolved against the insurer, or it
should be construed liberally in favor of the insured and strictly
against the insurer. Limitations of liability should be regarded with
extreme jealousy and must be construed
in such a way, as to preclude the insurer from non-compliance with
its obligation.
>If the terms of the contract are clear and unambiguous, there is no
room for construction and such terms cannot be enlarged or
diminished by judicial construction.
>An insurance contract is a contract of indemnity. It is settled that
the terms of the policy constitute the measure of the insurer's
liability. In the absence of statutory prohibition to the contrary,
insurance companies have the same rights as individuals to limit

YES.
Page 6 of 154

INSURANCE G01 CASE DIGESTS


their liability and to impose whatever conditions they deem best
upon their obligations not inconsistent with public policy.
>It was clear that Fortunes intention is to exclude and exempt from
protection and coverage losses arising from dishonest, fraudulent,
or criminal acts of persons granted or having unrestricted access to
Producers' money or payroll. When it used then the term
"employee," it must have had in mind any person who qualifies as
such as generally and universally understood, or jurisprudentially
established in the light of the four standards in the determination of
the employer-employee relationship, or as statutorily declared even
in a limited sense as in the case of Article 106 of the Labor Code
which considers the employees under a "labor-only" contract as
employees of the party employing them and not of the party who
supplied them to the employer.
>Fortune claims that Producers' contracts with PRC Management
Systems and Unicorn Security Services are "labor-only" contracts.
But even granting for the sake of argument that these contracts
were not "labor-only" contracts, and PRC Management Systems and
Unicorn Security Services were truly independent contractors, we
are satisfied that Magalong and Atiga were, in respect of the
transfer of Producer's money from its Pasay City branch to its head
office in Makati, its "authorized representatives" who served as
such with its teller Maribeth Alampay.
>Producers entrusted the three with the specific duty to safely
transfer the money to its head office, with Alampay to be
responsible for its custody in transit; Magalong to drive the armored
vehicle which would carry the money; and Atiga to provide the
needed security for the money, the vehicle, and his two other
companions. In short, for these particular tasks, the three acted as
agents of Producers. A "representative" is defined as one who
represents or stands in the place of another; one who represents

others or another in a special capacity, as an agent, and is


interchangeable with "agent."

71.

GREAT PACIFIC LIFE V . COURT OF APPEALS, 89 SCRA 543 (1979)

GREAT PACIFIC LIFE v. CA (NGO HING)


89 SCRA 543
DE CASTRO, J; April 30, 1979
!
NATURE
Petition for certiorari
!
FACTS
- On March 14, 1957, private respondent Ngo Hing filed an
application with the Great Pacific Life Assurance Co. (Pacific Life)
for a 20 year endowment policy of P50k on the life of his 1
year old daughter, Helen. Ngo Hing supplied the essetntial data
which petitioner Mondragon, branch manager of the Pacific Life
in Cebu, wrote on the corresponding form in his own
handwriting, later typing the data on an application form signed by
Ngo Hing. The latter paid the P1077.75 annual premium but
retained P1,317 as commission as he was also a duly
authorized agent of Pacific Life. The binding deposit receipt
was then issued to Ngo Hing;
Mondragon handwrote his strong recommendation for the
approval of the application on the
back of the form.
- On April 30, Mondragon received a letter from Pacific Life
which stated that the 20 year

Page 7 of 154

INSURANCE G01 CASE DIGESTS


endowment plan was not available for minors below 7, but that
Pacific Life could consider the same under the Juvenile Triple
Action Plan, advising that if the offer was acceptable, the
Juvenile Non-Medical Declaration be sent to the company.
-Mondragon allegedly failed to inform Ngo Hing of the nonacceptance of the insurance plan, instead writing Pacific Life
again, recommending the approval of the endowment plan to
children since customers had been asking for such coverage since
1954.
-On May 28, 1957, Helen died of influenza. Ngo Hing sought the
payment of the proceeds of the insurance, but having failed to do
so, filed an action for recovery with the CFI of Cebu. The Court
ordered Pacific Life to pay P50k with 6% interest, hence this
petition.

-This implies the receipt is merely an acknowledgement, on


behalf of the company, that the Cebu branch of Pacific Life had
received the premium and had accepted the application subject to
processing by the insurance company, which will approve or reject it
depending on whether the applicant is insurable on standard rates.
As such, the receipt was never in force-it does not insure outright.
No liability attaches until the principal approves the risk and a
receipt is given by the agent; because private respondent failed
to accept Pacific Life's offer for the Juvenile Triple Action plan,
there was no meeting of the minds and thus no contract. Also,
being an authorized agent of Pacific Life, Ngo Hing must have
known the company did not offer the insurance applied for and
merely took a chance on Mondragon's recommendation.
Disposition the decision appealed from is set aside, absolving
Pacific Life from their civil liabilities

!
ISSUE
WON the binding deposit receipt constituted a temporary
contract of the life insurance in question
!
HELD
NO
- The binding deposit receipt is merely a provisional contract
and only upon compliance with the ff conditions: (1) that the
company be satisfied that the applicant was insurable on
standard rates (2) that if the company does not accept the
application and offers a different policy, the insurance contract
shall not be binding until the applicant accepts the new policy
(3) that if the applicant is not found to be insurable on standard
rates and the application is disapproved, the insurance shall not be
in force at any time and the premium be returned to the applicant.

Page 8 of 154

INSURANCE G01 CASE DIGESTS


72.

BONIFACIO BROTHERS V . MORA, 20 SCRA 261 (1967)

FACTS:

Enrique Mora, owner of Oldsmobile sedan model 1956


mortgaged the same to the H.S. Reyes, Inc., with the
condition that the former would insure the automobile
with the latter as beneficiary.
The automobile was thereafter insured with the State
Bonding & Insurance Co., Inc., and motor car insurance
policy was issued to Enrique Mora.
During the effectivity of the insurance contract, the car met
with an accident. Enrique Mora, without the knowledge
and consent of the H.S. Reyes, Inc., authorized the
Bonifacio Bros. Inc. to furnish the labor and materials,
some of which were supplied by the Ayala Auto Parts Co.
For the cost of labor and materials, Enrique Mora was
billed at P2,102.73 through the H.H. Bayne Adjustment Co.
The insurance company after claiming a franchise in the
amount of P100, drew a check in the amount of P2,002.73,
as proceeds of the insurance policy, payable to the order
of Enrique Mora or H.S. Reyes,. Inc., and entrusted the
check to the H.H. Bayne Adjustment Co. for disposition and
delivery to the proper party.
In the meantime, the car was delivered to Enrique Mora
without the consent of the H.S. Reyes, Inc., and without
payment to the Bonifacio Bros. Inc. and the Ayala Auto
Parts Co. of the cost of repairs and materials.
Upon the theory that the insurance proceeds should be
paid directly to them, the Bonifacio Bros. Inc. and the
Ayala Auto Parts Co. filed a complaint with the MTC of
Manila against Enrique Mora and the State Bonding &
Insurance Co., Inc. for the collection of the sum of
P2,002.73.

The insurance company filed its answer with a


counterclaim for interpleader, requiring the Bonifacio
Bros. Inc. and the H.S. Reyes, Inc. to interplead in order to
determine who has better right to the insurance proceeds
in question.
Municipal Court rendered a decision declaring the H.S.
Reyes, Inc. as having a better right to the disputed amount
and ordering State Bonding & Insurance Co. Inc. to pay to
the H. S. Reyes, Inc. the said sum of P2,002.73.
From this decision, the appellants elevated the case to the
CFI of Manila which rendered a decision, affirming the
decision of the Municipal Court. The Bonifacio Bros. Inc.
and the Ayala Auto Parts Co. moved for reconsideration of
the decision, but the trial court denied the motion. Hence,
this appeal.

ISSUE: WON there is privity of contract between the Bonifacio Bros.


Inc. and the Ayala Auto Parts Co. on the one hand and the insurance
company on the other.
HELD: NONE
Appellant = Bonifacio Bro., Inc.
From the undisputed facts and from the pleadings it will be seen
that the appellants' alleged cause of action rests exclusively upon
the terms of the insurance contract. The appellants seek to recover
the insurance proceeds, and for this purpose, they rely upon
paragraph 4 of the insurance contract document executed by and
between the State Bonding & Insurance Company, Inc. and Enrique
Mora. The appellants are not mentioned in the contract as parties
thereto nor is there any clause or provision thereof from which we
can infer that there is an obligation on the part of the insurance
company to pay the cost of repairs directly to them.
Page 9 of 154

INSURANCE G01 CASE DIGESTS


It is fundamental that contracts take effect only between the
parties thereto, except in some specific instances provided by law
where the contract contains some stipulation in favor of a third
person.1Such stipulation is known as stipulation pour autrui or a
provision in favor of a third person not a pay to the contract. Under
this doctrine, a third person is allowed to avail himself of a benefit
granted to him by the terms of the contract, provided that the
contracting parties have clearly and deliberately conferred a favor
upon such person.

expressed or implied exists. We, therefore, agree with the trial court
that no cause of action exists in favor of the appellants in so far as
the proceeds of insurance are concerned. The appellants' claim, if
at all, is merely equitable in nature and must be made effective
through Enrique Mora who entered into a contract with the
Bonifacio Bros. Inc. This conclusion is deducible not only from the
principle governing the operation and effect of insurance contracts
in general, but is clearly covered by the express provisions of
section 50 of the Insurance Act which read:

In this connection, this Court has laid down the rule that the fairest
test to determine whether the interest of a third person in a
contract is a stipulation pour autrui or merely an incidental
interest, is to rely upon the intention of the parties as disclosed by
their contract.4 In the instant case the insurance contract does not
contain any words or clauses to disclose an intent to give any
benefit to any repairmen or materialmen in case of repair of the
car in question. The parties to the insurance contract omitted such
stipulation, which is a circumstance that supports the said
conclusion. On the other hand, the "loss payable" clause of the
insurance policy stipulates that "Loss, if any, is payable to H.S.
Reyes, Inc." indicating that it was only the H.S. Reyes, Inc. which
they intended to benefit.

The insurance shall be applied exclusively to the proper


interests of the person in whose name it is made unless
otherwise specified in the policy.

If it were the intention of the insurance company to make itself


liable to the repair shop or materialmen, it could have easily
inserted in the contract a stipulation to that effect.

The policy in question has been so framed that "Loss, if any, is


payable to H.S. Reyes, Inc.," which unmistakably shows the
intention of the parties.
The final contention of the appellants is that the right of the H.S.
Reyes, Inc. to the insurance proceeds arises only if there was loss
and not where there is mere damage as in the instant case. Suffice it
to say that any attempt to draw a distinction between "loss" and
"damage" is uncalled for, because the word "loss" in insurance law
embraces injury or damage. Indeed, according to sec. 120 of the
Insurance Act, a loss may be either total or partial.

Another cogent reason for not recognizing a right of action by the


appellants against the insurance company is that "a policy of
insurance is a distinct and independent contract between the
insured and insurer, and third persons have no right either in a
court of equity, or in a court of law, to the proceeds of it, unless
there be some contract of trust, expressed or implied between the
insured and third person."5 In this case, no contract of trust,
Page 10 of 154

INSURANCE G01 CASE DIGESTS


73.

HEIRS OF L.G. MARAMAG V . MARAMAG, 588 SCRA 774 (2009)

1. PETITIONERS were legitimate wife and children of Loreto


Maramag.
2. PETITIONERS' CONTENTION:
a. RESPONDENTS were Loreto''s illegitimate family.
b.Eva Maramag was concubine of Loreto and a suspect in the killing
of Loreto. Thus, she is disqualified to receive any proceeds from his
insurance policies from Insular Life Assurance (INSULAR) and Great
Pacific Life
Assurance (GREPALIFE)
c. The illegitimate children were entitled only to 1/2 of the legitime
of the legitimate children,
thus the proceeds released to the illegitimate children were
inofficious and should be
reduced.
d. x x x
3. TRO and writ of preliminary injunction were filed by the
petitioner because, some part of
the insurance proceeds had already been released in favor of one of
the illegitimate and the
rest are to be releases in favor of the other illegitimate.
4.Insular admitted that Loreto misrepresented Eva as his legitimate
wife and Odessa, Karl
Brian, and Trisha Angelie as his legitimate children, and that they
filed their claims for the
insurance proceeds of the insurance policies.
5. INSULAR ALLEGATION:

THE COMPLAINT OR PETITION FAILED TO STATE A CAUSE OF


ACTION AS TO
DECLARE AS VOID THE DESIGNATION OF EVA AS BENEFICIARY for
Loreto revoked her
designation and already disqualified her.
6. GREPALIFR CONTENTION: Eva was not designated as an insurance
policy beneficiary, that r claims of the illegitimate children were
denied because Loreto was ineligible for the insurance due to the
misrepresentation in his application form that he was not more
than 65 years old.
7.Both Insular and Grepalife countered that the insurance proceeds
belong exclusively to the
designated beneficiaries in the policies, not to the estate or to the
heirs of the insured.
Grepalife also reiterated that it had disqualified Eva as a beneficiary
when it ascertained that
Loreto was legally married to Vicenta Pangilinan Maramag.
8.RTC: In favor of the respondents. Neither could the plaintiffs
invoked (sic) the law on donations or the rules on testamentary
succession in order to defeat the right of herein defendants to
collect the insurance indemnity. The beneficiary in a contract of
insuranceis not the donee spoken in the law of donation. The rules
on testamentary succession cannot apply here, for the insurance
indemnity does not partake of a donation.
THE PROCEEDS BELONG EXCLUSIVELY THE BENEFICIARY AND NOT
TO THE ESTATE OF THE PERSON. NO SUFFICIENT CAUSE OF ACTION
AGAINST THE ILLEGITIMATE FLR THE REDUCTION AND/OR
DECLARATION OF INOFFICIOUS OF
DONATION AS PRIMARY BENEFICIARY. EVA AS THE CONCUBINE
CANNOT BE A
BENEFICIARY
Page 11 of 154

INSURANCE G01 CASE DIGESTS

9. CA: Affirmed the decision of the RTC. The distribution of the


insurance proceeds is governed primarily by the Insurance Code,
and the provisions of the Civil Code are irrelevant and inapplicable.
ISSUE:
Whether or not the legitimate children of the insured decease shall
receive the proceeds of
the insurance that originally designated to Eva?
HELD:
No. Even assuming Insular disqualified Eva as a beneficiary, her
share should not have been distributed to the legitimate heirs of
the insured deceased.
RATIONALE:
It is evident from the face of the complaint that petitioners are not
entitled to a favorable judgment in light of Article 2011 of the Civil
Code which expressly provides that insurance contracts shall be
governed by special laws, i.e., the Insurance Code. Section 53 of the
Insurance Code states

stipulations or indemnity. In such a case, third parties may directly


sue and claim from the insurer.
Petitioners are third parties to the insurance contracts with Insular
and Grepalife and, thus, are not entitled to the proceeds thereof.
Accordingly, respondents Insular and Grepalife have no legal
obligation to turn over the insurance proceeds to petitioners. The
revocation of Eva as a beneficiary in one policy and her
disqualification as such in another are of no moment considering
that the designation of the illegitimate children as beneficiaries in
Loreto's insurance policies remains valid. Because no legal
proscription exists in naming as beneficiaries the children of illicit
relationships by the insured,22 the shares of Eva in the insurance
proceeds, whether forfeited by the court in view of the prohibition
on donations under Article 739 of the Civil Code or by the insurers
themselves for reasons based on the insurance contracts, must be
awarded to the said illegitimate children, the designated
beneficiaries.

SECTION 53. The insurance proceeds shall be applied exclusively to


the proper interest of
the person in whose name or for whose benefit it is made unless
otherwise specified in the
policy.
Pursuant thereto, it is obvious that the only persons entitled to
claim the insurance proceeds are either the insured, if still alive; or
the beneficiary, if the insured is already deceased, upon the
maturation of the policy. The exception to this rule is a situation
where the insurance contract was intended to benefit third persons
who are not parties to the same in the form of favorable

Page 12 of 154

INSURANCE G01 CASE DIGESTS


74.

COQUIA V. FIELDMEN S INSURANCE , 26 SCRA 178 (1968)

Coquia vs Fieldmens Insurance


Facts:
1. Fieldmen's Insurance Company, Inc. issued, in favor of the
Manila Yellow Taxicab Co., Inc. a common carrier accident
insurance policy.
2. It was stipulated in said policy that the company will
indemnify the Insured in the event of accident caused by or
arising out of the use of Motor Vehicle against all sums
which the Insured will become legally liable to pay.
3. While the policy was in force, or on February 10, 1962, a
taxicab of the Insured, driven by Carlito Coquia, met a
vehicular accident at Mangaldan, Pangasinan, in
consequence of which Carlito died.
4. The Insured filed therefor a claim for P5,000.00 to which the
Company replied with an offer to pay P2,000.00, by way of
compromise.
5. The Insured rejected the same and made a counter-offer for
P4,000.00, but the Company did not accept it.
6. The insured and Carlitos parents or the Coquias filed a
complaint against the company to collect the proceeds of
the insurance policy.
7. As a defence, the company argued lack of cause of action on
the part of the Coquias.
8. RTC ruled for the plaintiffs sentencing the company to pay
4k.
Issue:
1. Whether or not the Coquias can claim under the policy even
if they are alleged to not have a cause of action against the
company as they are not parties to the insurance policy?

Held:
1. YES
2. While the general rule is that only parties to a contract may
bring an action based thereon, one exception is found
under Article 1311 of the Civil Code.
3. It provides that If a contract should contain some stipulation
in favor of a third person, he may demand its fulfillment
provided he communicated his acceptance to the obligor
before its revocation.
4. These are contracts pour autrui wherein enforcement of a
contract may be demanded by a third party for whose
benefit it was made, although not a party to the contract.
5. In this case, the policy contained a stipulation which states
the following: Section I Liability to Passengers. 1. The
Company will, subject to the Limits of Liability and under
the Terms of this Policy, indemnify the Insured in the event
of accident caused by or arising out of the use of Motor
Vehicle against all sums which the Insured will become
legally liable to pay in respect of: Death or bodily injury to
any fare-paying passenger including the Driver ... who is
riding in the Motor Vehicle insured at the time of accident
or injury.
6. Another stipulation provides that In the event of death of
any person entitled to indemnity under this Policy, the
Company will, in respect of the liability incurred by such
person, indemnify his personal representatives in terms of
and subject to the limitations of this Policy, provided, that
such representatives shall, as though they were the Insured,
observe, fulfill and be subject to the Terms of this Policy
insofar as they can apply.
7. Pursuant to these stipulations, the Company "will
indemnify any authorized Driver who is driving the Motor
Vehicle" of the Insured and, in the event of death of said
Page 13 of 154

INSURANCE G01 CASE DIGESTS


driver, the Company shall, likewise, "indemnify his personal
representatives."
8. Thus, the policy under consideration is typical of
contracts pour autrui, this character being made more
manifest by the fact that the deceased driver paid fifty
percent (50%) of the corresponding premiums, which were
deducted from his weekly commissions.
9. Under these conditions, it is clear that the Coquias who,
admittedly, are the sole heirs of the deceased have a
direct cause of action against the Company,3 and, since they
could have maintained this action by themselves, without
the assistance of the Insured, it goes without saying that
they could and did properly join the latter in filing the
complaint herein.

75.

LOPEZ V . DEL ROSARIO , 44 PHIL 98 (1922)

Lopez vs. Del Rosario (1922)


Facts:
1. Mrs. del Rosario owned a bonded warehouse in Manila.
Engaged in the business of a warehouse keeper, she stored
copra and other merchandise in building.
2. Among the persons who had copra deposited in the Del
Rosario warehouse was Froilan Lopez, the holder of
fourteen warehouse receipts in his own name, and the
name of Elias T. Zamora.
3. Lopez named a declared value of P107,990.40. The
warehouse receipts provided: (1) For insurance at the rate
of 1 per cent per month on the declared value; (2) the
company reserves to itself the right to raise and/or lower
the rates of storage and/or of insurance on giving one
calendar month's notice in writing; (3) this warrant carries
no insurance unless so noted on the face hereof, cost of
which is in addition to storage; (4) the time for which
storage and/or insurance is charged is thirty (30) days; (5)
payment for storage and/or insurance, etc., shall be made
in advance, and/or within five (5) days after presentation of
bill.
4. It is admitted that insurance was paid by Lopez to May 18,
1920, but not thereafter.
5. Mrs. Del Rosario secured insurance on the warehouse and
its contents with the National Insurance Co., Inc., the
Commercial Union Insurance Company, the Alliance
Insurance Company, the South British Insurance Co., Ltd.,
and the British Traders Insurance Co., Ltd.
6. The warehouse caught fire. Everything was destroyed. Only
copra worth P49,985 was salvaged.
7. Mrs. del Rosario was able to settle everything except the
account of Lopez.
Page 14 of 154

INSURANCE G01 CASE DIGESTS

Issue: Whether or not Mrs. Del Rosario acted as an agent of Lopez


in taking out an insurance on the contents of the warehouse
Held: YES. The agency can be deduced from the warehouse receipts,
the insurance policies, and the circumstances surrounding the
transaction. The law is that a policy effected by bailee and covering
by its terms his own property and property held in trust; inures, in
the event of a loss, equally and proportionately to the benefit of all
the owners of the property insured. Even if one secured insurance
covering his own goods and goods stored with him, and even if the
owner of the stored goods did not request or know of the
insurance, and did not ratify it before the payment of the loss, yet it
has been held by a reputable court that the warehouseman is liable
to the owner of such stored goods for his share.

Moreover, the Court found in two documents of Mrs. Del Rosario


against the insurance companies (agreement for arbitration and the
statement of claim) she acknowledged her responsibility to the
owners of the stored merchandise, against risk of loss by fire. The
award of the arbitrators covered not alone Mrs. Del Rosario's
warehouse but the products stored in the warehouse by Lopez and
others.

Lopez' rights to the insurance money have not been forfeited by


failure to pay the insurance provided for in the warehouse receipts.
A preponderance of the proof does not demonstrate that he ever
ordered the cancellation of his insurance with the Del Rosario. Nor
is it shown that the Lopez ever refused to pay the insurance when
the bills were presented to him, and that notice of an intention to
cancel the insurance was ever given to him. Lopez can recover from

the Del Rosario the sum of P81,093.65, with interest at 6 per cent
per annum from May 13, 1921, until paid.

76.

DEVELOPMENT BANK V, INTERMEDIATE APPELLATE COURT, 143


SCRA 62 (1986)

Development Insurance vs IAC


GR No. 71360 July 16, 1986
Facts:
A fire occurred in the building of Philippine Union Realty
Development Corporation (PURDC) and it sued for damages
from Devt Insurance based on an insurance contract. Devt
Insurance failed to answer on time and was declared in
default by the trial court. A judgment of default was
subsequently rendered on the strength of the evidence
submitted ex parte by PURDC, which was allowed full
recovery of its claim.
Devt Insuracne moved to lift the order of default, invoking
excusable neglect, and to vacate the judgment by default;
which was denied by the court. The IAC affirmed the
decision of the trial court.
The face value of the policy is P2,500,000. Devt Insurance is
claiming that since at the time of the fire, the building
insured was worth P5.8M, they can only be liable to the
extent of the proportion between the difference between
that amount and the face value, as against the total loss
sustained, which is P508,867; making them only liable for
only P67,629.31.

Issue:
Page 15 of 154

INSURANCE G01 CASE DIGESTS


Whether the insurer should be liable for the whole amount of the
loss

77.
TEAL MOTOR V . ORIENT INSURANCE , 59 PHIL. 809
Facts:

Ruling:
The insurer is liable for the whole amount.
The policy issued to PURDC is an open policy and is subject
to the express condition that in the event of loss, whether
total or partial, it is understood that the amount of the loss
shall be subject to appraisal and the liability of the
company, if established, shall be limited to the actual loss,
subject to the applicable terms, conditions, warranties and
clauses of the policy, and in no case shall exceed the
amount of the policy.
o An open policy is one in which the value of the thing
insured is not agreed upon but is left to be
ascertained in case of loss
There is no evidence on record that the building was worth
P5.8M at the time of the loss; only Devt Insurance says so,
and it does not back up its self-serving estimate with any
independent corroboration.
Since the building was insured at P2.5M, this must be
considered the value of the building on the day the fire
occurred.
The actual loss has been ascertained in this case and the
Court will respect such factual determination in the absence
of proof that it was arrived at arbitrarily. There is no such
showing.
Applying the open policy clause as expressly agreed upon by
the parties in their contract, the Court holds that PURDC is
entitled to the payment of indemnity in the total amount of
P508,867.

-These seven cases related to insurance policies covering the goods,


wares, and merchandise contained in the building in the Port Area
in the City of Manila which was damaged by a fire of unknown origin
the afternoon of Sunday, January 6, 1929.
-At the request of the insured, the companies gave additional time
for the filing of the claims of loss. These claims were definitely
rejected in writing by the insurance companies through their agents
on April 15, 1929.
- Among the special defenses of the insurance companies is one
based upon a clause in the policies which, with the exception of
those of the Atlas Assurance Company, Ltd., among other things
provides:

all

if the claim be made and rejected, and action or suit be not


commenced
within three months after such rejection, ...
under this Policy
shall be forfeited.

- While those cases were under advisement here, the Supreme


Court noticed that the provision relating to the Atlas policy reads:
if the claim be made and rejected and arbitration
proceedings
be not commenced in pursuance of the 18th
Condition of this
Policy within three months after such
rejection; all benefit under
this Policy shall be
forfeited.
- No such arbitration proceedings were instituted within the three
months' period.
Page 16 of 154

INSURANCE G01 CASE DIGESTS


- The seven suits were filed between the 3rd and the 15th day of
August, 1929, or more than three months after the rejection by the
defendant companies of plaintiff's claim.

78.

ANG V. FULTON FIRE INSURANCE , 2 SCRA 945 (1961)

G.R. No. L-15862

July 31, 1961

Issue:
Whether or NOT the claims were filed on time?
Ruling:
The Supreme Court held that the case was filed out of time. Plaintiff
was given such time as it deemed necessary to formulate and
present its claim of loss. That claim was investigated by the
adjusters for several months, and under the contract of insurance,
the insured had three months after rejection in which to bring suit.
The issues were virtually joined on the presentation of the claims
and their rejection by the companies in writing, and three months
thereafter is not an unreasonably short time to draft and file in
court an appropriate complaint on a contract of fire insurance.
Ratio:
A provision requiring presentation of claim within three months
after the fire, and the bringing of action within three months after
refusal of claim is valid.

PAULO ANG and SALLY C. ANG, plaintiffs-appellees,


vs.
FULTON FIRE INSURANCE CO., ET AL., defendants.
FULTON FIRE INSURANCE CO., defendant-appellant.
LABRADOR, J.:
FACTS

September 9, 1953 - Fulton Fire Insurance Company issued


a fire insurance policy in favor of P. & S Department Store
(owned by the Spouses Paulo Ang and Sally C. Ang) over
stocks of general merchandise, consisting principally of dry
goods, contained in a building occupied by the Angs at
Laoag, Ilocos Norte. The premium is P500.00 annually.

September 31, 1954 policy was renewed for another year.

December 17, 1954 - the store was destroyed by fire.

December 30, 1954 - the Angs executed the first claim form
together with all the necessary papers (books of accounts of
the insured for the year 1953-1954 and a clearance from
the Philippine Constabulary and the police), and they were
all forwarded to the Manila Adjustment Company, Fulton's
adjusters.

January 13, 1955 - Paulo Ang and 10 others were charged


for arson in a Criminal Case but Paulo Ang was eventually
acquitted.

April 6, 1956 - Fulton denied the claim.


Page 17 of 154

INSURANCE G01 CASE DIGESTS

additional sum of P2,000.00 as attorney's fees, and costs).


The CFI held that the bringing of the action May 11, 1956,
tolled the running of the 12 month period.Fulton appealed
directly to the Supreme Court.

April 19, 1956 - denial received by Angs.

May 11, 1956 Angs filed 1 case to assert the claim but
against Paramount Surety and Insurance Company (Fultons
agent).

September 3, 1957 - 1st case was dismissed without


prejudice.

May 5, 1958 Angs filed 2nd case against Fulton and


Paramount Surety to recover from them the face value of
the policy, but Paramount was eventually dropped from the
complaint.

Whether the Angs may validly claim on the policy even with the
prohibition on Paragraph 13 of the policy.

May 26, 1958 ANSWER OF FULTON: (1) denied that the


loss by the fire was accidental, alleging that it was
occasioned by the willful act of Paulo Ang himself.

NO.

st

(2) claimed that under paragraph


13 of the policy, if the loss or damage is
occasioned by the willful act of the
insured, or if the claim is made and rejected
but no action is commenced within 12
months after such rejection, all benefits
under the policy would be forfeited. Since
Angs received notice of denial on April 18,
1956, and they filed action only on May 5,
1958, all the benefits under the policy have
been forfeited.

February 12, 1959 REPLY OF ANGS: The 1st case was filed
May 11, 1956 but was dismissed without prejudice on
September 3, 1957. That period between May 11, 1956 to
September 3, 1957 must be deducted from the prescriptive
period of 12 months.
CFI 2nd case in favor of Angs (ordering Fulton to pay the
Angs the sum of P10,000.00, with interest, and an

ISSUE

SC HELD

The basic error committed by the trial court is its view that the filing
of the action against the agent of the defendant company was
"merely a procedural mistake of no significance or consequence,
which may be overlooked." The condition contained in the
insurance policy that claims must be presented within one year
after rejection is not merely a procedural requirement. The
condition is an important matter, essential to a prompt settlement
of claims against insurance companies, as it demands that insurance
suits be brought by the insured while the evidence as to the origin
and cause of destruction have not yet disappeared. It is in the
nature of a condition precedent to the liability of the insurer, or in
other terms, a resolutory cause, the purpose of which is to
terminate all liabilities in case the action is not filed by the insured
within the period stipulated.
The bringing of the action against the Paramount Surety &
Insurance Company, the agent of the defendant Company cannot
have any legal effect except that of notifying the agent of the claim.
Beyond such notification, the filing of the action can serve no other
purpose. There is no law giving any effect to such action upon the
principal. Besides, there is no condition in the policy that the action
Page 18 of 154

INSURANCE G01 CASE DIGESTS


must be filed against the agent, and this Court can not by
interpretation, extend the clear scope of the agreement beyond
what is agreed upon by the parties.

79.

The case of E. Macias & Co. vs. China Fire Insurance Co. has settled
the issue presented by the appellees in the case at bar definitely
against their claim. In that case, We declared that the contractual
station in an insurance policy prevails over the statutory
limitation, as well as over the exceptions to the statutory limitations
that the contract necessarily supersedes the statute (of limitations)
and the limitation is in all phases governed by the former. (E. Macias
& Co. vs. China Fire Insurance & Co., 46 Phil. pp. 345-353). As stated
in said case and in accordance with the decision of the Supreme
Court of the United States in Riddlesbarger vs. Hartford Fire
Insurance Co. (7 Wall., 386), the rights of the parties flow from the
contract of insurance, hence they are not bound by the statute of
limitations nor by exemptions thereto. In the words of our own
law, their contract is the law between the parties, and their
agreement that an action on a claim denied by the insurer must be
brought within one year from the denial, governs, not the rules on
the prescription of actions.

Emilio Tan took from Sun Insurance Office a P300,000.00 property


insurance policy to cover his interest in the electrical supply store of
his brother. Four days after the issuance of the policy, the building
was burned including the insured store. On August 20, 1983, Tan
filed his claim for fire loss with Sun Insurance Office, but on
February 29, 1984, Sun Insurance Office wrote Tan denying the
latters claim. On April 3, 1984, Tan wrote Sun Insurance Office,
seeking reconsideration of the denial of his claim. Sun Insurance
Office answered the letter, advising Tans counsel that the Insurers
denial of Tans claim remained unchanged.

The judgment appealed from is hereby set aside and the case
dismissed, with costs against the plaintiffs-appellees.
APPEAL GRANTED.

SUN I NSURANCE OFFICE V . CA, 195 SCRA 193 (1991)

FACTS:

ISSUES:
(1)WON the filing of a motion for reconsideration interrupts the 12
months prescriptive period to contest the denial of the insurance
claim; and (2)WON the rejection of the claim shall be deemed final
only of it contains words to the effect that the denial is final;
HELD:
(1) No. In this case, Condition 27 of the Insurance Policy of the
parties reads:
27. Action or suit clause - If a claim be made and rejected and an
action or suit be not commenced either in the Insurance
Commission or in any court of competent jurisdiction within twelve
(12) months from receipt of notice of such rejection, or in case of
arbitration taking place as provided herein, within twelve (12)
Page 19 of 154

INSURANCE G01 CASE DIGESTS


months after due notice of the award made by the arbitrator or
arbitrators or umpire, then the claim shall for all purposes be
deemed to have been abandoned and shall not thereafter be
recoverable hereunder.
As the terms are very clear and free from any doubt or ambiguity
whatsoever, it must be taken and understood in its plain, ordinary
and popular sense.
Tan, in his letter addressed to Sun Insurance Office dated April 3,
1984, admitted that he received a copy of the letter of rejection on
April 2, 1984. Thus, the 12-month prescriptive period started to run
from the said date of April 2, 1984, for such is the plain meaning and
intention of Section 27 of the insurance policy.
The condition contained in an insurance policy that claims must be
presented within one year after rejection is not merely a procedural
requirement but an important matter essential to a prompt
settlement of claims against insurance companies as it demands
that insurance suits be brought by the insured while the evidence as
to the origin and cause of destruction have not yet disappeared.
It is apparent that Section 27 of the insurance policy was stipulated
pursuant to Section 63 of the Insurance Code, which states that:
Sec. 63. A condition, stipulation or agreement in any policy of
insurance, limiting the time for commencing an action thereunder
to a period of less than one year from the time when the cause of
action accrues, is void.

It also begs to ask, when does the cause of action accrue? The
insureds cause of action or his right to file a claim either in the
Insurance Commission or in a court of competent jurisdiction
commences from the time of the denial of his claim by the Insurer,
either expressly or impliedly. But the rejection referred to should be
construed as the rejection in the first instance (i.e. at the first
occasion or for the first time), not rejection conveyed in a resolution
of a petition for reconsideration. Thus, to allow the filing of a
motion for reconsideration to suspend the running of the
prescriptive period of twelve months, a whole new body of rules on
the matter should be promulgated so as to avoid any conflict that
may be brought by it, such as:
a.whether the mere filing of a plea for reconsideration of a denial is
sufficient or must it be supported by arguments/affidavits/material
evidence;
b.how many petitions for reconsideration should be permitted?
(2) No. The Eagle Star case cited by Tan to defend his theory that
the rejection of the claim shall be deemed final only of it contains
words to the effect that the denial is final is inapplicable in the
instant case. Final rejection or denial cannot be taken to mean the
rejection of a petition for reconsideration. The Insurance policy in
the Eagle Star case provides that the insured should file his claim,
first, with the carrier and then with the insurer. The final rejection
being referred to in said case is the rejection by the insurance
company.

Page 20 of 154

INSURANCE G01 CASE DIGESTS


80.

PACIFIC BANKING CORP. V CA, 168 SCRA 1 (1988)

PACIFIC BANKING CORPORATION vs. CA


No. L-41014; November 29, 1988

Facts:
On October 21, 1963, an Open Fire Policy was issued to the
Paramount Shirt Manufacturing Co. (insured) by which private
respondent Oriental Assurance bound itself to indemnify the
insured for any los or damage, not exceeding P61,000.00, caused
by fire to its property consisting of stocks, materials and supplies
usual to a shirt factory while contained in the first to third floors
of the building where they are located for a period of one year
starting October 21, 1964.
At the time the policy was issued, Paramount Shirt was a debtor
of Pacific Bank amounting to P800,000.00. Goods in the
said policy were held in trust by Paramount for Pacific Bank
under trust receipts. Said policy was duly endorsed to the
petitioner bank as mortgagee/trustor of the properties insured,
with the knowledge and consent of private respondent to the
effect that loss if any under this policy is payable to the (Pacific
Bank)".
While the aforesaid policy was in full fore and effect, a fire broke
out on the subject premises destroying the goods contained in
its ground and second floors. Pacific Bank sent a letter of
demand to Oriental Assurance for indemnity, but the latter
wasnt ready to give since it was awaiting the adjusters report. It

then made an excuse that the insured had not filed any claim
with it, nor submitted proof of loss which is a clear violation of
Policy Condition No.11, as a result, determination of the liability
of private respondent could not be made.
Pacific Banking filed in the trial court an action for a sum of
money for P61,000.00 against Oriental Assurance. At the trial,
Pacific Bank presented communications of the insurance
adjuster to Asian Surety revealing undeclared co-insurances with
the following: P30,000 with Wellington Insurance; P25,000 with
Empire Surety and P250,000 with Asian Surety undertaken by
insured Paramount on the same property covered by its policy
with Oriental whereas the only co-insurances declared in the
subject policy are those of P30,000.00 with Malayan P50,000.00
with South Sea and P25.000.00 with Victory.
The defense of fraud, in the form of non-declaration of coinsurances which was not pleaded in the answer, was also not
pleaded in the Motion to Dismiss. The trial court denied the
respondents motion. Oriental filed another motion to include
additional evidence of the co-insurance which could amount to
fraud. The trial court rendered judgment making Oriental
Assurance liable for P61,000.00, but the Court of Appeals
reversed the RTC decision.

Issues:
1. Whether or not the unrevealed co-insurances violated policy
conditions no. 3?
2. Whether or not the insured failed to file the required proof of
loss prior to court action?

Page 21 of 154

INSURANCE G01 CASE DIGESTS


Held and Ratio:
1. YES. Policy Condition 3 provides that the insured must give
notice of any insurance already in effect or subsequently be in
effect covering same property being insured. Failure to do so,
the policy shall be forfeited. Failure to reveal before the loss of
the 3 other insurances is a clear misrepresentation or a false
declaration. The material fact was asked for but was not
revealed. Representations of facts are the foundations of the
contract. Pacific itself provided for the evidences in trial court
that proved existence of misrepresentation.

2. YES. Generally, the cause of action on the policy accrues when


the loss occurs, But when the policy provides that no action shall
be brought unless the claim is first presented extrajudicially in
the manner provided in the policy, the cause of action will accrue
from the time the insurer finally rejects the claim for payment.

In the case at bar, policy condition No. 11 specifically provides


that the insured shall on the happening of any loss or damage
give notice to the company and shall within fifteen (15) days
after such loss or damage deliver to the private respondent (a) a
claim in writing giving particular account as to the articles or
goods destroyed and the amount of the loss or damage and (b)
particulars of all other insurances, if any. Likewise, insured was
required "at his own expense to produce, procure and give to
the company all such further particulars, plans, specifications,
books, vouchers, invoices, duplicates or copies thereof,
documents, proofs and information with respect to the claim".

The evidence adduced shows that twenty-four (24) days after


the fire, petitioner merely wrote letters to private respondent to
serve as a notice of loss, thereafter, the former did not furnish
the latter whatever pertinent documents were necessary to
prove and estimate its loss. Instead, petitioner shifted upon
private respondent the burden of fishing out the necessary
information to ascertain the particular account of the articles
destroyed by fire as well as the amount of loss. It is noteworthy
that private respondent and its adjuster notified petitioner that
insured had not yet filed a written claim nor submitted the
supporting documents in compliance with the requirements set
forth in the policy. Despite the notice, the latter remained
unheedful.

Since the required claim by insured, together with the


preliminary submittal of relevant documents had not been
complied with, it follows that private respondent could not be
deemed to have finally rejected petitioner's claim and therefore
the latter's cause of action had not yet arisen. Compliance with
condition No. 11 is a requirement sine qua non to the right to
maintain an action as prior thereto no violation of petitioner's
right can be attributable to private respondent. This is so, as
before such final rejection, there was no real necessity for
bringing suit. Petitioner should have endeavored to file the
formal claim and procure all the documents, papers, inventory
needed by private respondent or its adjuster to ascertain the
amount of loss and after compliance await the final rejection of
its claim. Indeed, the law does not encourage unnecessary
litigation.
Page 22 of 154

INSURANCE G01 CASE DIGESTS


81.
TRAVELLERS INSURANCE V . CA, 272 SCRA 536 (1997)
TRAVELLERS INSURANCE & SURETY CORP. v. CA (MENDOZA)
272 SCRA 536
HERMOSISIMA, JR; May 22, 1997
NATURE
The petition herein seeks the review and reversal of the decision of
respondent Court of Appeals affirming in toto the judgment of the
Regional Trial Court in an action for damages filed by private
respondent Vicente Mendoza, Jr. as heir of his mother who was
killed in a vehicular accident.

can sue the insurer. Where the contract is for indemnity against
actual loss or payment, then third persons cannot proceed against
the insurer, the contract being solely to reimburse the insured for
liability actually discharged by him thru payment to third persons,
said third persons' recourse being thus limited to the insured alone.
But in the case at bar, there was no contract shown. What then was
the basis of the RTC and the CA to say that the insurance contract
was a third-party liability insurance policy? Consequently, the trial
court was confused as it did not distinguish between the private
respondent's cause of action against the owner and the driver of
the Lady Love taxicab and his cause of action against petitioner. The
former is based on torts and quasi-delicts while the latter is based
on contract.

FACTS
An old lady was hit by a taxicab. The taxicab was later identified and
a case was filed against the driver and owner. Later, an amendment
was filed to include the insurance company. RTC and CA ordered
that the owner, driver as well as the insurance company be held
solidarily liable.

ISSUE
WON RTC and CA erred
HELD

Even assuming arguendo that there was such a contract,


private respondent's cause of action can not prevail because he
failed to file the written claim mandated by the Insurance Code
(before it was amended-action must be brought within six months
from date of the accident (this is whats applicable here) ; after
amendment- "action or suit for recovery of damage due to loss or
injury must be brought in proper cases, with the Commissioner or
the Courts within one year from denial of the claim, otherwise the
claimant's right of action shall prescribe" ). He is deemed, under this
legal provision, to have waived his rights as against petitionerinsurer.

YES
Where the contract provides for indemnity against liability
to third persons, then third persons to whom the insured is liable
Page 23 of 154

INSURANCE G01 CASE DIGESTS


82.
LOPEZ V . COMPANIA DE SEGUROS, 16 SCRA 855 (1966)
Lopez vs. Filipinas Compaia de Seguros

G.R. No. L-19613 April 30, 1966

FACTS:

Plaintiff applied with the defendant company for the


insurance of his properties: Biederman truck tractor and a
Winter Weils trailer from loss or damage in the amount of
P20,000.00 and P10,000.00, respectively.
During the application, the defendant company inquired of
the plaintiff the ff:
Has any company in respect of the insurance of any
car or vehicle
(A) declined, cancelled or refused to renew your
insurance?
(B) increased your premium renewal?
Plaintiff answered in negative but the truth was that the
American International Underwriters of the Philippines
(AIU) had already declined similar application for insurance
by the plaintiff with respect of the above-mentioned
vehicles.
The defendant issued to the plaintiff two Commercial
Vehicle Comprehensive Policies covering the said
properties.
The vehicles mentioned figured in an accident resulting in
the total loss of the tractor and partial damage to the
trailer. Plaintiff demand upon the defendant for the
payment to him the total amt. of damages resulting from
the accident.
On April 28, 1960, defendant rejected the claim on the
ground of concealment of a material fact: that the insured

property previously been declined insurance by another


company.
May 27, 1960, the plaintiff filed with the Office of the
Insurance Commissioner a complaint against the said
company.
As suggested, the plaintiff was willing to submit his claim to
arbitration but was contested by the defendant since "the
claim of the plaintiff cannot be resolved by arbitration, as
recourse to arbitration referred to in the policy contract,
envisioned only differences or disputes, 'with respect to the
amount of the company's liability,' and not to cases where
the company does not admit its liability to the insured.
With this rejection, the plaintiff filed his complaint with the
CFI of Manila on September 19,1961.
Against the above complaint, the defendant-appellee filed
on September 29, 1961 a motion to dismiss on the ground
of prescription. The latter argued that the plaintiff's claim
had already prescribed since it was not filed within twelve
months from its rejection by the insurance company as
stipulated under paragraph 9 of the General Conditions of
Commercial Vehicle Comprehensive Policy Nos. 5598 and
5599, to wit:

If a claim be made and rejected and an action or suit be not


commenced within twelve months after such rejection or (in case of
an arbitration taking place as provided herein) within twelve
months after the arbitrator, arbitrators, or umpire shall have made
their award then the claim shall for all purposes be deemed to have
been abandoned and shall not thereafter be recovered hereunder.

Page 24 of 154

INSURANCE G01 CASE DIGESTS


ISSUE: Whether the complaint filed by the plaintiff-appellant with
the Office of the Insurance Comm. on May 27,1960 a
commencement of an "action or suit" within the meaning and
intent of general condition? No.

justice, the claim may not properly be categorized under either


term.

RATIO:

"Action" and "suit":

Rule 2, Section 1 of the Rules of Court


Section 1. Action defined.Action means an ordinary suit in a Court
of Justice by which one party prosecutes another for the
enforcement or protection of a right, or the prevention or redress of
a wrong. (Emphasis supplied.)

An "action or suit" is essentially "for the enforcement or


protection of a right, or the prevention or redress of a
wrong." (Rule 2, Sec. 1, Rules of Court). There is nothing in
the Insurance Law, which empowers the Insurance
Commissioner to adjudicate on disputes relating to an
insurance company's liability to an insured under a policy
issued by the former to the latter. The validity of an
insured's claim under a specific policy, its amount, and all
such other matters as might involve the interpretation and
construction of the insurance policy, are issues which only a
regular court of justice may resolve and settle.
Consequently, the complaint filed by the appellant herein
with the Office of the Insurance Commission could not have
been an "action or suit."

Jurisprudence
Suit is the prosecution or pursuit of some claim or demand in a
court of justice or any proceeding in a court of justice in which a
plaintiff pursues his remedy to recover a right or claim. (Emphasis
supplied.)

- Upon the authorities, therefore, it is settled that the terms


"action" and "suit" are synonymous. Moreover, it is clear that the
determinative or operative fact which converts a claim into an
"action or suit" is the filing of the same with a "court or justice."
Filed elsewhere, as with some other body or office not a court of
Page 25 of 154

INSURANCE G01 CASE DIGESTS


83.

ACCFA V. ALPHA INSURANCE , 24 SCRA 151 (1968)


FACTS

1. Alpha insurance issued 5,000 pesos bond to guarantee FACOMA


against loss on account of personal dishonesty, amounting to
larceny or estafa of its Secretary Treasurer.
2. FACOMA assigned its right to ACCFA, with approval of principal
and surety
3. During effectivity of bond, Secretary Treasurer converted and
misappropriated to his personal benefit 11,000 of FACOMA funds,
which 6,307.33 belonged to ACCFA.
4. Upon discovery of the loss, ACCFA immediately notified the
surety company, but despite repeated demands, surety refused to
pay
5. Alpha insurance moved to dismiss the complaint for failure to
state a cause of action, giving as reason that the same was filed
more than one year after plaintiff made claim for loss, contrary to
the eighth condition of the bond
6. Granted and motion to dismiss upon reconsideration kaya nagappeal si ACCFA.

ISSUE
Whether or not the provision of a fidelity bond that no action shall
be had or maintained thereon unless commenced within one year

from the making of a claim for the loss upon which the action is
based, is valid or void? VOID
HELD
Consequently, the condition of the bond in question, limiting the
period for bringing action thereon, is subject to the provisions of
Section 61-A of the Insurance Act (No. 2427), as amended by Act
4101 of the pre-Commonwealth Philippine Legislature, prescribing
that
SEC. 61-A A condition, stipulation or agreement in any policy of
insurance, limiting the time for commencing an action thereunder
to a period of less than one year from the time when the cause of
action accrues is void.
The cause of action does not accrue until the party obligated
refuses, expressly or impliedly, to comply with its duty (in this case,
to pay the amount of the bond). The year for instituting action in
court must be reckoned, therefore, from the time of appellee's
refusal to comply with its bond; it can not be counted from the
creditor's filing of the claim of loss, for that does not import that the
surety company will refuse to pay.
In so far, therefore, as condition eight of the bond requires action to
be filed within one year from the filing of the claim for loss, such
stipulation contradicts the public policy expressed in Section 61-A
of the Philippine Insurance Act. Condition eight of the bond,
therefore, is null and void, and the appellant is not bound to comply
with its provisions.
Page 26 of 154

INSURANCE G01 CASE DIGESTS


84.

SAURA I MPORT & EXPORT V . PHIL. INTL SURETY 8 SCRA 143


(1963)

G.R. No. L-15184, May 31, 1963


PAREDES, J.
Topic: Cancellation of non-life policy Sec. 62
FACTS:
1. Saura Import & Export Co Inc., mortgaged to PNB, a parcel
of land, to secure the payment of promissory note of P27,
000.00.
2. The mortgage was amended to guarantee an increased
amount, bringing the total mortgaged debt to P37, 000.00.
3. Erected on the land mortgaged, was a building of strong
materials owned by the mortgagor Saura Import & Export
Co., Inc., which had always been covered by insurance,
many years prior to the mortgage contract.
4. Pursuant to the requirement, Saura insured the building and
its contents with the Philippine International Surety for
P29,000.00 against fire for the period of one year from
October 2, 1954.
5. The insurance policy was endorsed to the mortgagee PNB,
in a Memo which states Loss if any, payable to the
Philippine National Bank as their interest may appear,
subject to the terms, conditions and warranties of this
policy.
6. Barely thirteen (13) days after the issuance of the policy, the
insurer cancelled the same. Notice of the cancellation was
given to appellee bank in writing. .

7. The building and its contents, worth P40,685.69 were


burned. Saura filed a claim with the Insurer and mortgagee
Bank.
8. Upon the presentation of notice of loss with the PNB, Saura
learned for the first time that the policy had previously
been cancelled by the insurer, when Saura's folder in the
Bank's filed was opened and the notice of cancellation
(original and duplicate) sent by the Insurer to the Bank, was
found.
9. Upon refusal of the Insurer Philippine International Surety
to pay the amount of the insurance this present case filed
with the Manila CFI against the Insurer, and the PNB was
later included as party defendant, after it had refused to
prosecute the case jointly with Saura Import & Export Co.,
Inc.
ISSUE:
Whether or not there is a valid cancellation of the fire insurance
policy.
HELD: NO
RATIO DECIDENDI:
The policy in question does not provide for the notice, its form
or period. The Insurance Law, Act No. 2427, does not likewise
provide for such notice. This being the case, it devolves upon the
Court to apply the generally accepted principles of insurance,
regarding cancellation of the insurance policy by the insurer.
From what has been stated, actual notice of cancellation in
a clear and unequivocal manner, preferably in writing, in view of
the importance of an insurance contract, should be given by the
insurer to the insured, so that the latter might be given an
Page 27 of 154

INSURANCE G01 CASE DIGESTS


opportunity to obtain other insurance for his own protection. The
notice should be personal to the insured and not to and/or
through any unauthorized person by the policy.

85.

MALAYAN INSURANCE V . CRUZ-ARNALDO , 154 SCRA 672 (1987)

Topic: Cancellation of non-life policy


In the case at bar, the defendant insurance company, must
have realized the paramount importance of sending a notice of
cancellation, when it sent the notice of cancellation of the policy to
the defendant bank (as mortgagee), but not to the insured with
which it (insurance company) had direct dealing. It was the primary
duty of the defendant-appellee insurance company to notify the
insured, but it did not. It should be stated that the house and its
contents were burned on April 6, 1955, at the time when the policy
was enforced (October 2, 1954 to October 2, 1955); and that under
the facts, as found by the trial court, to which We are bound, it is
evident that both the insurance company and the appellee bank
failed, wittingly or unwittingly, to notify the insured appellant Saura
of the cancellation made.
The defendant insurance company contends that it gave
notice to the defendant-appellee bank as mortgagee of the
property, and that was already a substantial compliance with its
duty to notify the insured of the cancellation of the policy. But
notice to the bank, as far appellant herein is concerned, is not
effective notice.

Facts:

In 1981, petitioner Malayan issued to the private


respondent Pinca, a Fire Insurance Policy on her property
for P14,000 effective July 22, 1981 untul July 22, 1982
Malayan allegedly cancelled the policy for non-payment of
premiums and sent notice to Pinca (October 15, 1981)
A couple of months after, payment of the premium for
Pinca was received by Adora, an agent of Malayan
(December 24, 1981)
Adora remitted the payment to Malayan
Three days after, Pincas property was completely burned
(Jan 18, 1982)
A couple of weeks after, Malayan returned Pincas payment
as the policy was previously cancelled
Pinca made demands for the proceeds of the policy, but
Malayan refused. The Insurance Commission decided in
favor of Pinca.
Malayan: There was no payment of premium and that the
policy had been canceled before the occurrence of the loss

Issue: Whether or not Malayan should be liable for the proceeds of


the policy

Held: YES. Malayans argument was not acceptable.


Malayan relies on Sec 77 of the Insurance Code:
Page 28 of 154

INSURANCE G01 CASE DIGESTS


SEC. 77. An insurer is entitled to payment of the premium as
soon as the thing is exposed to the peril insured against.
Notwithstanding any agreement to the contrary, no policy
or contract of insurance issued by an insurance company is
valid and binding unless and until the premium thereof has
been paid, except in the case of a life or an industrial life
policy whenever the grace period provision applies.
The above provision is not applicable because payment of the
premium was in fact eventually made. The premium invoice issued
to Pinca at the time of delivery of the policy on June 7, 1981 was
stamped, Payment Received of the amount of P930.60 on 12-2481 by Adora. It suggests an understanding between Malayan and
the insured that payment could be made later.

The payment was made on December 24, 1981, and the fire
occured on January 18, 1982. One wonders: suppose the payment
had been made and accepted in, say, August 1981, would the
commencement date of the policy have been changed to the date
of the payment, or would the payment have retroacted to July 22,
1981? If MALAYAN accepted the payment in December 1981 and
the insured property had not been burned, would that policy not
have expired just the same on July 22, 1982, pursuant to its original
terms, and not on December 24, 1982?
There is the petitioner's argument, however, that Adora was not
authorized to accept the premium payment because six months had
elapsed since the issuance by the policy itself. It is argued that this
prohibition was binding upon Pinca, who made the payment to

Adora at her own risk as she was bound to first check his authority
to receive it.

MALAYAN is taking an inconsistent stand. While contending that


acceptance of the premium payment was prohibited by the policy, it
at the same time insists that the policy never came into force
because the premium had not been paid. One surely, cannot have
his cake and eat it too.
We do not share MALAYAN's view that there was no existing
insurance at the time of the loss sustained by Pinca because her
policy never became effective for non-payment of premium.
Payment was in fact made, rendering the policy operative as of June
22, 1981, and removing it from the provisions of Article 77,
Thereafter, the policy could be cancelled on any of the supervening
grounds enumerated in Article 64 (except "nonpayment of
premium") provided the cancellation was made in accordance
therewith and with Article 65.

Section 64 reads as follows:


SEC. 64. No policy of insurance other than life shall be
cancelled by the insurer except upon prior notice thereof to
the insured, and no notice of cancellation shall be effective
unless it is based on the occurrence, after the effective date
of the policy, of one or more of the following:
(a) non-payment of premium;
(b) conviction of a crime arising out of acts increasing the
hazard insured against;
Page 29 of 154

INSURANCE G01 CASE DIGESTS


(c) discovery of fraud or material misrepresentation;
(d) discovery of willful, or reckless acts or commissions
increasing the hazard insured against;
(e) physical changes in the property insured which result in
the property becoming uninsurable;or
(f) a determination by the Commissioner that the
continuation of the policy would violate or would place the
insurer in violation of this Code.
As for the method of cancellation, Section 65 provides as follows:
SEC. 65. All notices of cancellation mentioned in the
preceding section shall be in writing, mailed or delivered to
the named insured at the address shown in the policy, and
shall state (a) which of the grounds set forth in section sixtyfour is relied upon and (b) that, upon written request of the
named insured, the insurer will furnish the facts on which
the cancellation is based.

All MALAYAN's offers to show that the cancellation was


communicated to the insured is its employee's testimony that the
said cancellation was sent "by mail through our mailing section."
without more
It stands to reason that if Pinca had really received the said notice,
she would not have made payment on the original policy on
December 24, 1981. Instead, she would have asked for a new
insurance, effective on that date and until one year later, and so
taken advantage of the extended period. The Court finds that if she
did pay on that date, it was because she honestly believed that the
policy issued on June 7, 1981, was still in effect and she was willing
to make her payment retroact to July 22, 1981, its stipulated
commencement date.
Adora, incidentally, had not been informed of the cancellation
either and saw no reason not to accept the said payment

A valid cancellation must, therefore, require concurrence of the


following conditions:
(1) There must be prior notice of cancellation to the insured;
(2) The notice must be based on the occurrence, after the effective
date of the policy, of one or more of the grounds mentioned;
(3) The notice must be (a) in writing, (b) mailed, or delivered to the
named insured, (c) at the address shown in the policy;
(4) It must state (a) which of the grounds mentioned in Section 64 is
relied upon and (b) that upon written request of the insured, the
insurer will furnish the facts on which the cancellation is based.

Page 30 of 154

INSURANCE G01 CASE DIGESTS

WARRANTIES

ISSUE: Whether or not the Court of Appeals erred in its legal


interpretation of 'Fire ExtinguishingAppliances Warranty' of the
policy.

86. AMERICAN HOME INSURANCE V. TANTUCO ENTERPRISES, 366


SCRA 740 (2011)
G.R. No. 138941October 8, 2001

FACTS: Respondent Tantuco Enterprises, Inc. is engaged in the


coconut oil milling and refining industry.It owns two oil mills which
were separately covered by fire insurance policies issued by
petitionerAmerican Home Assurance Co., Philippine Branch.
The first oil mill was insured for P3,000,000.00 underPolicy No. 3067432324-3 for the period March 1, 1991 to 1992. The new oil mill
was insured forP6,000,000.00 under Policy No. 306-7432321-9 for
the same term. Official receipts indicating paymentfor the full
amount of the premium were issued by the petitioner's agent.A fire
that broke out in the early morning of September 30,1991 gutted
and consumed the new oil mill.Respondent immediately notified
the petitioner of the incident but petitioner rejected
respondent'sclaim for the insurance proceeds on the ground that no
policy was issued by it covering the burned oilmill. It stated that the
description of the insured establishment referred to another
building thus: "Ourpolicy nos. 306-7432321-9 (Ps 6M) and 3067432324-4 (Ps 3M) extend insurance coverage to your oilmill under
Building No. 5, whilst the affected oil mill was under Building No.
14."

HELD: In construing the words used descriptive of a building


insured, the greatest liberality is shown bythe courts in giving effect
to the insurance. In view of the custom of insurance agents to
examinebuildings before writing policies upon them, and since a
mistake as to the identity and character of thebuilding is extremely
unlikely, the courts are inclined to consider that the policy of
insurance covers anybuilding which the parties manifestly intended
to insure, however inaccurate the description may
be.Notwithstanding, therefore, the misdescription in the policy, it is
beyond dispute, to our mind, that whatthe parties manifestly
intended to insure was the new oil mill. If the parties really intended
to protectthe first oil mill,then there is no need to specify it as new .
In determining what the parties intended, the courts will read and
construe the policy as a whole and if possible, give effect to all the
parts of the contract, keeping in mind always, however, the prime
rulethat in the event of doubt, this doubt is to be resolved against
the insurer. In determining the intent of the parties to the contract,
the courts will consider the purpose and object of the contract.

Page 31 of 154

INSURANCE G01 CASE DIGESTS


87. ANG GIOK V. SPRINGFIELD FIRE & MUTUAL INSURANCE , 56 PHIL.
375 (1931)
56 Phil. 375 (1931)
Warranties
Facts: Ang Giok Chip doing business under the name and style of
Hua Bee Kong Si was formerly the owner of a warehouse situated in
Manila.
The contents of the warehouse were insured with the three
insurance companies for the total sum of P60,000. One insurance
policy, in the amount of P10,000, was taken out with the Springfield
Fire & Marine Insurance Company the warranty includes a Rider:
no hazardous material be stored in the building, that the Insured be
permitted to stored a small quantity of the hazardous goods
specified below, but not exceeding in all 3 per cent of the total
value of the whole of the goods or merchandise contained in said
warehouse.
The warehouse was destroyed by fire on January 11, 1928, while
the policy issued by the latter company was in force. The plaintiff
instituted action in the Court of First Instance of Manila against the
defendant to recover a proportional part of the loss coming to
P8,170.59. Four special defenses were interposed on behalf of the
insurance company, one being planted on a violation of warranty F
fixing the amount of hazardous goods which might be stored in the
insured building.
The CFI rendered in favor of the plaintiff. But the SC held that it is
unnecessary for us to discuss three of the four special defenses
which were made by the insurance company. We think, however,
that it would be a reasonable deduction to conclude that more than
3 per cent of the total value of the merchandise contained in the
warehouse constituted hazardous goods, and that this per cent

reached as high as 39. We place reliance on the consular invoices


and on the testimony of the adjuster, Herridge. As such only one
issue is to be resolved.
Issue: WON a warranty referred to in the policy as forming part of
the contract of insurance and in the form of a rider to the insurance
policy, is null and void because not complying with the Philippine
Insurance Act?
Held: No, the Philippine Law was taken verbatim from law of
California. Thus, the court should follow in fundamental points, at
least, the construction placed by California courts on a California
Law. The SC reached the definite conclusion that warranty F, a rider
attached to the face of the insurance policy, and referred to in
contract of insurance, is valid and sufficient under section 65 of the
Insurance Act.
Ratio: Section 65 of the Insurance Act and its counterpart, section
265 of the Civil Code of California, will bear analysis as tested by
reason and authority. The law says that every express warranty
must be "contained in the policy itself." The word "contained,"
according to the dictionaries, means "included," inclosed,"
"embraced," "comprehended," etc. When, therefore, the courts
speak of a rider attached to the policy, and thus "embodied"
therein, or of a warranty "incorporated" in the policy, it is believed
that the phrase "contained in the policy itself" must necessarily
include such rider and warranty.
The SC thinks it wrong to hold that the California law represents a
radical departure from the basic principles governing the law of
insurance. We turn to two of such well recognized doctrines. In the
first place, it is well settled that a rider attached to a policy is a part
of the contract, to the same extent and with like effect as it actually
embodied therein. (I Couch, Cyclopedia of Insurance Law, sec. 159.)
In the second place, it is equally well settled that an express
Page 32 of 154

INSURANCE G01 CASE DIGESTS


warranty must appear upon the face of the policy, or be clearly
incorporated therein and made a part thereof by explicit reference,
or by words clearly evidencing such intention. (4 Couch, Cyclopedia
of Insurance Law, sec. 862.)
Referring to the jurisprudence of California, another rule of
insurance adopted in that State is in point. It is admitted that the
policy before us was accepted by the plaintiff. The receipt of this
policy by the insured without objection binds both the acceptor and
the insured to the terms thereof. The insured may not thereafter be
heard to say that he did not read the policy or know its terms, since
it is his duty to read his policy and it will be assumed that he did so.

88. QUA CHEE GAN V. LAW UNION AND ROCK INSURANCE , 98 PHIL
85 (1955)
QUA CHEE GAN vs LAW UNION AND ROCK INSURANCE represented
by agent, Warner, Barnes and Co., Ltd. (December 17, 1955)
FACTS:

Before the last war, Qua Chee Gan owned four warehouses
or bodegas in Tabaco, Albay, used for the storage of stocks
of copra and of hemp, baled and loose, in which the he
dealth extensively. They had been insured with Law Union
and Rock Insurance since 1937, and the loss made payable
to the Philippine National Bank as mortgage of the hemp
and crops, to the extent of its interest.
On June, 1940, the insurance stood as follows:
Policy No.
Property Insured
Amount
2637164
Bodega No. 1 (Building)
P15,000.00
(Exhibit "LL")
2637165
Bodega No. 2 (Building)
10,000.00
(Exhibit "JJ")
Bodega No. 3 (Building)
25,000.00
Bodega No. 4 (Building)
10,000.00
Hemp Press moved by steam
5,000.00
engine
2637345
Merchandise contents (copra and
150,000.00
(Exhibit "X")
empty sacks of Bodega No. 1)
2637346
Merchandise contents (hemp) of
150,000.00
(Exhibit "Y")
Bodega No. 3
2637067
Merchandise contents (loose
5,000.00
(Exhibit "GG") hemp) of Bodega No. 4
Total
P370,000.00

Page 33 of 154

INSURANCE G01 CASE DIGESTS

protected, with not less than 100 feet of hose piping and
nozzles for every two hydrants kept under cover in
convenient places, the hydrants being supplied with water
pressure by a pumping engine, or from some other source,
capable of discharging at the rate of not less than 200
gallons of water per minute into the upper story of the
highest building protected, and a trained brigade of not less
than 20 men to work the same.

On July 21, 1940, fire broke out near the Bodegas and last
for almost one week which completely destroyed Bodegas
1,2 and 4.
Qua Chee Gan claims 398,562.81 (reduced to 370,000, the
amount in the insurance). Fire adjusters conducted an
investigation. Law Union resisted the payment claiming
violation of warranties and conditions, filing of fraudulent
claims, and that the fire had been deliberately caused by
the insured/other persons in connivance.
Qua Chee Gan and Qua Chee Pao (brother)were tried for
Arson but the trial court acquitted them.
Qua Chee Gan then filed a civil case to claim the proceeds of
the fire insurance policies. CFI ruled in favour of him. PNB
filed a complaint in intervention but it was dismissed
because Qua Chee Gan managed to pay his indebtedness.

Since the bodegas had an external wall perimeter of 1,640


feet, there should have been 11 fire hydrants and Qua Chee
Gan only had 2. Another pair is available also but it belongs
to the municipality of Tabaco.

ISSUE: WON Qua Chee Gan breached provisions of warranty that he


cannot be entitled to claim insurance proceeds.
HELD: NO
1st cause of action - Hydrants: NO

Memo. of Warranty. The undernoted Appliances for the


extinction of fire being kept on the premises insured hereby,
and it being declared and understood that there is an ample
and constant water supply with sufficient pressure available
at all seasons for the same, it is hereby warranted that the
said appliances shall be maintained in efficient working
order during the currency of this policy, by reason whereof a
discount of 2 1/2 per cent is allowed on the premium
chargeable under this policy.
Hydrants in the compound, not less in number than one for
each 150 feet of external wall measurement of building,

Law Union is barred by estoppel because they know that


the number of hydrants demanded in the rider never
existed from the very beginning but they still issued the
policies in question subject to such warranty, and received
the corresponding premiums. It would be perilously close to
conniving at fraud upon the insured to allow Law Union to
claim now as void ab initio the policies that it had issued to
the Qua Chee Gan without warning of their fatal defect, of
which it was informed, and after it had misled the
defendant into believing that the policies were effective.
Court finds it unusual also that during the period for
insurance, the insurer suddenly realized that the premises
are hazardous and offered the rider warranty for a discount
on the premium.
Rule: Where the insurer, at the time of the issuance of a
policy of insurance, has knowledge of existing facts which, if
insisted on, would invalidate the contract from its very
inception, such knowledge constitutes a waiver of
conditions in the contract inconsistent with the facts, and
the insurer is estopped thereafter from asserting the breach
Page 34 of 154

INSURANCE G01 CASE DIGESTS

of such conditions. The law is charitable enough to assume,


in the absence of any showing to the contrary, that an
insurance company intends to execute a valid contract in
return for the premium received; and when the policy
contains a condition which renders it voidable at its
inception, and this result is known to the insurer, it will be
presumed to have intended to waive the conditions and to
execute a binding contract, rather than to have deceived
the insured into thinking he is insured when in fact he is
not, and to have taken his money without consideration.
(29 Am. Jur., Insurance, section 807, at pp. 611-612.)
Ratio: To allow a company to accept one's money for a
policy of insurance which it then knows to be void and of no
effect, though it knows as it must, that the assured believes
it to be valid and binding, is so contrary to the dictates of
honesty and fair dealing, and so closely related to positive
fraud, as to the abhorent to fairminded men. It would be to
allow the company to treat the policy as valid long enough
to get the preium on it, and leave it at liberty to repudiate it
the next moment. This cannot be deemed to be the real
intention of the parties. To hold that a literal construction of
the policy expressed the true intention of the company
would be to indict it, for fraudulent purposes and designs
which we cannot believe it to be guilty of (Wilson vs.
Commercial Union Assurance Co., 96 Atl. 540, 543-544).
The alleged violation of the warranty of 100 feet of fire hose
for every two hydrants, must be equally rejected, since it is
based on the assumption that the insured was bound to
maintain no less than eleven hydrants (one per 150 feet of
wall), which requirement appellant is estopped from
enforcing.
As to maintenance of a trained fire brigade of 20 men, the
record is preponderant that the same was organized, and
drilled, from time to give, although not maintained as a
permanently separate unit, which the warranty did not

require. Anyway, it would be unreasonable to expect the


insured to maintain for his compound alone a fire fighting
force that many municipalities in the Islands do not even
possess.
2nd cause of action Hemp Warranty: No

Prohibition on Oils (animal and/or vegetable and/or mineral


and/or their liquid products having a flash point below 300
degrees Fahrenheit Law Union argues that Qua Chee Gan
admitted that he had 36 cans of gasoline in Bodega 2.
It is well to note that gasoline is not specifically mentioned
among the prohibited articles listed in the so-called "hemp
warranty." It is ambiguous and uncertain; for in ordinary
parlance, "Oils" mean "lubricants" and not gasoline or
kerosene. And how many insured, it may well be wondered,
are in a position to understand or determine "flash point
below 300 Fahrenheit. Here, again, by reason of the
exclusive control of the insurance company over the terms
and phraseology of the contract, the ambiguity must be
held strictly against the insurer and liberraly in favor of the
insured, specially to avoid a forfeiture.
We see no reason why the prohibition of keeping gasoline
in the premises could not be expressed clearly and
unmistakably, in the language and terms that the general
public can readily understand, without resort to obscure
esoteric expression. If the company intended to rely upon a
condition of that character, it ought to have been plainly
expressed in the policy.
Another point that is in favor of the insured is that the
gasoline kept in Bodega No. 2 was only incidental to his
business, being no more than a customary 2 day's supply for
the five or six motor vehicles used for transporting of the
stored merchandise. "It is well settled that the keeping of
inflammable oils on the premises though prohibited by the
policy does not void it if such keeping is incidental to the
Page 35 of 154

INSURANCE G01 CASE DIGESTS


business." Bachrach vs. British American Ass. Co., 17 Phil.
555, 560); and "according to the weight of authority, even
though there are printed prohibitions against keeping
certain articles on the insured premises the policy will not
be avoided by a violation of these prohibitions, if the
prohibited articles are necessary or in customary use in
carrying on the trade or business conducted on the
premises." (45 C. J. S., p. 311; also 4 Couch on Insurance,
section 966b). It should also be noted that the "Hemp
Warranty" forbade storage only "in the building to which
this insurance applies and/or in any building communicating
therewith", and it is undisputed that no gasoline was stored
in the burned bodegas, and that "Bodega No. 2" which was
not burned and where the gasoline was found, stood
isolated from the other insured bodegas.

89. PIONEER INSURANCE V. YAP, 61 SCRA 426 (1974)


GR L-36232; 61 SCRA 426, (1947)
Topic: Warranties: Effect Warranties
Facts:
Olivia Yap owned a two-storey building in Manila where she sold
shopping bags and footwear. Her son-in-law Chua Soon Poon was in charge
of the store. She took a fire insurance of Php25,000 on the building with
the petitioner to cover her stocks and furniture. One of the conditions in
the policy stated that the insured should notify the insurance company of
any insurance policies already effected or may be subsequently effected
covering the property unless such notice be given and the particulars of
such insurance or insurances be stated in, or endorsed on this Policy by or
on behalf of the Company before the occurrence of any loss or damage, all
benefits under this Policy shall be forfeited any false declarations will
render this policy null and void.
At the time the petitioner issued the policy, Great American
Insurance issued another policy on the same properties of Yap. This
issuance was noted with the petitioner. Later in the year however, she
took out another policy with Federal Insurance covering the same
properties again, only this time she did not notify the petitioner.
In December 1962, a fire burned down the building. Yap filed an
insurance claim against the petitioner but the latter denied. The petitioner
claimed there was a breach of contract. Yap filed a complaint for payment
of the value of the insurance policy with the petitioner. Petitioner claimed
she is not entitled because (1) none of the properties covered by the policy
were destroyed by the fire, and (2) even if these properties were burned,
she is still not entitled for breach of contract. Petitioner claims the
proceeds were forfeited.
The RTC decided in favor of Yap. The CA affirmed.
Issue:
Should the petitioner be liable based on the fire insurance policy?
Held:
NO

Page 36 of 154

INSURANCE G01 CASE DIGESTS


Rationale:
There was a violation by respondent Oliva Yap of the co-insurance clause
contained in Policy No. 4219 that resulted in the avoidance of petitioner's
liability. The insurance policy for P20,000.00 issued by the Great American
Insurance Company covering the same properties of respondent Yap and
duly noted on Policy No. 4219 as c-insurance, ceased, by agreement of the
parties (Exhibit "1-L"), to be recognized by them as a co-insurance policy.
The Court of Appeals says that the Great American Insurance policy was
substituted by the Federal Insurance policy for the same amount, and
because it was a mere case of substitution, there was no necessity for its
endorsement on Policy No. 4219. This finding, as well as reasoning, suffers
from several flaws. There is no evidence to establish and prove such a
substitution. If anything was substituted for the Great American Insurance
policy, it could only be the Northwest Insurance policy for the same
amount of P20,000.00. The endorsement (Exhibit "1-K") quoted above
shows the clear intention of the parties to recognize on the date the
endorsement was made (August 29, 1962), the existence of only one coinsurance, and that is the Northwest Insurance policy, which according to
the stipulation of the parties during the hearing, was issued on August 20,
1962 (t.s.n., January 12, 1965, pp. 3-4) and endorsed only on August 20,
1962. The finding of the Court of Appeals that the Great American
Insurance policy was substituted by the Federal Insurance policy is
unsubstantiated by the evidence of record and indeed contrary to said
stipulation and admission of respondent, and is grounded entirely on
speculation, surmises or conjectures, hence, not binding on the Supreme
Court.
By the plain terms of the policy, other insurance without the consent of
petitioner would ipso facto avoid the contract. It required no affirmative
act of election on the part of the company to make operative the clause
avoiding the contract, wherever the specified conditions should occur. Its
obligations ceased, unless, being informed of the fact, it consented to the
additional insurance.
The validity of a clause in a fire insurance policy to the effect that the
procurement of additional insurance without the consent of the insurer
renders ipso facto the policy void is well-settled:
In Milwaukee Mechanids' Lumber Co., vs. Gibson, 199
Ark. 542, 134 S. W. 2d 521, 522, a substantially identical
clause was sustained and enforced, the court saying:
"The rule in this state and practically all of the states is to

the effect that a clause in a policy to the effect that the


procurement of additional insurance without the consent
of the insurer renders the policy void is a valid provision.
The earlier cases of Planters Mutual Insurance Co., vs.
Green, 72 Ark. 305, 80 S.W. 92, are to the same effect."
And see Vance, Insurance, 2nd Ed., 725. (Reach vs.
Arkansas Farmers Mut. Fire Ins. Co., [Ark. Nov. 14, 1949]
224 S. W. 2d 48, 49.)
2. Where a policy contains a clause providing that the
policy shall be void if insured has or shall procure any
other insurance on the property, the procurement of
additional insurance without the consent of the insurer
avoids the policy." (Planters' Mut. Ins. Ass'n vs. Green
[Supreme Court of Arkansas, March 19, 1904] 80 S.W.
151.)
The obvious purpose of the aforesaid requirement in the policy is to
prevent over-insurance and thus avert the perpetration of fraud. The
public, as well as the insurer, is interested in preventing the situation in
which a fire would be profitable to the insured. According to Justice Story:
"The insured has no right to complain, for he assents to comply with all the
stipulation on his side, in order to entitle himself to the benefit of the
contract, which, upon reason or principle, he has no right to ask the court
to dispense with the performance of his own part of the agreement, and
yet to bind the other party to obligations, which, but for those stipulation
would not have been entered into."

Page 37 of 154

INSURANCE G01 CASE DIGESTS


90. PRUDENTIAL GUARANTEE V. TRANS A SIA SHIPPING, 491 SCRA
411 (2006)
Chico-Nazario, J.
Facts
1. TRANS-ASIA is the owner of the vessel M/V Asia Korea.
2. In consideration of payment of premiums, defendant
[PRUDENTIAL] insured M/V Asia Korea for loss/damage of
the hull and machinery arising from perils, inter alia, of fire
and explosion for the sum of P40 Million, beginning [from]
the period [of] July 1, 1993 up to July 1, 1994.
3. This is evidenced by a Marine Policy.
4. On October 25, 1993, while the policy was in force, a fire
broke out while [M/V Asia Korea was] undergoing repairs at
the port of Cebu.
5. On October 26, 1993 plaintiff [TRANS-ASIA] filed its notice
of claim for damage sustained by the vessel. This is
evidenced by a letter/formal claim of even date.
6. Plaintiff [TRANS-ASIA] reserved its right to subsequently
notify defendant [PRUDENTIAL] as to the full amount of the
claim upon final survey and determination by average
adjuster Richard Hogg International (Phil.) of the damage
sustained by reason of fire. An adjusters report on the fire
in question was submitted by Richard Hogg International.
7. Plaintiff [TRANS-ASIA] executed a document denominated
"Loan and Trust receipt in which Trans-Asia received from
Prudential 3M.
8. Thereafter, Prudential denied the insurance claim of TransAsia because Trans-Asia violated the Warranty which stated,
"WARRANTED VESSEL CLASSED AND CLASS MAINTAINED".

9. Prudential requested for the return of the 3M.


10. TRANS-ASIA filed a Complaint5 for Sum of Money against
PRUDENTIAL with the RTC of Cebu City alleging that the 3M
was the balance of the indemnity due.
11. Prudential answered saying that they wanted the 3M back
because Trans-Asia violated the warranty abovementioned.
12. RTC- in favor of Prudential.
a. Trans-Asia was required to maintain the vessel and
found that Trans-Asia violated it.
13. CA reversed.
a. Prudential was not able to prove that Trans-Asia
violated the warranty.
b. The rider stating the warranty clause was attached
without the intervention of Trans-Asia.
c. The loan was in effect a subrogation. 3M was partial
payment of the indemnity.
14. Hence, this petition.

Issues
1. Whether or not Trans-Asia violated its warranty to maintain
the vessel.
o NO.
2. Whether or not the loan agreement as evidenced by the
trust receipt was a form of subrogation.
o Yes.
Ruling:
First Issue
Page 38 of 154

INSURANCE G01 CASE DIGESTS

We sustain the findings of the Court of Appeals that


PRUDENTIAL was not successful in discharging the burden
of evidence that TRANS-ASIA breached the subject policy
condition on CLASSED AND CLASS MAINTAINED.
Foremost, PRUDENTIAL, through the Senior Manager of its
Marine and Aviation Division, Lucio Fernandez, made a
categorical admission that at the time of the procurement
of the insurance contract in July 1993, TRANS-ASIAs vessel,
"M/V Asia Korea" was properly classed by Bureau Veritas.
We are not unmindful of the clear language of Sec. 74 of the
Insurance Code which provides that, "the violation of a
material warranty, or other material provision of a policy on
the part of either party thereto, entitles the other to
rescind." It is generally accepted that "[a] warranty is a
statement or promise set forth in the policy, or by reference
incorporated therein, the untruth or non-fulfillment of
which in any respect, and without reference to whether the
insurer was in fact prejudiced by such untruth or nonfulfillment, renders the policy voidable by the insurer."25
However, it is similarly indubitable that for the breach of a
warranty to avoid a policy, the same must be duly shown by
the party alleging the same. We cannot sustain an allegation
that is unfounded. Consequently, PRUDENTIAL, not having
shown that TRANS-ASIA breached the warranty condition,
CLASSED AND CLASS MAINTAINED, it remains that TRANSASIA must be allowed to recover its rightful claims on the
policy.
Assuming arguendo that TRANS-ASIA violated the policy
condition on WARRANTED VESSEL CLASSED AND CLASS
MAINTAINED, PRUDENTIAL made a valid waiver of the
same. The Court of Appeals, in reversing the Judgment of

the RTC which held that TRANS-ASIA breached the warranty


provision on CLASSED AND CLASS MAINTAINED,
underscored that PRUDENTIAL can be deemed to have
made a valid waiver of TRANS-ASIAs breach of warranty as
alleged, ratiocinating, thus: Third, after the loss, Prudential
renewed the insurance policy of Trans-Asia for two (2)
consecutive years, from noon of 01 July 1994 to noon of 01
July 1995, and then again until noon of 01 July 1996. This
renewal is deemed a waiver of any breach of warrant.
Second Issue

What is clear from the wordings of the so-called "Loan and


Trust Receipt Agreement" is that appellant is obligated to
hand over to appellee "whatever recovery (Trans Asia) may
make and deliver to (Prudential) all documents necessary to
prove its interest in the said property." For all intents and
purposes therefore, the money receipted is payment under
the policy, with Prudential having the right of subrogation
to whatever net recovery Trans-Asia may obtain from third
parties resulting from the fire. In the law on insurance,
subrogation is an equitable assignment to the insurer of all
remedies which the insured may have against third person
whose negligence or wrongful act caused the loss covered
by the insurance policy, which is created as the legal effect
of payment by the insurer as an assignee in equity. The loss
in the first instance is that of the insured but after
reimbursement or compensation, it becomes the loss of the
insurer. It has been referred to as the doctrine of
substitution and rests on the principle that substantial
justice should be attained regardless of form, that is, its
Page 39 of 154

INSURANCE G01 CASE DIGESTS

basis is the doing of complete, essential, and perfect justice


between all the parties without regard to form.31
We agree. Notwithstanding its designation, the tenor of the
"Loan and Trust Receipt" evidences that the real nature of
the transaction between the parties was that the amount of
P3,000,000.00 was not intended as a loan whereby TRANSASIA is obligated to pay PRUDENTIAL, but rather, the same
was a partial payment or an advance on the policy of the
claims due to TRANS-ASIA.

SC affirms. Petition denied.


Side Issue on attorneys fees.

In pursuant to Sec. 244 of the Insurance Code, there was an


unreasonable delay on the part of PRUDENTIAL to pay
TRANS-ASIA, as in fact, it refuted the latters right to the
insurance claims, from the time proof of loss was shown
and the ascertainment of the loss was made by the
insurance adjuster. Evidently, PRUDENTIALs unreasonable
delay in satisfying TRANS-ASIAs unpaid claims compelled
the latter to file a suit for collection.

Page 40 of 154

INSURANCE G01 CASE DIGESTS

PREMIUM
o
91. ARCE V. CAPITAL INSURANCE , 117 SCRA 63 (1982)
COMPANY/INSURER: Capital Insurance

COMPANY to have his signature on the check


Identified preparatory to encashment.
At that time the COMPANY reiterated that the
check was given "not as an obligation, but as a
concession" because the renewal premium had not
been paid, The INSURED cashed the check but then
sued the COMPANY on the policy.

INSURED: Pedro Arce

Issue

Facts

Arces house was insured under a fire insurance policy


On November 27, 1965, the insurer sent to the insured a
Renewal Certificate to cover the period December 5, 1965
to December 5, 1966.
The COMPANY also requested payment of the
corresponding premium in the amount of P 38.10.
Anticipating that the premium could not be paid on time,
the INSURED, thru his wife, promised to pay it on January 4,
1966.
The COMPANY accepted the promise but the premium was
not paid on January 4, 1966.
On January 8, 1966, the house of the INSURED was totally
destroyed by fire.
On January 10, 1966, INSURED's wife presented a claim for
indemnity to the COMPANY.
o She was told that no indemnity was due because
the premium on the policy was not paid.
o Nonetheless the COMPANY tendered a check for
P300.00 as financial aid which was received by the
INSURED's daughter, Evelina R. Arce as full
settlement
o The INSURED and his wife went to the office of the

Whether or not the insurer can be held liable despite the nonpayment of the premiums
Held and Ratio
No
Relevant Provisions
Insurance Code
SEC. 72. An insurer is entitled to payment of premium as soon as the
thing insured is exposed to the perils insured against, unless there is
clear agreement to grant credit extension for the premium due. No
policy issued by an insurance company is valid and binding unless
and until the premium thereof has been paid
Contract Stipulation
IT IS HEREBY DECLARED AND AGREED that not. withstanding
anything to the contrary contained in the within policy, this
insurance will be deemed valid and binding upon the Company only
when the premium and documentary stamps therefor have actually
been paid in full and duly acknowledged in an official receipt signed
by an authorized official/representative of the Company,
Page 41 of 154

INSURANCE G01 CASE DIGESTS


Ruling
It is obvious from both the Insurance Act, as amended, and the
stipulation of the parties that time is of the essence in respect of the
payment of the insurance premium so that if it is not paid the
contract does not take effect unless there is still another stipulation
to the contrary. In the instant case, the INSURED was given a grace
period to pay the premium but the period having expired with no
payment made, he cannot insist that the COMPANY is nonetheless
obligated to him.

92. PHIL. PHOENIX SURETY & ASSURANCE V. WOODWORKS, I NC., 20


SCRA 1270 (1967)
Facts:
That on April 1, 1960, plaintiff issued to defendant Fire Policy No.
9652 for the amount of P300,000.00, under the terms and
conditions therein set forth in said policy a copy of which is hereto
attached and made a part hereof as Annex "A";
That the premiums of said policy as stated in Annex "A" amounted
to P6,051.95; the margin fee pursuant to the adopted plan as an
implementation of Republic Act 2609 amounted to P363.72, copy of
said adopted plan is hereto attached as Annex "B" and made a part
hereof, the documentary stamps attached to the policy was P96.42;
That the defendant paid P3,000.00 on September 22, 1960 under
official receipt No. 30245 of plaintiff;

WON the non payment of premium due resulted in the cancellation


of the contract of insurance
Held:
No
Ratio:
We can not agree with appellant's theory that non-payment by it of
the premium due, produced the cancellation of the contract of
insurance. Such theory would place exclusively in the hands of one
of the contracting parties the right to decide whether the contract
should stand or not. Rather the correct view would seem to be this:
as the contract had become perfected, the parties could demand
from each other the performance of whatever obligations they had
assumed. In the case of the insurer, it is obvious that it had the right
to demand from the insured the completion of the payment of the
premium due or sue for the rescission of the contract. As it chose to
demand specific performance of the insured's obligation to pay the
balance of the premium, the latter's duty to pay is indeed
indubitable.
PHIL PHOENIX V WOODWORKS 92 SCRA 419 (1979)
Facts:
July 21, 1960: Woodworks, Inc. was issued a fire policy for its
building machinery and equipment by Philippine Phoenix Surety
& Insurance Co. for P500K covering July 21, 1960 to July 21,
1961. Woodworks did not pay the premium totalling
to P10,593.36.
April 19, 1961: It was alleged that Woodworks notified
Philippine Phoenix the cancellation of the Policy so Philippine
Phoenix credited P3,110.25 for the unexpired period of 94 days
and demanded in writing the paymentof P7,483.11

That plaintiff made several demands on defendant to pay the


amount of P3,522.09.
Issue:
Page 42 of 154

INSURANCE G01 CASE DIGESTS

Woodworks refused stating that it need not pay premium


"because the Insurer did not stand liable for any indemnity
during the period the premiums were not paid."
Philippine Phoenix filed with the CFI to recover its earned
premium of P7,483.11
Woodworks: to pay the premium after the issuance of the
policy put an end to the insurance contract and rendered the
policy unenforceable
CFI: favored Philippine Phoenix

ISSUE: W/N there was a valid insurance contract despite no


premium payment was paid

HELD:
NO. Reversed

Policy provides for pre-payment of premium. To constitute an


extension of credit there must be a clear and express
agreement therefor and there nust be acceptance of the
extension - none here
Since the premium had not been paid, the policy must be
deemed to have lapsed.
failure to make a payment of a premium or assessment at the
time provided for, the policy shall become void or forfeited, or
the obligation of the insurer shall cease, or words to like effect,
because the contract so prescribes and because such a
stipulation is a material and essential part of the contract. This
is true, for instance, in the case of life, health and accident, fire
and hail insurance policies
Explicit in the Policy itself is plaintiff's agreement to indemnify
defendant for loss by fire only "after payment of
premium" Compliance by the insured with the terms of the
contract is a condition precedent to the right of recovery.

The burden is on an insured to keep a policy in force by


the payment of premiums, rather than on the insurer to exert
every effort to prevent the insured from allowing a policy to
elapse through a failure to make premium payments.

93. MAKATI TUSCANY CONDOMINIUM V. COURT OF APPEALS, 215


SCRA 463 (1992)
215 SCRA 462
BELLOSILLO; November 6, 1992
NATURE
Appeal from decision of the CA
FACTS
- American Home Assurance Co. (AHAC), represented by
American International Underwriters (Phils.), Inc., issued in
favor of petitioner Makati Tuscany Condominium
Corporation an insurance policy on the latter's building and
premises, for the period 1 March 1982 to1 March 1983. The
premium was paid on installments all of which were
accepted by AHAC.

- A second policy was issued to renew the first one, this


time covering the period 1 March 1983 to 1 March 1984.
This was also pain in installment basis.

- A third policy was again issued for the period 1 March


1984 to 1 March 1985. For this, petitioner made two
installment payments, both accepted by AHAC. Thereafter,
Page 43 of 154

INSURANCE G01 CASE DIGESTS


petitioner refused to pay the balance of the premium. AHAC
filed an action to recover the unpaid balance of
P314,103.05.

- Petitioner explained that it discontinued the payment of


premiums because the policy did not contain a credit clause
in its favor and the receipts for the installment payments
covering the policy for 1984-85, as well as the two (2)
previous policies, stated the following reservations:
2. Acceptance of this payment shall not waive any of the
company rights to deny liability on any claim under the
policy arising before such payments or after the expiration
of the credit clause of the policy; and
3. Subject to no loss prior to premium payment. If there be
any loss such is not covered.

- Petitioner further claimed that the policy was never


binding and valid, and no risk attached to the policy. It then
pleaded a counterclaim for P152k for the premiums already
paid for 1984-85, and in its answer with amended
counterclaim, sought the refund of P924,206.10
representing the premium payments for 1982-85.

- Trial court dismissed the complaint and the counterclaim


upon the following findings: (1) payment of the premiums
of the three policies were made during the term of said
policies, hence, it could not be said, inspite of the
reservations, that no risk attached under the policies; (2) as
regards the unpaid premiums, in view of the reservation in

the receipts ordinarily issued by AHAC on premium


payments the only plausible conclusion is that AHAC has no
right to demand their payment after the lapse of the term
of said policy on March 1, 1985. Therefore, Tuscany was
justified in refusing to pay the same.

- CA modified the decision by ordering Tuscany to pay the


balance of the premiums due on the third policy plus legal
interest until fully paid, and affirming the denial of the
counterclaim.

Petitioners Claims
Petitioner argues that where the premiums is not actually
paid in full, the policy would only be effective if there is an
acknowledgment in the policy of the receipt of premium
pursuant to Sec. 78 of the Insurance Code. The absence of
an express acknowledgment in the policies of such receipt
of the corresponding premium payments, and petitioner's
failure to pay said premiums on or before the effective
dates of said policies rendered them invalid. Petitioner thus
concludes that there cannot be a perfected contract of
insurance upon mere partial payment of the premiums
because under Sec. 77 of the Insurance Code, no contract of
insurance is valid and binding unless the premium thereof
has been paid, notwithstanding any agreement to the
contrary.
ISSUE
WON payment by installment of the premiums due on an
insurance policy invalidates the contract of insurance
Page 44 of 154

INSURANCE G01 CASE DIGESTS

HELD
Ratio Where the risk is entire and the contract is indivisible,
the insured is not entitled to a refund of the premiums paid
if the insurer was exposed to the risk insured for any period,
however brief or momentary.

Reasoning
- The obligation to pay premiums when due is ordinarily as
indivisible obligation to pay the entire premium. Here, the
parties herein agreed to make the premiums payable in
installments, and there is no pretense that the parties never
envisioned to make the insurance contract binding between
them. And the insured never informed the insurer that it
was terminating the policy because the terms were
unacceptable.

- Section 78 of the Insurance Code in effect allows waiver by


the insurer of the condition of prepayment by making an
acknowledgment in the insurance policy of receipt of
premium as conclusive evidence of payment so far as to
make the policy binding despite the fact that premium is
actually unpaid. Section 77 merely precludes the parties
from stipulating that the policy is valid even if premiums are
not paid, but does not expressly prohibit an agreement
granting credit extension, and such an agreement is not
contrary to morals, good customs, public order or public
policy.

- At the very least, both parties should be deemed in


estoppel to question the arrangement they have voluntarily
accepted.
Disposition Judgment affirmed. Costs against petitioner.

- There is nothing in Section 77 which suggests that the


parties may not agree to allow payment of the premiums in
installment, or to consider the contract as valid and binding
upon payment of the first premium.

- The records clearly show that petitioner and private


respondent intended subject insurance policies to be
binding and effective notwithstanding the staggered
payment of the premiums. Acceptance of payments speaks
loudly of the insurer's intention to honor the policies it
issued to petitioner.
Page 45 of 154

INSURANCE G01 CASE DIGESTS


94. TIBAY V. COURT OF APPEALS, 257 SCRA 126 (1996)
FACTS:

Insurer: Fortune
Insured: Violeta Tibay and/or Nicolas Ronaldo
Property Insured: 2 storey residential building located in
Makati
Insurance Contract: Fire Insurance Policy for 600,000php
Total Amount of Premium: 2,983.50php but Tibay only paid
600php thus leaving a balance.
Building completely destroyed by fire and two days after,
Tibay paid the unpaid premium and filed with Fortune a
claim on fire insurance policy.

ISSUE:

HELD:

No policy of insurance can ever pretend to be efficacious or


effective until premium has been fully paid.
Premium is the elixir vitae of insurance business, and all
actuarial calculations and various tabulations of
probabilities of losses under the risks insured against are
based on the sound hypothesis of prompt payment of
premium.

95. MALAYAN INSURANCE V. COURT OF APPEALS, 154 SCRA 672


(1987)
MALAYAN INSURANCE CO., INC. v. ARNALDO and PINCA 154 SCRA
672 CRUZ;
October 12, 1987 !

W/N fire insurance policy is valid, binding, and enforceable upon


mere partial payment of premium?

NO
The consideration is the premium, which must be paid at
the time and in the way and manner specified in the policy,
if not paid, the policy will lapse and be forfeited by its own
terms.
Clearly the policy provides for payment in FULL
The premium has only been partially paid and balance paid
only after the peril insured against occurred thus contract
did not take effect and insured cannot collect at all on the
policy. But the controversy lies in the phrase unless and
until the premium thereof has been paid.

FACTS !
- On June 7, 1981, Malayan Insurance Co. (MICO), issued fire
insurance for the amount of P14,000 on the property of private
respondent, Pinca, effective July 1981-1982. MICO later allegedly
cancelled the policy for non-payment of the premium and sent a
notice to Pinca. On Dec. 24 Adora, an agent of MICO, received
Pinca's payment, which was remitted to MICO. On Jan. 18, 1982,
Pinca's property was completely burned. On Feb. 5, MICO
returned Pinca's payment to Adora on the ground that her
policy had been cancelled; the latter refused to accept it. Her
demand for payment having been rejected by MICO, Pinca went to
the Insurance Commission. Public respondent Arnaldo, the
Insurance Commissioner, sustained Pinca, hence this petition
from MICO. Records show MICO received Arnaldo's decision on
Page 46 of 154

INSURANCE G01 CASE DIGESTS


April 10; MICO filed a MFR on April 25 which was denied on June 4;
MICO received notice of this denial on June 14; instant petition was
filed on July 2.

of any court in all cases at 15 days from the notice of the


decision appealed from. Since the MFR was filed only 15 days
after receiving notice of the decision, it was already 18 days late by
July 2. So whichever is applied, the petition is still late.

ISSUES
Procedural
1. WON the petition should be dismissed for late filing
Substantive
2. WON there was a valid insurance contract at the time of the loss
3. WON Adora was authorized to receive such payment
4. WON an adjuster is indispensable in the valuation of the loss

HELD
Procedural
1. YES - Petitioner invokes Sec 416 of the Insurance Code which
grants it 30 days from notice of the Insurance Commission within
which to appeal by certiorari with the Court. MICO filed its MFR on
April 25, 15 days after the notice; the reglementary period began to
run again after June 13. Since the petition was filed only on July 2, it
was tardy by 4 days. Alternatively it invokes Rule 45 of the Rules of
Court for certiorari but the petition still exceeds the 15 day limit
from the June 13 notice. -Respondents, on the other hand, invoke
Sec. 39 of B.P. 129 which pegs the period for appeal from decisions

Substantive
2. YES - A valid cancellation requires the following conditions based
on Sections 64-65 of the Code: prior notice which must be based on
the occurrence of one or more of the grounds mentioned in Sec 64
(in this case, non-payment of premium), after the effective date of
the policy; the notice must be written and mailed to the address on
the policy; it must state the ground(s) for cancellation and the
insurer must furnish details upon the request of the insured. - It
is undisputed that payment of premium was made. Petitioner relies
heavily on Sec 77 of the Insurance Code to contest this, the
said provision requiring payment of premium as soon as the
thing is exposed to the peril insured against and that the policy
is invalid without it. However, this is not applicable in the
instant case as payment was eventually made. It is to be noted that
the premium invoice was stamped "Payment Received#,
indicating an understanding between the parties that payment
could be made later. This is furthered by the fact that Adora had
earlier told her to call him anytime she was ready with her payment.
The Court also finds it strange that MICO only sought to return
Pinca's Jan. 15 payment only on Feb. 5, long after her house had
burned downthis makes petitioner's motives highly suspect. MICO claims to have sent a notice to Pinca, who flatly denied
receiving one. Pinca did not have to prove this since the strict
Page 47 of 154

INSURANCE G01 CASE DIGESTS


language of Sec 64 requires that MICO ensure the cancellation was
actually sent to and received by the insured.
- MICO also suggests that Pinca knew the policy had been
cancelled and was paying the premium in order to renew the
policy. A close study of the transcripts show, however, that Pinca
only meant to renew the policy had it been cancelled but not if it
was still in effectit was conditional. Payment was thus legally
made on the original transaction and validly received by Adora,
who was not informed of the alleged cancellation and thus saw no
reason to reject the payment.
3. YES - Sec. 306 of the Insurance Code provides that any
insurance company that delivers a policy to its agent is deemed
to have authorized such agent to receive payment of premium
on its behalf. It is a well-known principle under the law of
agency that payment to an authorized agent is equivalent to
payment to the principal himself. MICO's acknowledgement of
Adora as its agent thus defeats its contention that he was not
authorized to receive payments on its behalf. 4.
NO - In absence of fraud, the amount of the loss may be
determined on the basis of such proof offered by the insured. Here,
the certification of the Integrated National Police as the extent of
the loss should suffice.
Disposition petition is DENIED.

96. UCPB GENERAL INSURANCE V. MASAGANA TELEMART, 356


SCRA 307 (2001)
Facts:

On April 15, 1991, petitioner issued five (5) insurance


policies covering respondent's various property described
therein against fire, for the period from May 22, 1991 to
May 22, 1992.
In March 1992, petitioner evaluated the policies and
decided not to renew them upon expiration of their terms
on May 22, 1992. Petitioner advised respondent's broker,
Zuellig Insurance Brokers, Inc. of its intention not to renew
the policies.
On April 6, 1992, petitioner gave written notice to
respondent of the non-renewal of the policies at the
address stated in the policies.
On June 13, 1992, fire razed respondent's property covered
by three of the insurance policies petitioner issued.
On July 13, 1992, respondent presented to petitioner's
cashier at its head office five (5) manager's checks in the
total amount of P225,753.95, representing premium for the
renewal of the policies from May 22, 1992 to May 22, 1993.
No notice of loss was filed by respondent under the policies
prior to July 14, 1992.
On July 14, 1992, respondent filed with petitioner its formal
claim for indemnification of the insured property razed by
fire.
On the same day, July 14, 1992, petitioner returned to
respondent the five (5) manager's checks that it tendered,
and at the same time rejected respondent's claim for the
reasons:
o that the policies had expired and were not
renewed, and
Page 48 of 154

INSURANCE G01 CASE DIGESTS


o

that the fire occurred on June 13, 1992, before


respondent's tender of premium payment.
On July 21, 1992, respondent filed with the Regional Trial
Court, Branch 58, Makati City, a civil complaint against
petitioner for recovery of P18,645,000.00, representing the
face value of the policies covering respondent's insured
property razed by fire, and for attorney's fees.
After due trial, on March 10, 1993, the Regional Trial Court,
Branch 58, Makati, rendered decision in favor of the
plaintiff and against the defendant.
In due time, petitioner appealed to the Court of
Appeals. The CA promulgated its decision affirming that of
the RTC.
Hence, this appeal.

Issue: Whether the fire insurance policies issued by petitioner to the


respondent covering the period May 22, 1991 to May 22, 1992, had
expired on the latter date or had been extended or renewed by an
implied credit arrangement though actual payment of premium was
tendered on a later date after the occurrence of the risk (fire)
insured against.
HELD: NO
An insurance policy, other than life, issued originally or on
renewal, is not valid and binding until actual payment of the
premium. Any agreement to the contrary is void. 11The parties may
not agree expressly or impliedly on the extension of creditor time to
pay the premium and consider the policy binding before actual
payment.
The case of Malayan Insurance Co., Inc. vs. Cruz-Arnaldo, 12 cited by
the Court of Appeals, is not applicable. In that case, payment of the
premium was in fact actually made on December 24, 1981, and the
fire occurred on January 18, 1982. Here, the payment of the

premium for renewal of the policies was tendered on July 13, 1992,
a month after the fire occurred on June 13, 1992. The assured did
not even give the insurer a notice of loss within a reasonable time
after occurrence of the fire.

97. AMERICAN HOME INSURANCE V. TANTUCO ENTERPRISES, 366


SCRA 740 (2001)
PREMIUMS
AMERICAN HOME ASSURANCE COMPANY V TANTUCO ENTERPRISES
FACTS:
1. Respondent Tantuco Enterprises, Inc. is engaged in the coconut
oil mining and refining industry.
a. It owns 2 mills.
b. Both located at its factory compound in Lucena City.
2. Respondent Tantuco commenced its business in ONLY ONE OIL
MILL. In 1998, it started operating its second mill and it was referred
to as the new oil mill.
3. The two mills were separately covered by fire insurance policies
issued by Petitioner American Home Assurance Co., Philippine
Branch.
a. FIRST OIL MILL: insured for P.3,000,000.00
b. SECOND OIL MILL: insured for P.6,000,000.00

Page 49 of 154

INSURANCE G01 CASE DIGESTS


c. Official receIpts indicating PAYMENT FOR THE FULL AMOUNT OF
THE PREMIUM were issued by the petitioner's agent.

Whether or not Petitioner American Home Assurance is liable on


the insurance policy?

4. Fire broke out and consumed the new oil mill. Respondent
Tantuco immediately informed the petitioner of the incident.

HELD:

5. PETITIONER REJECTED THE RESPONDEN'D CLAIM FOR THE


INSURANCE PROCEEDS
BASIS: a. that no policy was issued by it covering the burned oil mill.
It stated that the description of the insured establishment referred
to another building (the 2 policies extended the coverage to oil mill
under Building No. 5 but the affected oil mill was under Building no.
14).
b. The respondent is barred by estoppel from claiming that the
description of the insured oil mill in the policy was wrong, because it
retained the policy without having the same corrected before the
fire.
c. The petitioner included the Important Notice in the policy that
states Please read and examine the policy and if incorrect, return it
immediately for alteration.

Yes. The Petitioner American Home Assurance is liable on the


insurance policy.
RATIONALE:
The petition is lack of merit.
In construing the words used descriptive of a building insured, the
greatest liberality is shown by the courts in giving effect to the
insurance. The courts are inclined to consider that the policy of
insurance covers any building which the parties manifestly intended
to insure, however inaccurate the description may be. Also, in
determining what the parties intended, the courts will read and
construe the policy as a whole and if possible, give effect to all the
parts of the contract, that in case there is doubt, this doubt is to be
resolved against the insurer. Object and purpose of the contract
must be considered.
PREMIUM ISSUE:
PETITIONER'S CONTENTIONS:

6. RTC: Petitioner Insurance is liable on the insurance policy.


7. CA: Affirmed the decision of the RTC in toto.

ISSUE:

a. To further attempt to avoid the liability, petitioner claims that


respondent forfeited the renewal policy for it failed to pay the full
amount of the premium and breach of the Fire
Extinguishing Appliances Warranty.

Page 50 of 154

INSURANCE G01 CASE DIGESTS


b. Respondent only paid for P75,147.00 instead of P89,770.20 which
leaves the difference of 14,623.20. The petitioner argues that the
deficiency suffices to invalidate the policy under Section 77 of the
Insurance Code.
c. Also, that even if Mr. Borja, who acknowledged the that the
amount was lacking by reason of rebate is illegal.
SC ruled that it fails to impress. It is true that there is a policy's
condition for payment of the renewal premium on time and the
respondent's non-compliance with it. Yet, it did not contain any
specific and definite allegation that the respondent did not pay the
premium, or that it did not pay the full amount, or that it sis not pay
the amount on time.
The issue of inadequacy of payment was never raised in the pre-trial
proceedings and there is the petitioner failed to present any witness
to testify that respondent indeed failed to pay the full amount of
the premium.
NO BREACH FOR NOT PROVIDING FIRE EXTINGUISHING
APPLIANCES.

98. GREAT PACIFIC LIFE INSURANCE V. COURT OF APPEALS, 184


SCRA 501 (1990)

Facts:
1. This case involves an insured's claim for refund of the first
premium on the endowment policy on his life, upon being
notified by the insurer that the policy never took effect
despite the premium payment.
2. Teodoro Cortez applied for a 20-year endowment policy for
30,000.
3. His application was accepted, approved and a policy was
issued.
4. The policy was delivered to him by the underwriter
Margarita Seiga.
5. The effective date indicated on the face of the policy in
question was December 25, 1972. The annual premium was
P1,416.60. Mrs. Siega assured him that the first premium
may be paid within the grace period of thirty (30) days from
date of delivery of the policy
6. The first premium was then paid in 3 installments.
7. In a letter dated June 1, 1973 (Exh. E), defendant advised
plaintiff that Policy No. 221944 (Exh. A) was not in force. To
make it enforceable and operative, plaintiff was asked to
remit the balance of P1,015.60 to complete his initial annual
premium due December 15, 1972, and to see Dr. Felipe V.
Remollo for another full medical examination at his own
expense.
8. Cortez' reaction to the company's act was to immediately
inform it that he was cancelling the policy and he
demanded the return of his premium plus damages.
9. RTC Ruled in favour of Cortez
10. CA Affirmed
Issue:
Page 51 of 154

INSURANCE G01 CASE DIGESTS


1. Whether or not Cortez is entitled to a refund of his
premium.
Held:
1. YES
2. When the petitioner advised private respondent on June 1,
1973, four months after he had paid the first premium, that
his policy had never been in force, and that he must pay
another premium and undergo another medical
examination to make the policy effective, the petitioner
committed a serious breach of the contract of insurance.
3. Petitioner should have informed Cortez of the deadline for
paying the first premium before or at least upon delivery of
the policy to him, so he could have complied with what was
needful and would not have been misled into believing that
his life and his family were protected by the policy, when
actually they were not.
4. And, if the premium paid by Cortez was unacceptable for
being late, it was the company's duty to return it.
5. By accepting his premiums without giving him the
corresponding protection, the company acted in bad faith.
6. Sections 79, 81 and 82 of P.D. 612 of the Insurance Code of
1978 provide when the insured is entitled to the return of
premium paid.
7. Since his policy was in fact inoperative or ineffectual from
the beginning, the company was never at risk, hence, it is
not entitled to keep the premium.

99. STOKES V. MALAYAN, 127 SCRA 766 (1984)


FACTS:
1. Daniel Adolfson had a subsisting Malayan car insurance
policy with coverage against own damage as well as 3rd
party liability when his car figured in a vehicular accident
with another car, resulting to damage to both vehicles.
2. At the time of the accident, Adolfsons car was being driven
by James Stokes, who was authorized to do so by Adolfson.
Stokes, an Irish tourist who had been in the Philippines for
only 90 days, had a valid and subsisting Irish drivers license
but without a Philippine drivers license.
3. Adolfson filed a claim with Malayan but the latter refused to
pay contending that Stokes was not an authorized driver
under the Authorized Driver clause of the insurance policy
in relation to Section 21 of the Land Transportation Office.
ISSUE: Whether or not Malayan is liable to pay the insurance claim
of Adolfson
HELD: NO. A contract of insurance is a contract of indemnity upon
the terms and conditions specified therein. When the insurer is
called upon to pay in case of loss or damage, he has the right to
insist upon compliance with the terms of the contract. If the insured
cannot bring himself within the terms and conditions of the
contract, he is not entitled as a rule to recover for the loss or
damage suffered. For the terms of the contract constitute the
measure of the insurers liability, and compliance therewith is a
condition precedent to the right of recovery.
At the time of the accident, Stokes had been in the Philippines for
more than 90 days. Hence, under the law, he could not drive a
motor vehicle without a Philippine drivers license. He was therefore
not an authorized driver under the terms of the insurance policy
in question, and Malayan was right in denying the claim of the
insured.
Page 52 of 154

INSURANCE G01 CASE DIGESTS


100. CAPITAL INSURANCE V. PLASTIC ERA, 65 SCRA 134 (1975)
Acceptance of premium within the stipulated period for payment
thereof, including the agreed period of grace, merely assures
continued effectivity of the insurance policy in accordance with its
terms. Such acceptance does not estop the insurer from interposing
any valid defense under the terms of the insurance policy.
The principle of estoppel is an equitable principle rooted upon
natural justice which prevents a person from going back on his own
acts and representations to the prejudice of another whom he has
led to rely upon them. The principle does not apply to the instant
case. In accepting the premium payment of the insured, Malayan
was not guilty of any inequitable act or representation. There is
nothing inconsistent between acceptance of premium due under an
insurance policy and the enforcement of its terms.

GR No. L-22375 July 18, 1975


Facts:
Capital Insurance delivered to Plastic Era its open fire policy.
Capital Insurance undertook to insure Plastic Era's building,
equipments, raw materials, products and accessories
located in Mandaluyong, Rizal.
The policy expressly provided that if the property insured
would be destroyed or damaged by fire after the payment
of the premiums, at any time between Dec. 15, 1960 and
1PM of Dec. 15, 1961, the insurance company shall make
good all such loss or damage in an amount not exceeding
100K.
Plastic Era failed to pay the corresponding insurance
premium. However, its duly authorized representative
executed an acknowledgment receipt with a corresponding
'promise to pay.'
Plastic Era delivered to Capital Insurance a check for the
amount of 1K, as partial payment of the premium,
postdated Jan 16, 1961 and drawn against the Bank of
America. When Capital Insurance tried to deposit the check
a month after the date, it was dishonored for lack of funds.
January 18, 1961, 2 days after the premium was due, the
property insured by Plastic Era was destroyed by fire. They
notified Capital of the loss and filed its claim for indemnity
through the Manila Adjustment Company, who estimated
the loss at P283,875.
The same property was insured with Philamgen for 200K.
Plastic Era demanded from Capital the payment of the sum
of 100K as indemnity, but was denied because they had
failed to pay the premium.
Issue:
Page 53 of 154

INSURANCE G01 CASE DIGESTS


Whether there was a perfected contract of insurance between
Capital and Plastic Era
Whether there was payment of premium (either through the
acknowledgment receipt with a promise to pay, or through the
postdated check)

Ruling:
Capital Insurance was held to be liable to Plastic Era
It is clear from the terms of the policy that it is only upon
payment of the premiums that Capital agrees to insure the
properties of Plastic Era against loss or damage in an
amount not exceeding P100K.
The only question/issue is whether there was payment.
The issuance of the postdated check did not produce the
effect of payment.
o The mere delivery of a bill of exchange in payment
of a debt does not immediately effect payment. It
simply suspends the action arising from the original
obligation in satisfaction of which it was delivered,
until payment is accomplished either actually or
presumptively. It shall only produce the effect of
payment when they have been encashed, or when
through the fault of the creditor they have been
impaired.
However, in Capital's acceptance of the promissory note, it
implicitly agreed to modify the tenor of the insurance policy
and in effect, waived the provision that it would only pay for
the loss or damage in case the same occurs after the
payment of the premium.
The insurance policy is silent as to the mode of payment.
Therefore, Capital Insurance is deemed to have accepted
the promissory note in payment of the premium. This
acceptance of the note rendered the policy immediately
operative on the date it was delivered.

When the damage or loss of the insured property occurred,


the insurance policy was in full force and effect. The fact
that the check issued by Plastic Era in partial payment of the
promissory note was later on dishonored did not in any way
operate as a forfeiture of its rights under the policy, there
being no express stipulation therein to that effect.
Moreover, by accepting its promise to pay the insurance
premium within 30 days from the effectivity date of the
policy, Capital Insurance had in effect extended credit to
Plastic Era. The payment of the premium on the insurance
policy therefore became an independent obligation the
non-fulfillment of which would entitle Capital Insurance to
recover. It did not have the right to cancel the policy for
nonpayment of the premium except by putting Plastic Era in
default and giving it personal notice to that effect.
Capital is likewise estopped from claiming a forfeiture of its
policy for non-payment even if the check was dishonored.
Although the check was due for payment on Jan 16, and
Plastic Era had sufficient funds during that date, Capital
decided to hold the check of 35 days before presenting it
for payment.

Page 54 of 154

INSURANCE G01 CASE DIGESTS

LOSS

AUTHORIZED DRIVER:
Any of the following:

101.

CCC INSURANCE V. COURT OF APPEALS, 31 SCRA 264

Facts:
1. Carlos F. Robes took an insurance, with the CCC Insurance
Corporation, on his Dodge Kingsway car against loss or
damage through accident for an amount not exceeding
P8,000.00
2. The insured vehicle, while being driven by the owner's
driver, became involved in a vehicular collision along Rizal
Avenue Extension, Potrero, Malabon, Rizal. The car was
damaged, and the repair was estimated to cost P5,300.00.
3. The insurance company refused to pay for the repair or
restore the car.
4. The insurance company disclaimed liability for payment,
alleging that there had been violation of the insurance
contract because the one driving the car at the time of the
incident was not an "authorized driver."
5. RTC Ruled for plaintiff and ordered defendant insurer to
pay.
6. The CA affirmed the RTC
Issue:
1.

Whether or not the damage to the insured car was not


covered by the insurance policy because at the time of the
accident it was being driven by one who was not an
authorized driver.

Held:

(a) The insured;


(b) Any person driving on the Insured's order or with his
permission, provided that the person driving
is permitted in accordance with licensing laws or
regulations to drive the motor vehiclecovered by this
Policy, or has been so permitted and is not disqualified
by order of a court of law or by reason of any
enactment or regulation from driving such Motor
Vehicle. (Emphasis ours)
It has been found as a fact by the Court of Appeals that
Domingo Reyes, the, driver who was at the wheel of the
insured car at the time of the accident, does not know how
to read and write; that he was able to secure a driver's
license, without passing any examination therefor, by
paying P25.00 to a certain woman
4. The appellant insurer insists that, under the established
facts of this case, Reyes, being admittedly one who cannot
read and write, who has never passed any examination for
drivers, and has not applied for a license from the duly
constituted government agency entrusted with the duty of
licensing drivers, cannot be considered an authorized driver.
5. Under Section 24 of the Revised Motor Vehicles Law, Act
3992 of the Philippine Legislature, as amended by Republic
Acts Nos. 587, 1204 and 2863, the issuance of a driving
license without previous examination does not necessarily
imply that the license issued is invalid.
3.

The damage is under the insurance policy. The driver falls


under the definition of an authorized driver
2. Under the law
1.

Page 55 of 154

INSURANCE G01 CASE DIGESTS

102. COUNTRY BANKERS INSURANCE V. LLANGA BAY AND COMMUNITY


MULTI-PURPOSE COOPERATIVE , 374 SCRA 653 (2002)
COUNTRY BANKERS INSURANCE CORPORATION, petitioner,
vs. LIANGA BAY AND COMMUNITY MULTI-PURPOSE
COOPERATIVE, INC., respondent.
DECISION

DE LEON, JR., J.:


FACTS:
~Petitioner Country Bankers is a domestic corporation principally
engaged in the insurance business.

This insurance does not cover any loss or damage


occasioned by or through or in consequence,
directly or indirectly, of any of the following
occurrences, namely:
xxx
xxx
(d) Mutiny, riot, military or popular uprising,
insurrection, rebellion, revolution, military or
usurped power.

~Respondent filed in the trial court the complaint for recovery of


loss, damage or liability against petitioner.

~Petitioner and the respondent entered into a 1-year fire insurance


contract over respondents stocks-in-trade (inside their building at
Lianga Bay, Surigao del Sur) for the sum of Two Hundred Thousand
Pesos (P200,000.00).

RTC --- Trial court decided in favor of the respondent ordering


petitioner to pay respondent P200,000.00, with interest at twelve
percent (12%) per annum from the date of filing of the complaint
until fully paid, as well as Fifty Thousand Pesos (P50,000.00) as
actual damages, Fifty Thousand Pesos (P50,000.00) as exemplary
damages, Five Thousand Pesos (P5,000.00) as litigation expenses,
Ten Thousand Pesos (P10,000.00) as attorneys fees, and the costs
of suit.

~Respondents building was gutted by fire and reduced to ashes.

CA --- affirmed

~Respondent filed an insurance claim submitting: (a) the Spot


Report of Pfc. Arturo V. Juarbal, INP Investigator; (b) the Sworn
Statement of Jose Lomocso; and (c) the Sworn Statement of Ernesto
Urbiztondo.

ISSUE: Whether petitioner Country Bankers is liable to pay the


claim plus interest, damages, attorneys fees and costs of suit.

~Respondent Lianga Coop is a duly registered cooperative judicially


declared insolvent.

~The petitioner denied the insurance claim on the ground that,


based on the submitted documents, the building was set on fire by
two (2) NPA rebels who wanted to obtain canned goods, rice and
medicines as provisions for their comrades in the forest, and that
such loss was an excepted risk under paragraph No. 6 of the policy,
which states:

HELD: NO, Country Bankers is liable only for the insurance claim
but not for 12% interest, damages and fees.
Since petitioners defense is non-coverage by reason of the
exemption or exception clause in the fire insurance policy, it has the
Page 56 of 154

INSURANCE G01 CASE DIGESTS


burden of proving the facts upon which such excepted risk is based,
by a preponderance of evidence.[7] But petitioner failed to do so.
The petitioner relied on the Sworn Statements of Lomocso and
Urbiztondo and the Spot Report of Pfc. Arturo V. Juarbal.
But the Sworn Statements were found to be inadmissible in
evidence, for being hearsay, inasmuch as they did not take the
witness stand and could not therefore be cross-examined.
As to Investigator Juarbals Spot Report, it was based on the
personal knowledge of the caretaker Lomocso who witnessed every
single incident surrounding the facts and circumstances of the case.
And so said Spot Report was also inadmissible for being hearsay as it
was not based on the investigators personal knowledge although
he took the witness stand.
The petitioners evidence to prove its defense is sadly wanting
and thus, it is liable to the respondent under the Fire Insurance
Policy. But we do not sustain the trial courts imposition of 12%
interest on the insurance claim as well as the monetary award for
actual and exemplary damages, litigation expenses and attorneys
fees for lack of legal and valid basis.
Concerning the application of the proper interest rates, the
following guidelines were set in Eastern Shipping Lines, Inc. v. Court
of Appeals and Mercantile Insurance Co., Inc.:[15]
I.
When an obligation, regardless of its source,
i.e., law, contracts, quasi-contracts, delicts or quasidelicts, is breached, the contravenor can be held
liable for damages. The provisions under Title XVIII
on Damages of the Civil Code govern in
determining the measure of recoverable damages.
II.
With regard particularly to an award of
interest in the concept of actual and compensatory
damages, the rate of interest, as well as the accrual
thereof, is imposed, as follows:

1.
When the obligation is breached, and it
consists in the payment of a sum of money, i.e., a
loan or forbearance of money, the interest due
should be that which may have been stipulated in
writing. Furthermore, the interest due shall itself
earn legal interest from the time it is judicially
demanded. In the absence of stipulation, the rate
of interest shall be 12% per annum to be computed
from default, i.e., from judicial or extrajudicial
demand under and subject to the provisions of
Article 1169 of the Civil Code.
2.
When an obligation, not constituting a loan
or forbearance of money, is breached, an interest
on the amount of damages awarded may be
imposed at the discretion of the court at the rate of
6% per annum. No interest, however, shall be
adjudged on unliquidated claims or damages except
when or until the demand can be established with
reasonable certainty. Accordingly, where the
demand is established with reasonable certainty,
the interest shall begin to run from the time the
claim is made judicially or extrajudicially (Art. 1169,
Civil Code) but when such certainty cannot be so
reasonably established at the time the demand is
made, the interest shall begin to run only from the
date the judgment of the court is made (at which
time the quantification of damages may be deemed
to have been reasonably ascertained). The actual
base for the computation of legal interest shall, in
any case, be on the amount finally adjudged.
3.
When the judgment of the court awarding
a sum of money becomes final and executory, the
rate of legal interest, whether the case falls under
paragraph 1 or paragraph 2, above, shall be 12% per
annum from such finality until its satisfaction, this
Page 57 of 154

INSURANCE G01 CASE DIGESTS


interim period being deemed to be by then an
equivalent to a forbearance of credit.

The insurance claim in this case is evidently not a forbearance


of money, goods or credit, and thus the interest rate should be 6%
from the date of filing of the complaint.
We find no justification for the award of actual
damages. Well-entrenched is the doctrine that actual,
compensatory and consequential damages must be proved, and
cannot be presumed. The justification, if any, for such an award of
actual damages does not appear in the body of the decision of the
trial court.
We likewise find no legal and valid basis for granting
exemplary damages. Article 2229 of the New Civil Code provides
that exemplary damages may be imposed by way of example or
correction for the public good. However, it cannot be recovered as a
matter of right. It is based entirely on the discretion of the court.
And we find no justification for the award of litigation
expenses and attorneys fees. Article 2208 of the New Civil Code
enumerates the instances where such may be awarded and, in all
cases, it must be reasonable, just and equitable if the same were to
be granted. The award of attorneys fees is the exception rather
than the general rule. As such, it is necessary for the court to make
findings of facts and law that would bring the case within the
exception and justify the grant of such award. We find none in this
case.
WHEREFORE, the appealed Decision is MODIFIED. The rate of
interest on the adjudged principal amount of Two Hundred
Thousand Pesos (P200,000.00) shall be six percent (6%) per annum
computed from the date of filing of the Complaint in the trial court.
The awards in the amounts of Fifty Thousand Pesos (P50,000.00) as
actual damages, Fifty Thousand Pesos (P50,000.00) as exemplary

damages, Five Thousand Pesos (P5,000.00) as litigation expenses,


and Ten Thousand Pesos (P10,000.00) as attorneys fees are hereby
DELETED. Costs against the petitioner.
SO ORDERED.

103. PARIS-MANILA PERFUMERY V. PHOENIX ASSURANCE , 49 PHIL.


753 (1926)

Paris-Manila Perfume Co. v. Phoenix Assurance Co.


G.R. No. L-25845
December 17, 1926
Lessons Applicable: Loss, the immediate cause of which was the
peril insured against, if the proximate cause thereof was NOT
excepted in the contract (Insurance)
Facts:

May 22, 1924: A fire insurance policy was issued by Phoenix


Assurance Company, Limited to Messrs. Paris-Manila
Perfumery Co. (Peter Johnson, Prop.) for P13,000
o also insured with other insurance companies for
P1,200 and P5,000 respectively
July 4, 1924: The Perfumery was burned unknown of the
cause totalling a loss of P38.025.56
Phoenix refused to pay nor to appoint an arbitrator stating
that the policy did not cover any loss or damage occasioned
by explosion and stating that the claim was fraudulent
RTC: ordered Phoenix to pay P13,000
Phoenix appealed
o The insurance policy contains:
Page 58 of 154

INSURANCE G01 CASE DIGESTS


Unless otherwise expressly stated in the policy the
insurance does not cover
(h) Loss or damage occasioned by the explosion; but loss
or damage by explosion of gas for illuminating or domestic
purposes in a building in which gas is not generated and
which does not form a part of any gas works, will be
deemed to be loss by fire within the meaning of this policy.
ISSUE:

104.

FGU INSURANCE V. COURT OF APPEALS, 454 SCRA 337 (2005)

FGU Insurance vs. Court of Appeals


G.R. No. 137775; March 31, 2005

Facts:
Anco Enterprises, a partnership between Ang Gui and Co To, was

W/N Phoenix should be liable for the loss because there was no
explosion which is an exemption from the policy

engaged in the shipping business. It owned the M/T ANCO


tugboat and the D/B Lucio barge. Since the D/B Lucio had no
engine of its own, it could not maneuver by itself and had to be
towed by a tugboat for it to move from one place to another.

HELD:
On Sept. 23, 1979, San Miguel Corporation shipped from

YES.

If it be a fact that the fire resulted from an explosion that


fact, if proven, would be a complete defense, the burden of
the proof of that fact is upon the defendant, and upon that
point, there is a failure of proof
lower court found as a fact that there was no fraud in the
insurance, and that the value of the property destroyed by
the fire was more than the amount of the insurance.

Mandaue City, Cebu on board the D/B Lucio, for towage by M/T
ANCO, the following cargoes:

25,000 cases Pale Pilsen and 350


cases Cerveza Negra - consignee SMCs
Beer Marketing Division (BMD)Estancia Beer Sales Office, Estancia,
Iloilo
15,000 cases Pale Pilsen and 200
cases Cerveza Negra - consignee SMCs
BMD-San Jose Beer Sales Office, San
Jose, Antique
The D/B Lucio was towed by the M/T ANCO all the way from

Mandaue City, Cebu to San Jose, Antique. It arrived at San Jose


at around 1:00 PM of September 30, 1979.
Page 59 of 154

INSURANCE G01 CASE DIGESTS

In the afternoon of the same day, the clouds over the area were

dark and the waves were already big. The arrastre workers
unloading the cargoes of SMC on board the D/B Lucio began to
complain about their difficulty in unloading the cargoes. SMCs
District Sales Supervisor, Fernando Macabuag, requested ANCOs
representative to transfer the barge to a safer place because the
vessel might not be able to withstand the big waves.
ANCOs representative did not heed the request because he was

confident that the barge could withstand the waves. At that


time, only the M/T ANCO was left at the wharf of San Jose,
Antique, as all other vessels already left the wharf to seek
shelter. With the waves growing bigger and bigger, only 10,790
cases of beer were discharged into the custody of the arrastre
operator.
At about 10:00 to 11:00 pm of October 1, 1979, the crew of D/B

Lucio abandoned the vessel because the barges rope attached


to the wharf was cut off by the big waves. At around midnight,
the barge run aground and was broken and the cargoes of beer
in the barge were swept away.
As a result, ANCO failed to deliver to SMCs consignee 29,210

cases of Pale Pilsen and 550 cases of Cerveza Negra. The value
per case of Pale Pilsen was P42.50. The value of a case of
Cerveza Negra was P47.10, hence, SMCs claim against ANCO
amounted P1,346,197.00.
SMC filed a complaint for breach of contract of carriage against

ANCO for the amount of P1,346,197. Upon Ang Guis death,

ANCO, as a partnership, was dissolved hence, on January 26,


1993, SMC filed a second amended complaint which was
admitted by the Court impleading the surviving partner, Co To
and the Estate of Ang Gui represented by Lucio, Julian and Jaime,
all surnamed Ang.
ANCO admitted that the cases of beer Pale Pilsen and Cerveza

Negra mentioned in the complaint were indeed loaded on the


vessel belonging to ANCO. It claimed however that it had an
agreement with SMC that ANCO would not be liable for any
losses or damages resulting to the cargoes by reason of
fortuitous event. Since the cases of beer Pale Pilsen and Cerveza
Negra were lost by reason of a storm, a fortuitous event which
battered and sunk the vessel in which they were loaded, they
should not be held liable.
ANCO further asserted that there was an agreement between
them and SMC to insure the cargoes in order to recover
indemnity in case of loss. Pursuant to that agreement, the
cargoes to the extent of 20,000 cases was insured with FGU
Insurance Corporation (FGU) for the total amount of
P858,500.00 per Marine Insurance Policy No. 29591.
In its answer to the Third-Party complaint, FGU admitted the

existence of the Insurance Policy but maintained that the alleged


loss of the cargoes covered by the said insurance policy cannot
be attributed directly or indirectly to any of the risks insured
against in the said insurance policy. According to FGU, it is only
liable under the policy to Third-party Plaintiff ANCO and/or
Plaintiff SMC in case of any of the following:

a) Total loss of the entire shipment;


b) Loss of any case as a result of the sinking of the vessel; or
c) Loss as a result of the vessel being on fire.
Page 60 of 154

INSURANCE G01 CASE DIGESTS

Furthermore, FGU alleged that the Third-Party Plaintiff ANCO

and Plaintiff SMC failed to exercise ordinary diligence or the


diligence of a good father of the family in the care and
supervision of the cargoes insured to prevent its loss and/or
destruction.
The RTC held that while the cargoes were indeed lost due to

fortuitous event, there was failure on ANCOs part, through their


representatives, to observe the degree of diligence required that
would exonerate them from liability. The trial court thus held
the Estate of Ang Gui and Co To liable to SMC for the amount of
the lost shipment.
With respect to the Third-Party complaint, the RTC found FGU

liable to 53% of the amount of the lost cargoes. It was upheld by


the Court of Appeals.
Issue: Whether or not FGU can be held liable under the insurance
policy to reimburse ANCO for the loss of the cargoes despite the
findings of the Court of Appeals that such loss was occasioned by
the blatant negligence of the latters employees.
Held: NO. The third-party complaint against FGU insurance is
dismissed but ANCO is still liable to SMC.
Ratio:
To be exempt from responsibility, the natural disaster should
have been the proximate and only cause of the loss. But,
according to the Court of Appeals, there was blatant negligence
on the part of M/T ANCOs crewmembers, first in leaving the
engine-less barge D/B Lucio at the mercy of the storm without
the assistance of the tugboat, and again in failing to heed the
request of SMCs representatives to have the barge transferred
to a safer place, as was done by the other vessels in the port;

thus, making said blatant negligence the proximate cause of the


loss of the cargoes.
One of the purposes for taking out insurance is to protect the
insured against the consequences of his own negligence and that
of his agents. Thus, it is a basic rule in insurance that the
carelessness and negligence of the insured or his agents
constitute no defense on the part of the insurer. This rule
however presupposes that the loss has occurred due to causes
which could not have been prevented by the insured, despite the
exercise of due diligence.
The question now is whether there is a certain degree of
negligence on the part of the insured or his agents that will
deprive him the right to recover under the insurance contract.
We say there is. However, to what extent such negligence must
go in order to exonerate the insurer from liability must be
evaluated in light of the circumstances surrounding each case.
When evidence show that the insureds negligence or
recklessness is so gross as to be sufficient to constitute a willful
act, the insurer must be exonerated.
The United States Supreme Court has made a distinction
between ordinary negligence and gross negligence or negligence
amounting to misconduct and its effect on the insureds right to
recover under the insurance contract. According to the Court,
while mistake and negligence of the master or crew are incident
to navigation and constitute a part of the perils that the insurer
is obliged to incur, such negligence or recklessness must not be
of such gross character as to amount to misconduct or wrongful
acts; otherwise, such negligence shall release the insurer from
liability under the insurance contract.

Page 61 of 154

INSURANCE G01 CASE DIGESTS


In the case at bar, both the trial court and the appellate court
had concluded from the evidence that the crewmembers of both
the D/B Lucio and the M/T ANCO were blatantly negligent. To
wit:
There was blatant negligence on the part of the
employees of defendants-appellants when the patron
(operator) of the tug boat immediately left the barge at
the San Jose, Antique wharf despite the looming bad
weather. Negligence was likewise exhibited by the
defendants-appellants representative who did not heed
Macabuags request that the barge be moved to a more
secure place. The prudent thing to do, as was done by
the other sea vessels at San Jose, Antique during the
time in question, was to transfer the vessel to a safer
wharf. The negligence of the defendants-appellants is
proved by the fact that on 01 October 1979, the only
simple vessel left at the wharf in San Jose was the D/B
Lucio.

Page 62 of 154

INSURANCE G01 CASE DIGESTS

NOTICE AND PROOF OF LOSS


105. MALAYAN INSURANCE VS. CRUZ-ARNALDO, 154 SCRA 672
(1987)
MALAYAN INSURANCE CO., INC. (MICO), petitioner, vs. GREGORIA
CRUZ ARNALDO, in her capacity as the INSURANCE
COMMISSIONER, and CORONACION PINCA, respondents.
G.R. No. L-67835 October 12, 1987
Facts:
1. Jun 7 1981: MICO issued to Coronacion Pinca a Fire Insurance
Policy for her property effective July 22, 1981 to July 22, 1982
2. Oct 151981: MICO allegedly cancelled the policy for nonpayment, of the premium and sent the corresponding notice to
Pinca
3. Dec 24 1981: payment of the premium for Pinca was received
by Domingo Adora, as agent of MICO
4. Jan 15 1982: Adora remitted this payment to MICO, together
with other payments
5. Jan 18 1982: Pinca's property was completely burned
6. Feb 5 1982: Pinca's payment was returned by MICO to Adora on
the ground that her policy had been cancelled earlier but Adora
refused to accept it and instead demanded for payment
7. MICO filed its MR on April 25, 1981 (or 15 days after such
notice)
a. The reglementary period began to run again after Jun
13, 1981 (date of its receipt of notice of the denial of
the said MR)
b. Under Section 416 of the Insurance Code, the period for
appeal is thirty days from notice of the decision of the
Insurance Commission.

c. As the herein petition was filed on July 2, 1981, or 19


days later, there is no question that it is tardy by four
days.
8. Insurance Commission ruling: in favor of Pinca
9. Hence this appeal
a. MICO contends that it should not be held liable
because:
i. Adora was not authorized to receive premium
payment because 6 months had elapsed since
the issuance by the policy itself... this
prohibition was binding upon Pinca, who made
the payment to Adora at her own risk as she
was bound to first check his authority to receive
it
ii. No payment of premium and that the policy had
been cancelled before the occurrence of the
loss
iii. The claim was allowed without sufficient proof
of loss

ISSUE: (We will now focus on the final issue raised by MICO, the
proper valuation of the loss incurred.) Whether the valuation fixed
based on the certification issued by the Integrated National Police is
sufficient
HELD: Yes, petition DENIED.
RATIO:
1.

The last point raised by the petitioner should not pose much
difficulty. The valuation fixed in fire insurance policy is
conclusive in case of total loss in the absence of fraud,
which is not shown here.
Page 63 of 154

INSURANCE G01 CASE DIGESTS

2.

3.

4.

a. Loss and its amount may be determined on the


basis of such proof as may be offered by the
insured, which need not be of such persuasiveness
as is required in judicial proceedings.
b. If, as in this case, the insured files notice and
preliminary proof of loss and the insurer fails to
specify to the former all the defects thereof and
without unnecessary delay, all objections to notice
and proof of loss are deemed waived under Section
90 of the Insurance Code.
The certification 26 issued by the Integrated National
Police, Lao-ang, Samar, as to the extent of Pinca's loss
should be considered sufficient. Notably,MICO submitted
no evidence to the contrary nor did it even question the
extent of the loss in its answer before the Insurance
Commission. It is also worth observing that Pinca's property
was not the only building bumed in the fire that razed the
commercial district of Lao-ang, Samar, on January 18, 1982.
There is nothing in the Insurance Code that makes the
participation of an adjuster in the assessment of the loss
imperative or indespensable, as MICO suggests. Section
325, which it cites, simply speaks of the licensing and duties
of adjusters.
We see in this cases an obvious design to evade or at least
delay the discharge of a just obligation through efforts
bordering on bad faith if not plain duplicity, We note that
the motion for reconsideration was filed on the fifteenth
day from notice of the decision of the Insurance
Commission and that there was a feeble attempt to show
that the notice of denial of the said motion was not
received on June 13, 1982, to further hinder the

proceedings and justify the filing of the petition with this


Court fourteen days after June 18, 1982. We also look
askance at the alleged cancellation, of which the insured
and MICO's agent himself had no knowledge, and the
curious fact that although Pinca's payment was remitted to
MICO's by its agent on January 15, 1982, MICO sought to
return it to Adora only on February 5, 1982, after it
presumably had learned of the occurrence of the loss
insured against on January 18, 1982. These circumstances
make the motives of the petitioner highly suspect, to say
the least, and cast serious doubts upon its candor and bona
fides.

Page 64 of 154

INSURANCE G01 CASE DIGESTS


106. YU BAN CHUAN V. FIELDMENS INSURANCE , 14 SCRA 491
(1965)
Yu Ban Chuan vs. Fieldmen's Insurance Co. Inc.
G.R. No. L-19851. June 29, 1965
Facts:
On March, 1959, plaintiff Yu Ban Chuan began his business
enterprise under the name of "CMC Trading," which was engaged in
the wholesale dealing of general merchandise and school supplies.
The plaintiff insured against fire the stock merchandise with
defendant Fieldmen's
Insurance Co. Inc. on December 14, 1959, with an "open" policy
limiting the insurer's liability to the amount of P200,000 for a period
of one year. The plaintiff again insured against fire the same stock of
merchandise with another company which was Paramount Surety &
Insurance Co., on January 7, 1960, with another "open" policy
limiting liability thereunder to P140,000 for also a period of one
year.
Plaintiff transferred his business establishment. Both Fieldmen's and
Paramount acknowledged the existence of its co-insurance and
both agreed to transfer the coverage of its respective insurance
policies to the plaintiff's new store.
On January 31, 1960, while both insurance policies were in full force
and effect, plaintiff's business establishment was totally destroyed
by fire. The next day after the occurrence of the fire, plaintiff
verbally notified the respective agents of the defendants- insurers
of such incident; and on the same day, plaintiff and H. H. Bayne

Adjustment Co. and Manila Adjustment Co., adjusters of defendants


Fieldmen's and Paramount, respectively, executed "non-waiver"
agreements for the purpose of determining the circumstances of
the fire and the value or amount of loss and damage to the
merchandise insured under said policies.
Pursuant to such agreements, H. H. Bayne Adjustment Co. and
Manila Adjustment Co. sent letters requiring the plaintiff to submit
certain papers and documents. On February
8, 1960, plaintiff gave a written notice of the occurrence of the fire
to the defendants, and, in answer to the letters of the adjusters,
plaintiff submitted his separate formal fire claims, together with
some of the supporting papers that where required.
Because of plaintiff's non-compliance or failure to submit the
required documents the defendants rejected plaintiff's claims, and
denied liability under their respective policies, evidently upon their
respective adjusters' recommendations.
The plaintiff commenced suit in the Court of First Instance of
Manila. The defendants answered the complaint with identical
special defenses; to wit:
1) insured's failure to prove the loss claimed;
2) false and fraudulent claim; and
3) arson or causes not independent of the will of the insured;
After trial, the CFI upheld the claim of the plaintiff, but refused to
award damages or interest at more than the legal rate. Both parties
appealed. In proving the value of his loss, the plaintiff relied upon a
merchandise inventory as of December 31, 1959, which he had
Page 65 of 154

INSURANCE G01 CASE DIGESTS


allegedly submitted on January 15, 1960 to the Bureau of Internal
Revenue.
The inventory reflected the total value of stocks at P328,202.67. The
plaintiff also claimed purchases for the month of January 1960 in
the amount of P34,505.08 and sales in the amount of P12,000, thus
the resulting balance of the stocks allegedly burned was estimated
by the plaintiff to be P350,707.75.
Issue:
Whether or not plaintiff's appeal for the award of damages at more
than the legal rate must be allowed? No.
Ratio:
The fact of the filing of the inventory as of January 15, 1960 should
be considered as true, since there was no evidence to the contrary
that would necessarily show the falsity of the inventory as of the
date stamped therein as certified by the chief of the administrative
division.
The court a quo, however, committed error in accepting as true the
actual existence at the burned premises of the stocks mentioned in
the inventory. Six of the many copies of the invoices submitted by
the plaintiff to the adjusters uncover a clear case of fraud and
misrepresentation as to the liability of Fieldmen's and Paramount's
respective policies.
Five of the invoices alone inflate the supposed stocks by
P248,370.00. On sight, the exhibit excites incredulity. The plaintiff,
Yu Ban Chuan, adopted a uniform, too uniform, in fact, to be
believed, explanation for all the invoices, that he did not buy the

merchandise at the companies' addresses but bought from the


agents who brought the goods to him and that the originals of the
invoices were burned and that he requested for true copies from
the agents whom he met casually in the streets after the fire and
these agents delivered the exhibits to him but he did not remember
or know the names of these agents, nor did he know their
whereabouts. In other words, he wants the court to believe also
that these agents performed a vanishing act after each one of them
had turned in the copy of each invoice to the plaintiff.
The plaintiff adhered to the inventory as the immaculate basis for
the actual worth of stocks that were burned, on the ground that it
was made from actual count, and in compliance with law. But this
inventory is not binding on the defendants, since it was prepared
without their intervention.
It is well to note that plaintiff had every reason to show that the
value of his stock of goods exceeded the amount of insurance that
he carried. The inventory having been made prior to the fire, was no
proof of the existence of these goods at the store when the fire
occurred.
True, there were merchandise that were actually destroyed by fire,
but when fraud is conceived, what is true is subtly hidden by the
schemer beneath proper and legal appearances, including the
preparation of the inventory.
Shielding himself under Section 82 of the Insurance Act, the plaintiff
asserts that in submitting his proof of loss he was "not bound to
give such proof as would be necessary in a court of justice". The
assertion is correct, but does not give him any justification for
Page 66 of 154

INSURANCE G01 CASE DIGESTS


submitting false proofs. Their falsity is the best evidence of the
fraudulent character and the unmeritoriousness of plaintiff's claim.
For the foregoing reasons, the appealed judgment was hereby
reversed, and the plaintiff's appeal against the non-award of
damages to him must be dismissed.

107.

GO LU V. YORKSHIRE INSURANCE , 43 PHIL. 633 (1922)


GO LU V. YORKSHIRE
(Proof of Loss)
FACTS

1. Go Lu was a merchant buying and selling Bolt goods which he


stored in a bodega in Manila.
2. Yorkshire issued insurance for the goods stored in the bodega
3. Unfortunately, the bodega was razed by a fire
4. At the time of the fire, Go Lu claims that there were 66 cases of
bolt goods stored in the bodega
5. Yorkshire admit the issuance of the policies but contends that not
more than 16 cases were destroyed. It alleges that Go Lu submitted
fraudulent proof of the amount of loss.
ISSUE
What is the amount and value of the amount of goods which Go Lu
had in the bodega at the time of the fire?

HELD
(16 cases only and not 66cases)
The fire was seen by a large number of people, and if it be a
fact that the plaintiff had 66 cases of piece goods in the building at
the time, it was his duty to have offered the evidence of some
disinterested eyewitness as to the identity of the pieces or particles
remaining of the 50 cases, and of the physical facts, for the purpose
of showing that the 50 cases were in the bodega at the time of the
fire.
Although the original entries in plaintiff's books would be
evidence which should have some weight as to the amount of stock
which he had in March, and which he purchased during the months
of April, May, and June, and what he sold during that time, such
entries are of but little, if any, value as to the amount of goods
which he had in the bodega at the time of the fire. In any event,
they are not sufficient to overcome the absence of any evidence of
the physical facts existing after the fire, and the rule of reason that
the 50 cases of goods would not be consumed and completely
wiped out of existence, without leaving some evidence of their
destruction, which could be found among the remains and debris in
the building after the fire.
Here, the facts existing at and after the fire are conclusive
evidence that there were only 16 cases of goods in thebodega at
the time of the fire, and the majority of this court are of the opinion
that plaintiff's claim is not only fraudulent, but that he knew it was
fraudulent at the time it was made, and that, for such reason, he is
not entitled to recover anything.
Page 67 of 154

INSURANCE G01 CASE DIGESTS

108. FINMAN S GENERAL ASSURANCE V. COURT OF APPEALS, 361


SCRA 214 (2001)
FINMAN'S GENERAL ASSURANCE vs. COURT OF APPEALS and
USUPHIL
Topic: Notice and Proof of Loss
FACTS:
1. Usuphil obtained a fire insurance policy from Finman's
Assurance covering certain properties, e.g., office, furniture,
fixtures, shop machinery and other trade equipment, for
damages and loss arising from fire.
2. Usuphil filed a claim for the loss of the insured properties due
to fire. Finman's appointed Adjuster H.H. Bayne to undertake
the valuation and adjustment of the loss. H.H. Bayne then
required Usuphil to file a formal claim and submit proof of loss.
3.Usuphil submitted its Sworn Statement of Loss and Formal
Claim signed by its Manager. It likewise submitted Proof of Loss
signed by its Accounting Manager and countersigned by H.H.
Baynes Adjuster.
4. Despite repeated demands by private respondent, Finman's
refused to pay the insurance claim. Hence, Usuphil filed a
complaint against Finman's for the unpaid insurance claim.
5. In its Answer, Finman's maintained that the claim could not
be allowed because it failed to comply with Policy Condition No.
13 regarding the submission of certain documents to prove the
loss.
Policy Condition No. 13 reads:

13. The insured shall give immediate written notice to the


Company of any loss, protect the property from further
damage, forthwith separate the damaged and undamaged
personal property, put it in the best possible order, furnish a
complete inventory of the destroyed, damaged, and
undamaged property, showing in detail quantities, costs, actual
cash value and the amount of loss claimed; AND WITHIN SIXTY
DAYS AFTER THE LOSS, UNLESS SUCH TIME IS EXTENDED IN
WRITING BY THE COMPANY, THE INSURED SHALL RENDER TO
THE COMPANY A PROOF OF LOSS, signed and sworn to by the
insured, stating the knowledge and belief of the insured as to
the following: the time and origin of the loss, the interest of the
insured and of all others in the property, the actual cash value
of each item thereof and the amount of loss thereto, all
encumbrances thereon, all other contracts of insurance,
whether valid or not, covering any of said property, any changes
in the title, use, occupation, location, possession or exposures
of said property since the issuing of this policy by whom and for
what purpose any buildings herein described and the several
parts thereof were occupied at the time of loss and whether or
not it then stood on leased ground, and shall furnish a copy of
all the descriptions and schedules in all policies, and if required
verified plans and specifications of any building, fixtures, or
machinery destroyed or damaged. The insured, as often as may
be reasonably required, shall exhibit to any person designated
by the company all that remains of any property herein
described, and submit to examination under oath by any person
named by the Company, and subscribe the same; and, as often
as may be reasonably required, shall produce for examination
all books of account, bills, invoices, and other vouchers or
certified copies thereof if originals be lost, at such reasonable
time and place as may be designated by the Company or its
representative and shall permit extracts and copies thereof to
be made.
Page 68 of 154

INSURANCE G01 CASE DIGESTS

No claim under this policy shall be payable unless the terms of


this condition have been complied with.
6. RTC ruled in favor of Usuphil. CA affirmed. Hence this present
petition. Finman's alleged that complate diregard of the
required documents communicated by Bayne, through a letter
justified their failure to grant Usuphil's claim.
ISSUE:
WON disallowance of Usuphils claim is justified by its failure to
submit the required documents in accordance with Policy
Condition No. 13

that at a meeting between Usuphil and Finman's, the latter


summoned the said manager to reconcile the claims. One who
clothes another with apparent authority as his agent and holds
him to the public as such, cannot later be allowed to deny the
authority of such person to act as his agent when such third
person entered into the contract in good faith and in an honest
belief that he is such agent.

109.

PACIFIC TIMBER V. COURT OF APPEALS, 112 SCRA 199 (1982)

Topic: Notice and Proof of Loss

HELD: NO

Pacific Timber Export Corp vs Court of Appeals

RATIO DECIDENDI:

Facts:

According to the facts of this case, after the occurrence of the


fire, Usuphil immediately notified Finman's thereof. Thereafter,
it submitted the following documents: (1) Sworn Statement of
Loss and Formal Claim and; (2) Proof of Loss.
The submission of these documents constitutes substantial
compliance with the above provision. Indeed, as regards the
submission of documents to prove loss, substantial, not strict as
urged by petitioner, compliance with the requirements will
always be deemed sufficient.
In any case, Finman itself acknowledged its liability when
through its Finance Manager, it signed the document indicating
that the amount due private Usuphil is P842,683.40.
The contention that such manager was without authority to
bind the corporation is without merit. The evidence indicate

March 19, 1963 Pacific Timber Export Corp secured


temporary insurance from Workmens Insurance Company
for its exportation of 1,250,000 board feet of Philippine
lauan and Apitong logs to be shipped from Diapitan Bay,
Quezon Province to Okinawa and Tokyo, Japan
o Cover Note No. 1010 was issued insuring the said
cargo subject to the terms and conditions of the 2
marine cargo policies issued on April 2, 1963
March 29, 1963 after the issuance of the cover note but
before the issuance of the two marine policies, some of the
logs intended to be exported were lost during loading
operations in the Diapitan Bay.
In a letter dated April 4, 1963, Pacific Timber informed the
insurer about the loss of approximately 32 pieces of logs
On July 17, 1963, Insurer requested their adjuster to inspect
the loss and assess the damage. The latter reported that the
loss of 30 pieces of logs is not covered by the Policies.
Page 69 of 154

INSURANCE G01 CASE DIGESTS

The insurer denied the claim on the ground that the


investigation revealed that the entire shipment of logs
covered by 2 policies were received in good order at their
point of destination. Loss cannot be considered covered
under the Cover Note as it became null and void by virtue of
the issuance of the marine policies.

Issue: Whether or not the Insurance Company was correct in


denying the claim based on the Cover Note issued without
consideration?
Held: NO.
The Cover Note was not without consideration.
The fact that no separate premium was paid on the Cover Note
before the loss insured against occurred, does not militate against
the validity of petitioners contention for no such premium could
have been paid since by the nature of the Cover Note,, it did not
contain, as all Cover Notes do not contain particulars of the
shipment that would serve as basis for the computation of
premiums.
If the Note is to be treated as a separate policy instead of
integrating it to the regular policies subsequently issued, the
purpose and function of the Cove Note would be set at naught or
rendered meaningless.

therefore no cause for petitioner to lose what is due it as if there


had been payment of premium.
The defense of DELAY cannot be sustained.
The law requires this ground of delay to be promptly and specifically
asserted when a claim on the insurance agreement is made.
Instead of invoking the ground of delay in objecting to petitioners
claim of recovery on the Note, it took steps clearly indicative that
his particular ground for objection to the claim was never in its
mind.
The nature of the round places the insurer on duty to inquire when
the loss took place, so that it could determine whether delay would
be a valid ground.
From April 1963 to July 1963, enough time was available for Insurer
to determine if petitioner was guilty of delay in communicating the
loss to Insurer
Sec 84 of the Insurance Act provides:
Sec. 84 Delay in the presentation to an insurer of notice or
proof of loss is waived if caused by any act of his or if he omits to
take objection promptly and specifically upon that ground.
Waiver can successfully be raised against private respondent

It was not necessary to ask petitioner to pay premium on the Note,


for the loss insured against having already occurred, the more
practical procedure is simply to deduct the premium form the
amount due the petitioner on the Note. The non payment is
Page 70 of 154

INSURANCE G01 CASE DIGESTS

DOUBLE INSURANCE
110. STA. ANA V. COMMERCIAL UNION ASSURANCE , 55 PHIL. 329
(1930)
Facts
On the 1st of October, 1925, the plaintiff Ulpiano Santa Ana
took out a three-thousand-peso fire insurance policy on the house
in the Phoenix Assurance Company (Exhibit C), and six-thousandpeso policy in the Guardian Assurance Company, Limited (Exhibit D),
for a period of one year from that date until 4 o'clock in the
afternoon of October 1, 1926, paying the respective premiums of
P97.50 and P196 to said companies through their duly authorized
Philippine agent, Kerr & Company. (Exhibit C and D)
On November 19, 1925, the plaintiff Ulpiano Santa Ana
mortgaged said house to the plaintiff Rafael Garcia for P5,000, for a
period of two years, the contract being drawn up as a retro sale
(Exhibit A) for the sum of P5,000, and the policies issued by the
Phoenix Assurance Company and the Guardian Assurance Company,
Limited, were endorsed to the mortgagee, Rafael Garcia (Exhibits C,
D, and E). On December 16, 1925, the plaintiff Urpiano Santa Ana
reinsured said house with the defendant companies, the Globe and
Rutgers Fire Insurance Company of New York, and the Commercial
Union Assurance Company, Limited of London, through their
common agent duly authorized to represent them in the Philippine
Islands, the Pacific Commercial Company, for the amount of P3,000
each, paying the 90-peso premium due upon each policy, which was
to be effective for one year from the aforementioned date until 4
o'clock in the afternoon of December 16, 1926 (Exhibits B and B-1).

On September 20, 1926, Ulpiano Santa Ana took out another


insurance policy on the house in question for P6,000 in the
"Filipinas, Compania de Seguros, which issued the one-year policy
Exhibit E, upon receiving from said plaintiff the amount of P195 as
premium thereon.
Issue: W/N the notice to the other insurance companies of the fact
that ulpiano has procured other insurances are enough to not annul
the insurance in her favour.
Held: no.
It should be noted that clause three of the "Filipinas" policy drawn
up in Spanish, and the english policies issued by the four other
companies, provided that any outstanding insurance upon the
whole or a portion of the objects thereby assured must be declared
by the insured in writing and he must cause the company to add or
insert it in the policy, without which such policy shall be null and
avoid, and the insured will not be entitled to indemnity in case of
loss.lawphil.net
Ulpiano Santa Ana maintains that he gave the required notice
to all the insurance companies; To Kerr & Company through their
sub-agent, Mariano Morelos; to the Pacific Commercial Company
through their employee, Guillermo de Leon; and to the "Filipinas,
Compania de Seguros" through their agent, Juan Grey; telling them
he had paid for other insurance on the same property. But he has
been contradicted in this by all the persons mentioned, and this
deprives his allegations of probative force, especially considering
that such advises or notices, so basic and essential to the existence
and validity of the policies, must be given in writing as required in
the noted attached to the four policies above mentioned, and must
be given in writing as required in the note attached to the four
policies above mentioned, and must be endorsed upon each of
them, so that in case of necessity, as in the instant one, when a loss
Page 71 of 154

INSURANCE G01 CASE DIGESTS


occurs, the insured may clearly show that he has fulfilled this
indispensable requisite, since all companies, to which people apply
for insurance upon property already assured, have an interest in
knowing what other policies issued by other companies the insured
already holds, for the purpose of knowing just what interest the
applicant has in the preservation of the property, and the care and
precaution to be taken for the prevention of loss.
Without deciding whether notice of other insurance upon the same
property must be given in writing, or whether a verbal notice is
sufficient to render an insurance valid which requires such notice,
whether oral or written , we hold that in the absolute absence of
such notice when it is one of the conditions specified in the fire
insurance policy, the policy is null and void.

111. UNION MANUFACTURING V. PHI GUARANTY, 47 SCRA 271


(1972)

UNION MANUFACTURING CO., INC. VS. PHILIPPINE GUARANTY


CO., INC.
47 SCRA 271 (G.R. NO. L-27932)
OCTOBER 30, 1972
DOUBLE INSURANCE
Petitioner: Republic Bank
Respondent: Philippine Guaranty Co.. Inc.
J. Fernando:
FACTS:
On January 12, 1962, the Union Manufacturing Co., Inc. obtained
certain loans from the Republic Bank in the total sum of
415,000.00. To secure the payment thereof, UMC executed real and
chattel mortgage on certain properties.
The Republic Bank procured from the defendant Philippine
Guaranty Co., Inc. an insurance coverage on loss against fire for
500,000.00 over the properties of the UMC, as described in
defendants cover note dated September 25, 1962, with the
annotation that loss or damage, if any, under said cover note is
payable to Republic Bank as its interest may appear, subject
however to the printed conditions of said defendants Fire
Insurance Policy Form.
On September 6, 1964, a fire occurred in the premises of UMC and
on October 6, 1964, UMC filed its fire claim with the PGC Inc., thru
its adjuster, H.H. Bayne Adjustment Co., which was denied by said
defendant in its letter dated November 26, 1964 on the following
ground: Policy Condition No. 3 and/or the Other Insurance Clause
Page 72 of 154

INSURANCE G01 CASE DIGESTS


of the policy was violated because you did not give notice to us of
the other insurance which you had taken from New India for
80,000.00. Sincere Insurance for 25,000.00 and Manila Insurance
for 200,000.00 with the result that these insurances of which we
became aware of only after the fire, were not endorsed on our
policy.

The insurance contract may be rather onerous, but that in itself


does not justify the abrogation of its express terms, terms which the
insured accepted or adhered to and which is the law between the
contracting parties.
**lifted digest

ISSUE: Whether Republic Bank can recover.


HELD: Without deciding- whether notice of other insurance upon
the same property must be given in writing, or whether a verbal
notice is sufficient to render an insurance valid which requires such
notice, whether oral or written, we hold that in the absolute
absence of such notice when it is one of the conditions specified in
the fire insurance policy, the policy is null and void. (Santa Ana vs.
Commercial Union Ass. Co., 55 Phil. 128).

112. PIONEER INSURANCE V. YAP, 61 SCRA 426 (1974)


INSURANCE: Double Insurance
PIONEER INSURANCE and SURETY CORPORATION vs OLIVA YAP
(December 19, 1974)
FACTS:

If the insured has violated or failed to perform the conditions of the


contract, and such a violation or want of performance has not been
waived by the insurer, then the insured cannot recover. Courts are
not permitted to make contracts for the parties. The functions and
duty of the courts consist simply in enforcing and carrying out the
contracts actually made.
While it is true, as a general rule, that contracts of insurance are
construed most favorably to the insured, yet contracts of insurance,
like other contracts, are to be construed according to the sense and
meaning of the terms which the parties themselves have used. If
such terms are clear and unambiguous they must be taken and
understood in their plain, ordinary and popular sense.
The annotation then, must be deemed to be a warranty that the
property was not insured by any other policy. Violation thereof
entitles the insurer to rescind. xxx The materiality of non disclosure
of other insurance policies is not open to doubt.

Oliva Yap was the owner of a store in a 2-storey building in


Manila where she sells bags and footwear. Chua Soon Poon,
the son-in-law was in charge of the store.
Yap procured a fire insurance policy from Pioneer Insurance
and Surety Corp. (25,000 face value) on her stocks and
office furniture. One of the conditions in the policy is that
she shall notify the insurer of any other insurance already
effected or subsequently procured, and state or endorse it
in the policy. Otherwise the Policy shall be forfeited.
There was another insurance from Great American
Insurance Company which was noted on Pioneers policy
and the parties endorsed Great Americans policy.
Yap took out another fire insurance policy from Federal
Insurance Company but without notice to and consent of
Pioneer.

Page 73 of 154

INSURANCE G01 CASE DIGESTS

A fire broke out in the building and the store was burned.
Yap filed an insurance claim but Pioneer refused saying
theres a violation of terms and conditions.
CFI and CA: In favour of Oliva Yap

ISSUE: WON Pioneer should be absolved from liability on account of


Yaps violation of co-insurance clause.
Yap argues that Federal merely substituted Great American.
HELD: YES. Yap violated it.

The insurance policy issued by Great American Insurance


Company duly noted on Pioneers Policy co-insurance,
ceased, by agreement of the parties to be recognized by
them as a co-insurance policy. The Court of Appeals says
that Great American policy was substituted by the Federal
Insurance policy for the same amount, and because it was a
mere case of substitution, there was no necessity for its
endorsement on Pioneer policy. There is no evidence to
establish and prove such a substitution. If anything was
substituted for Great American policy, it could only be the
Northwest Insurance policy for the same amount of
P20,000.00. The endorsement shows the clear intention of
the parties to recognize on the date the endorsement was
made (August 29, 1962), the existence of only one coinsurance, and that is the Northwest Insurance policy, which
according to the stipulation of the parties during the
hearing, was issued on August 20, 1962 and endorsed only
on August 20, 1962. The finding that Great American policy
was substituted by the Federal Insurance policy is
unsubstantiated.
By the plain terms of the policy, other insurance without the
consent of Pioneer would ipso facto avoid the contract. It
required no affirmative act of election on the part of the

company to make operative the clause avoiding the


contract, wherever the specified conditions should occur. Its
obligations ceased, unless, being informed of the fact, it
consented to the additional insurance. The obvious purpose
of the aforesaid requirement in the policy is to prevent
over-insurance and thus avert the perpetration of fraud.
The public, as well as the insurer, is interested in preventing
the situation in which a fire would be profitable to the
insured. According to Justice Story: "The insured has no
right to complain, for he assents to comply with all the
stipulation on his side, in order to entitle himself to the
benefit of the contract, which, upon reason or principle, he
has no right to ask the court to dispense with the
performance of his own part of the agreement, and yet to
bind the other party to obligations, which, but for those
stipulation would not have been entered into."

Page 74 of 154

INSURANCE G01 CASE DIGESTS


113. GEAGONIA V. COURT OF APPEALS, 241 SCRA 152 (1995)
Geagonia vs CA
GR 114427, 241 SCRA 152; February 6, 1995
Topic: Double Insurance: Payment of claim where there is overinsurance by double insurance
Facts:
Petitioner owned Normans Mart in Agusan del Sur. In
December 1989, he filed insurance policy (fire insurance) from
respondent for Php100,000, valid for one year, covering Stock-intrade consisting principally of dry goods such as RTW's for men and
women wear and other usual to assured's business." Under the
policy, he noted Mercantile Insurance as co-insurer for Php50,000.
The policy contained the condition (Condition 3) that the insured
shall notify the company of any insurance/s already affected or
subsequently effected covering the same properties. The stipulation
further states that unless notice be given and the particulars of such
insurance or insurances be stated therein or endorsed in this policy
pursuant to Section 50 of the Insurance Code, all benefits under this
policy shall be deemed forfeited, provided however, that this
condition shall not apply when the total insurance or insurances in
force at the time of the loss or damage is not more than
P200,000.00. His goods totaled a valued of Php392,130. The market
burned in May 1990 and the petitioners stocks were completely
destroyed. He filed to claim the insurance proceeds from the
respondent, but was denied because the same goods were also
covered by a third insurer (Philippines First Insurance) not disclosed
with the respondent. The policies with the third insurer indicate
petitioner as insured and show that there was even a mortgage
attached over the properties with a loss payable clause to the
mortgagee.
Petitioner filed with the Insurance Commission for recovery
of the Php100,000 and attorneys fees. He also claims that the value
of the goods actually valued at Php1,000,000 at time of loss.

Petition was denied. Upon reconsideration, he attached a document


Annex A admitting that at the time he obtained the policy from
the respondent, the policies by Philippine First Insurance were
already existing. He claims though that nothing in the respondents
policy required him to disclose other subsisting policies and that the
respondents agent failed to inform him of this, if there was any.
Respondent denied the allegations and claimed he is not entitled to
the proceeds for violating the condition.
The insurance commission reversed and stated the
petitioner had no knowledge that its creditor Cebu Tesing obtained
the policy from Philippine First Insurance over the goods, and even
benefitted from it upon the fire. Respondents motion for
reconsideration was denied, thus it appealed to the CA. The CA
reversed the insurance commission and found the petitioner knew
of the existence of the third policy because it was in his name and
he even paid the premiums.
Issue:
1. Did the petitioner actually have knowledge of the third
policys existence thus violating condition 3 of the policy
with the respondent?
2. If he had, may he still recover the proceeds?
Held:
1. YES
2. NO
Rationale:
We agree with the Court of Appeals that the petitioner knew of the
prior policies issued by the PFIC. His letter of 18 January 1991 to the
private respondent conclusively proves this knowledge. His
testimony to the contrary before the Insurance Commissioner and
which the latter relied upon cannot prevail over a written admission
madeante litem motam. It was, indeed, incredible that he did not
know about the prior policies since these policies were not new or
Page 75 of 154

INSURANCE G01 CASE DIGESTS


original. Policy No. GA-28144 was a renewal of Policy No. F-24758,
while Policy No. GA-28146 had been renewed twice, the previous
policy being F-24792.
Condition 3 of the private respondent's Policy No. F-14622
is a condition which is not proscribed by law. Its incorporation in the
policy is allowed by Section 75 of the Insurance Code which
provides that "[a] policy may declare that a violation of specified
provisions thereof shall avoid it, otherwise the breach of an
immaterial provision does not avoid the policy." Such a condition is
a provision which invariably appears in fire insurance policies and is
intended to prevent an increase in the moral hazard. It is commonly
known as the additional or "other insurance" clause and has been
upheld as valid and as a warranty that no other insurance exists. Its
violation would thus avoid the policy. However, in order to
constitute a violation, the other insurance must be upon same
subject matter, the same interest therein, and the same risk.
It must be underscored that unlike the "other insurance"
clauses involved in General Insurance and Surety Corp. vs. Ng Hua or
in Pioneer Insurance & Surety Corp. vs. Yap, Condition 3 in the
private respondent's policy No. F-14622 does not absolutely declare
void any violation thereof. It expressly provides that the condition
"shall not apply when the total insurance or insurances in force at
the time of the loss or damage is not more than P200,000.00."
It is a cardinal rule on insurance that a policy or insurance
contract is to be interpreted liberally in favor of the insured and
strictly against the company, the reason being, undoubtedly, to
afford the greatest protection which the insured was endeavoring
to secure when he applied for insurance. It is also a cardinal
principle of law that forfeitures are not favored and that any
construction which would result in the forfeiture of the policy
benefits for the person claiming, will be avoided, if it is possible to
construe the policy in a manner which would permit recovery, as,
for example, by finding a waiver for such forfeiture. Stated
differently, provisions, conditions or exceptions in policies which
tend to work a forfeiture of insurance policies should be construed

most strictly against those for whose benefits they are inserted, and
most favorably toward those against whom they are intended to
operate. The reason for this is that, except for riders which may
later be inserted, the insured sees the contract already in its final
form and has had no voice in the selection or arrangement of the
words employed therein. On the other hand, the language of the
contract was carefully chosen and deliberated upon by experts and
legal advisers who had acted exclusively in the interest of the
insurers and the technical language employed therein is rarely
understood by ordinary laymen.
With these principles in mind, we are of the opinion that
Condition 3 of the subject policy is not totally free from ambiguity
and must, perforce, be meticulously analyzed. Such analysis leads us
to conclude that (a) the prohibition applies only to double
insurance, and (b) the nullity of the policy shall only be to the extent
exceeding P200,000.00 of the total policies obtained.
The first conclusion is supported by the portion of the
condition referring to other insurance "covering any of the property
or properties consisting of stocks in trade, goods in process and/or
inventories only hereby insured," and the portion regarding the
insured's declaration on the subheading CO-INSURANCE that the coinsurer is Mercantile Insurance Co., Inc. in the sum of P50,000.00. A
double insurance exists where the same person is insured by
several insurers separately in respect of the same subject and
interest. As earlier stated, the insurable interests of a mortgagor
and a mortgagee on the mortgaged property are distinct and
separate. Since the two policies of the PFIC do not cover the same
interest as that covered by the policy of the private respondent, no
double insurance exists. The non-disclosure then of the former
policies was not fatal to the petitioner's right to recover on the
private respondent's policy.
Furthermore, by stating within Condition 3 itself that such
condition shall not apply if the total insurance in force at the time of
loss does not exceed P200,000.00, the private respondent was
amenable to assume a co-insurer's liability up to a loss not
Page 76 of 154

INSURANCE G01 CASE DIGESTS


exceeding P200,000.00. What it had in mind was to discourage
over-insurance. Indeed, the rationale behind the incorporation of
"other insurance" clause in fire policies is to prevent over-insurance
and thus avert the perpetration of fraud. When a property owner
obtains insurance policies from two or more insurers in a total
amount that exceeds the property's value, the insured may have an
inducement to destroy the property for the purpose of collecting
the insurance. The public as well as the insurer is interested in
preventing a situation in which a fire would be profitable to the
insured.

114. PACIFIC BANKING CORP. V. COURT OF APPEALS, 168 SCRA 1


(1968)
Paras, J.
Facts
1. An open fire policy was issued to Paramount Shirt by
insurer, Oriental Assurance.
a. Oriental bound itself to indemnify the insured for
any loss or damage, not exceeding P61,000.00,
caused by fire to its property consisting of stocks,
materials and supplies usual to a shirt factory from
Oct. 21, 1965 to Oct. 21, 1964
2. Paramount Shirt owed money from Pacific Banking in the
amount of P800,000. As security, the goods insured was
held by Pacific Banking in trust.
3. Said policy was duly endorsed to petitioner as mortgagee/
trustor of the properties insured, with the knowledge and
consent of Oriental to the effect that "loss if any under this
policy is payable to the Pacific Banking Corporation".

4. In Jan. 1964, a fire broke out destroying the subject


property.
5. Pacific Banking demanded payment from Oriental.
a. Pacific did not present any proof of loss.
6. Oriental did not accede to the demand because it was
waiting for the insurance adjuster report.
7. The insurance adjust notified Pacific Banking that
Paramount had not filed any claim with Oriental yet, nor
submitted proof of loss which is a clear violation of Policy
Condition No.11, and for which reason, determination of
the liability of Oriental could not be had.
a. Pacific Banking answered the insurance adjuster
saying that it can get a proof of loss by verifying
from the records of the Bureau of Customs the
entries of merchandise taken into the customs
bonded warehouse razed by fire.
8. But still, Oriental refused to pay.
9. Pacific Banking filed an action for sum of money against
Oriental P61,000.
10. Oriental raised the following defenses:
a. lack of formal claim by insured over the loss and
b. premature filing of the suit as neither plaintiff nor
insured had submitted any proof of loss on the basis
of which defendant would determine its liability and
the amount thereof
11. During the trial, Pacific presented in evidence a document
revealing undeclared co-insurances with 6 other insurance
companies taken by Paramount covering the same goods
destroyed.
a. It will be noted that the defense of fraud and/or
violation of Condition No. 3 in the Policy, in the
Page 77 of 154

INSURANCE G01 CASE DIGESTS


form of non-declaration of co-insurances which was
not pleaded in the answer was also not pleaded in
the Motion to Dismiss.
12. RTC ruled in favor of Pacific Banking. Oriental should pay.
13. CA reversed the decision.
14. Hence, this petition.

Issues
1. Whether or not the non-declaration of co-insurances
violates the policy condition which required the insured to
reveal other insurances already effected.
a. Yes.
2. Whether or not failure of the insured to file the required
proof of loss prior to court action.
a. Yes.

Ruling
First Issue

As the insurance policy against fire expressly required that


notice should be given by the insured of other insurance
upon the same property, the total absence of such notice
nullifies the policy.
It is not disputed that the insured failed to reveal before the
loss three other insurances. As found by the Court of
Appeals, by reason of said unrevealed insurances, the
insured had been guilty of a false declaration; a clear
misrepresentation and a vital one because where the
insured had been asked to reveal but did not, that was

deception. Otherwise stated, had the insurer known that


there were many co-insurances, it could have hesitated or
plainly desisted from entering into such contract. Hence,
the insured was guilty of clear fraud (Rollo, p. 25).
Petitioner's contention that the allegation of fraud is but a
mere inference or suspicion is untenable. In fact, concrete
evidence of fraud or false declaration by the insured was
furnished by the petitioner itself when the facts alleged in
the policy under clauses "Co-Insurances Declared" and
"Other Insurance Clause" are materially different from the
actual number of co-insurances taken over the subject
property.
On the question of Pacific Bankings right to the proceeds
as the mortgagee, the SC held that:
o It is but fair and just that where the insured who is
primarily entitled to receive the proceeds of the
policy has by its fraud and/or misrepresentation,
forfeited said right, with more reason Pacific which
is merely claiming as indorsee of said insured,
cannot be entitled to such proceeds.
o It will be noted that the fact of fraud was tried by
express or at least implied consent of the parties.
Pacific did not only object to the introduction of
evidence but on the contrary, presented the very
evidence that proved its existence.

Second Issue

Generally, the cause of action on the policy accrues when


the loss occurs, But when the policy provides that no action
shall be brought unless the claim is first presented
Page 78 of 154

INSURANCE G01 CASE DIGESTS

extrajudicially in the manner provided in the policy, the


cause of action will accrue from the time the insurer finally
rejects the claim for payment (Eagle Star Insurance v. Chia
Yu, 55 Phil 701 [1955]).
In the case at bar, policy condition No. 11 specifically
provides that the insured shall on the happening of any loss
or damage give notice to the company and shall within
fifteen (15) days after such loss or damage deliver to the
private respondent (a) a claim in writing giving particular
account as to the articles or goods destroyed and the
amount of the loss or damage and (b) particulars of all other
insurances, if any. Likewise, insured was required "at his
own expense to produce, procure and give to the company
all such further particulars, plans, specifications, books,
vouchers, invoices, duplicates or copies thereof, documents,
proofs and information with respect to the claim". (Record
on Appeal, pp. 18-20).
The evidence adduced shows that twenty-four (24) days
after the fire, petitioner merely wrote letters to private
respondent to serve as a notice of loss, thereafter, the
former did not furnish the latter whatever pertinent
documents were necessary to prove and estimate its loss.
Instead, petitioner shifted upon private respondent the
burden of fishing out the necessary information to ascertain
the particular account of the articles destroyed by fire as
well as the amount of loss. It is noteworthy that private
respondent and its adjuster notified petitioner that insured
had not yet filed a written claim nor submitted the
supporting documents in compliance with the requirements
set forth in the policy. Despite the notice, the latter
remained unheedful. Since the required claim by insured,

together with the preliminary submittal of relevant


documents had not been complied with, it follows that
private respondent could not be deemed to have finally
rejected petitioner's claim and therefore the latter's cause
of action had not yet arisen.
SC affirms CA. Petition dismissed.

Page 79 of 154

INSURANCE G01 CASE DIGESTS

REINSURANCE
115. PIONEER INSURANCE V. COURT OF APPEALS, 175 SCRA 668
(1989)
PIONEER INSURANCE & SURETY CORP V CA, BORMAHECO,
MAGLANA AND LIM G.R. No. 84197

LIM V CA, PIONEER INSURANCE AND SURETY CORP, BORMAHECO,


CERVANTES AND MAGLANA
FACTS:
2 consolidated petitions
1965 Jacob S. Lim was the owner-operator of Southern Air
Lines (SAL) a single proprietorship
May 17, 1985 at Tokyo, Japan, Japan Domestic Airlines (JDA)
and Lim entered into and executed a sales contract for the
sale and purchase of and purchase of two (2) DC-3A Type
aircrafts and one (1) set of necessary spare parts for the
total agreed price of US $109,000.00 to be paid in
installments. One DC-3 Aircraft with Registry No. PIC-718,
arrived in Manila on June 7,1965 while the other aircraft,
arrived in Manila on July 18,1965.
On May 22, 1965, Pioneer Insurance and Surety Corporation
(Pioneer, petitioner in G.R. No. 84197) as surety executed
and issued its Surety Bond No. 6639 (Exhibit C) in favor of
JDA, in behalf of its principal, Lim, for the balance price of
the aircrafts and spare parts.
It appears that Border Machinery and Heavy Equipment
Company, Inc. (Bormaheco), Francisco and Modesto
Cervantes (Cervanteses) and Constancio Maglana
(respondents in both petitions) contributed some funds
used in the purchase of the above aircrafts and spare parts.

The funds were supposed to be their contributions to a new


corporation proposed by Lim to expand his airline business.
They executed two (2) separate indemnity agreements
(Exhibits D-1 and D-2) in favor of Pioneer, one signed by
Maglana and the other jointly signed by Lim for SAL,
Bormaheco and the Cervanteses.
indemnity agreements stipulated that the indemnitors
principally agree and bind themselves jointly and severally
to indemnify and hold and save harmless Pioneer from and
against any/all damages, losses, costs, damages, taxes,
penalties, charges and expenses of whatever kind and
nature which Pioneer may incur in consequence of having
become surety
June 10, 1965 - Lim doing business under the name and
style of SAL executed in favor of Pioneer as deed of chattel
mortgage as security for the latter's suretyship in favor of
the former. It was stipulated therein that Lim transfer and
convey to the surety the two aircrafts. The deed (Exhibit D)
was duly registered with the Office of the Register of Deeds
of the City of Manila and with the Civil Aeronautics
Administration pursuant to the Chattel Mortgage Law and
the Civil Aeronautics Law (Republic Act No. 776),
respectively.
Lim defaulted on his subsequent installment payments
prompting JDA to request payments from the surety.
Pioneer paid a total sum of P298,626.12.
Pioneer then filed a petition for the extrajudicial foreclosure
of the said chattel mortgage before the Sheriff of Davao
City. The Cervanteses and Maglana, however, filed a third
party claim alleging that they are co-owners of the aircrafts,
On July 19, 1966, Pioneer filed an action for judicial
foreclosure with an application for a writ of preliminary
Page 80 of 154

INSURANCE G01 CASE DIGESTS


attachment against Lim and respondents, the Cervanteses,
Bormaheco and Maglana.
ISSUES/HELD/RATIO:
G.R. No. 84197-relevant discussion
1. Has Pioneer a cause of action against defendants with respect to
so much of its obligations to JDA as has been paid with reinsurance
money? NO
2. If the answer to the preceding question is in the negative, has
Pioneer still any claim against defendants, considering the amount it
has realized from the sale of the mortgaged properties? NO
The total amount paid by Pioneer to JDA is P299,666.29. Since
Pioneer has collected P295,000.00 from the reinsurers, the
uninsured portion of what it paid to JDA is the difference between
the two amounts, or P3,666.28. This is the amount for which
Pioneer may sue defendants, assuming that the indemnity
agreement is still valid and effective. But since the amount realized
from the sale of the mortgaged chattels are P35,000.00 for one of
the airplanes and P2,050.00 for a spare engine, or a total of
P37,050.00, Pioneer is still overpaid by P33,383.72. Therefore,
Pioneer has no more claim against defendants.
The payment to the petitioner made by the reinsurers was not
disputed in the appellate court. Considering this admitted payment,
the only issue that cropped up was the effect of payment made by
the reinsurers to the petitioner. Therefore, the petitioner's
argument that the respondents had no interest in the reinsurance
contract as this is strictly between the petitioner as insured and the
reinsuring company pursuant to Section 91 (should be Section 98) of
the Insurance Code has no basis.

In general a reinsurer, on payment of a loss


acquires the same rights by subrogation as are
acquired in similar cases where the original insurer
pays a loss (Universal Ins. Co. v. Old Time Molasses
Co. C.C.A. La., 46 F 2nd 925).
The rules of practice in actions on original
insurance policies are in general applicable to
actions or contracts of reinsurance. (Delaware, Ins.
Co. v. Pennsylvania Fire Ins. Co., 55 S.E. 330,126 GA.
380, 7 Ann. Con. 1134).
Hence the applicable law is Article 2207 of the new Civil Code, to
wit:
Art. 2207. If the plaintiffs property has been
insured, and he has received indemnity from the
insurance company for the injury or loss arising out
of the wrong or breach of contract complained of,
the insurance company shall be subrogated to the
rights of the insured against the wrongdoer or the
person who has violated the contract. If the amount
paid by the insurance company does not fully cover
the injury or loss, the aggrieved party shall be
entitled to recover the deficiency from the person
causing the loss or injury.
Interpreting the aforesaid provision, we ruled in the case of Phil. Air
Lines, Inc. v. Heald Lumber Co. (101 Phil. 1031 [1957]) which we
subsequently applied in Manila Mahogany Manufacturing
Corporation v. Court of Appeals (154 SCRA 650 [1987]):
Note that if a property is insured and the owner
receives the indemnity from the insurer, it is
provided in said article that the insurer is deemed
Page 81 of 154

INSURANCE G01 CASE DIGESTS


subrogated to the rights of the insured against the
wrongdoer and if the amount paid by the insurer
does not fully cover the loss, then the aggrieved
party is the one entitled to recover the deficiency.
Evidently, under this legal provision, the real party in
interest with regard to the portion of the indemnity
paid is the insurer and not the insured. (Emphasis
supplied).
It is clear from the records that Pioneer sued in its own name and
not as an attorney-in-fact of the reinsurer. Moreover, the
indemnity agreement ceased to be valid and effective after the
execution of the chattel mortgage. This is judicial admission and
aside from the chattel mortgage there is no other security for the
claim sought to be enforced by this action, which necessarily means
that the indemnity agreement had ceased to have any force and
effect at the time this action was instituted. Sec 2, Rule 129, Revised
Rules of Court.
The indemnity agreement was ipso jure extinguished upon the
foreclosure of the chattel mortgage. These defendants, as
indemnitors, would be entitled to be subrogated to the right of
Pioneer should they make payments to the latter. Articles 2067 and
2080 of the New Civil Code of the Philippines.
Other bases/issues:
Independently of the preceding proposition Pioneer's election of
the remedy of foreclosure precludes any further action to recover
any unpaid balance of the price. SAL or Lim, having failed to pay the
second to the eight and last installments to JDA and Pioneer as
surety having made of the payments to JDA, the alternative
remedies open to Pioneer were as provided in Article 1484 of the
New Civil Code, known as the Recto Law (=p). Pioneer exercised the

remedy of foreclosure of the chattel mortgage both by


extrajudicial foreclosure and the instant suit.
Also, Pioneer's liability as surety to JDA had already prescribed
when Pioneer paid the same. Consequently, Pioneer has no more
cause of action to recover from these defendants, as supposed
indemnitors, what it has paid to JDA. By virtue of an express
stipulation in the surety bond, the failure of JDA to present its claim
to Pioneer within ten days from default of Lim or SAL on every
installment, released Pioneer from liability from the claim.
Therefore, Pioneer is not entitled to exact reimbursement from
these defendants thru the indemnity. (Art 1318)
Petition in G.R. No. 84197 is not meritorious.
G.R. No. 84157
1. What legal rules govern the relationship among co-investors
whose agreement was to do business through the corporate vehicle
but who failed to incorporate the entity in which they had chosen to
invest?
2. How are the losses to be treated in situations where their
contributions to the intended 'corporation' were invested not
through the corporate form?
Principles governing: While it has been held that as between
themselves the rights of the stockholders in a defectively
incorporated association should be governed by the supposed
charter and the laws of the state relating thereto and not by the
rules governing partners, it is ordinarily held that persons who
attempt, but fail, to form a corporation and who carry on business
under the corporate name occupy the position of partners inter.
Thus, where persons associate themselves together under articles
to purchase property to carry on a business, and their organization
Page 82 of 154

INSURANCE G01 CASE DIGESTS


is so defective as to come short of creating a corporation within the
statute, they become in legal effect partners inter se, and their
rights as members of the company to the property acquired by the
company will be recognized.
Petitioner denied having received any amount from respondents
Bormaheco, the Cervanteses and Maglana. The trial court and the
appellate court, however, found through Exhibit 58, that the
petitioner received the amount of P151,000.00 representing the
participation of Bormaheco and Atty. Constancio B. Maglana in the
ownership of the subject airplanes and spare parts. The record
shows that defendant Maglana gave P75,000.00 to petitioner Jacob
Lim thru the Cervanteses.
It is therefore clear that the petitioner never had the intention to
form a corporation with the respondents despite his
representations to them. This gives credence to the cross-claims of
the respondents to the effect that they were induced and lured by
the petitioner to make contributions to a proposed corporation
which was never formed because the petitioner reneged on their
agreement.
Applying therefore the principles of law earlier cited to the facts of
the case, necessarily, no de facto partnership was created among
the parties which would entitle the petitioner to a reimbursement
of the supposed losses of the proposed corporation. The record
shows that the petitioner was acting on his own and not in behalf of
his other would-be incorporators in transacting the sale of the
airplanes and spare parts.
DISPOSITIVE: petitions are DISMISSED. Decision of CA AFFIRMED.

116.

GIBSON V. REVILLA, 92 SCRA 219 (1979)

GIBSON v. REVILLA and LEPANTO CONSOLIDATED MINING


COMPANY
FACTS:
Lepanto Consolidated Mining Company filed a complaint against
Malayan Insurance Company, Inc. The civil suit thus instituted
by Lepanto against Malayan was founded on the fact that
Malayan issued a Marine Open Policy covering all shipments of
copper, gold, and silver concentrates in bulk from Poro, San
Fernando, La Union to Tacoma, Washington or to other places
in the United States. Thereafter, Malayan obtained reinsurance
abroad through Sedgwick, Collins & Co., Limited, a London
insurance brokerage. The Memorandum of Insurance issued by
Sedgwick to Malayan listed three groups of underwriters or
reinsurers Lloyds 62.808%, Companies (I.L.U.) 34.705%, Other
companies 2.487%. At the top of the list of underwriting
members of Lloyds is Syndicate No. 448, assuming 2.48% of the
risk assumed by the reinsurer, which syndicate number
petitioner Ivor Robert Dayton Gibson claims to be himself.
Petitioner then filed a motion to intervene as defendant, which
motion was denied by the lower court.
ISSUE: WHETHER OR NOT THE LOWER COURT COMMITTED,
REVERSIBLE ERROR IN REFUSING THE INTERVENTION OF THE
PETITIONER IN THE SUIT BETWEEN LEPANTO AND MALAYAN
COMPANIES.
HELD:
No. The respondent Judge committed no error of law in denying
petitioners Motion to Intervene and neither has he abused his
discretion in his denial of petitioners Motion for Intervention.
We agree with the holding of the respondent court that since
Page 83 of 154

INSURANCE G01 CASE DIGESTS


movant Ivor Robert Dayton Gibson appears to be only one of
several re-insurers of the risks and liabilities assumed by
Malayan Insurance Company, Inc., it is highly probable that
other re-insurers may likewise intervene. If petitioner is allowed
to intervene, We hold that there is good and sufficient basis for
the Court a quo to declare that the trial between Lepanto and
Malayan would be definitely disrupted and would certainly
unduly delay the proceedings between the parties especially at
the stage where Lepanto had already rested its case and that
the issue would also be compounded as more parties and more
matters will have to be litigated. In other words, the Courts
discretion is justified and reasonable.
We also hold that respondent Judge committed no reversible
error in further sustaining the fourth ground of Lepantos
Opposition to the Motion to Intervene that the rights, if any, of
petitioner are not prejudiced by the present suit and will be
fully protected in a separate action against him and his coinsurers by Malayan. Petitioners contention that he has to pay
once Malayan is finally adjudged to pay Lepanto because of the
very nature of a contract of reinsurance and considering that
the re-insurer is obliged to pay as may be paid thereon
(referring to the original policies), although this is subject to
other stipulations and conditions of the reinsurance contract, is
without merit.
The general rule in the law of reinsurance is that there-insurer is
entitled to avail itself of every defense which the re-insured
(which is Malayan)might urge in an action by the person
originally insured (which is Lepanto). As to the effect of the
clause to pay as may be paid thereon contained in petitioners
re-insurance contract, Arnould, on the Law of Marine Insurance
and Average, 13thEd., Vol. 1, Section 327, p. 315, states the
rule, this: It has been decided that this clause does not
preclude the reinsurer from insisting upon proper proof that a

loss strictly within the terms of the original policy has taken
place. This clause does not enable the original underwriter to
recover from his reinsurer to an extent beyond the subscription
of the latter. Wherefore, in view of the foregoing, the petition
is hereby dismissed. No costs.

117. ARTEX DEVELOPMENT V. WELLINGTON INSURANCE , 51 SCRA


352 (1973)

ARTEX DEVELOPMENT CO INC v. WELLINGTON INSURANCE CO INC


51 SCRA 352
TEEHANKEE; June 27, 1973
FACTS
- Wellington Insurance Co. Inc. insured for P24,346,509.00 the
buildings, stocks and machinery of plaintiff Artex Development Co.
Inc. against loss or damage by fire or lighting upon payment of the
plaintiff of the corresponding premiums; that said properties were
insured for an additional sum of P883,034.00; that defendant
insured plaintiff against business interruption (use and occupancy)
for P5,200,000.00;
Wellington entered into a contract of
reinsurance with Alexander and Alexander, Inc. of New York. USA.
- The buildings, stocks and machineries of plaintiffs spinning
department were burned.
- Notice of the loss and damage was given the defendant; that as
per report of the adjusters, the total property loss of the plaintiff
was the sum of P10,106,554.40 and the total business interruption
loss was P3,000,000.00;
Page 84 of 154

INSURANCE G01 CASE DIGESTS


- That defendant has paid to the plaintiff the sum of P6,481,870.07
of the property loss suffered by plaintiff and P1,864,134.08 on its
business interruption loss, leaving a balance of P3,624,683.43 and
P1,748,460.00, respectively.

benefit or favor to the insured, the insured, not being privy to the
reinsurance contract, has no cause of action against the reinsurer. It
is expressly provided in Section 91 the Insurance Act 1 that "(T)he
original insured has no interest in a contract of insurance."

- The counsel for Artex filed a Manifestation saying that in view of


the Deeds of Discharge and Collateral Agreement, the only
remaining liability subject of litigation shall be the proportion of the
loss reinsured with or through Alexander and Alexander, Inc. of New
York, USA, namely, P397,813.00.
- The document recited further that Artex acknowledges receipt of
the sum of P3.6M paid by the insurer in full and final settlement of
all or any claims of Artex against its insurer. It discharges its insurer
from all actions, proceedings, claims, demands, costs and expenses
in respect thereof.
- With regard the balance unpaid, Wellington contends that Artex
should have been directed against the reinsurers to cover the
liability and not against Wellington.

ISSUE
WON the insured (Artex) has a cause of action against the reinsurer
HELD
NO
- Unless there is a specific grant in, or assignment of, the
reinsurance contract in favor of the insured or a manifest intention
of the contracting parties to the reinsurance contract to grant such
Page 85 of 154

INSURANCE G01 CASE DIGESTS

MARINE INSURANCE
118. ROQUE V. INTERMEDIATE APPELLATE COURT, 139 SCRA 596
(1985)
ROQUE V. INTERMEDIATE APPELLATE COURT, 139 SCRA 596 (1985)
Petitioner: Isabela Roque, doing busines under the name and style
of Isabela Roque Timber Enterprises and Ong Chiong
Respondent: Intermediate Appelate Court and Pioneer Insurance
And Surety Corporation

FACTS:
Manila Bay Lighterage Corporation (Manila Bay) a common carrier,
entered into a contract with petitioners whereby the former would
load and cary on board its barge Marble 10 about 422.18 cubic
meters of logs from Malampaya Sound, Palawan to North Harbor
Manila. The petitioners insured the logs against loss for
100,000.00 with respondent Pioneer Insurance and Surety
Corporation (Pioneer).
The petitioner loaded on the barge, 811 pieces of logs at
Malampaya Sound, Palawan for carriage and delivery to North
Harbor, Port of Manila, but the shipment never reached its
destination because Marble 10 sank with the 811 pieces of logs
somewhere off Cabuli Point in Palawan on its way to Manila. As
alleged by the petitioners in their complaint and as found by both
the trial and appellate courts, the barge where the logs were loaded
was not seaworthy such that it developed a leak. The appellate
court further found that one of the hatches was left open causing
water to enter the barge and because the barge was not provided
with the necessary cover or tarpauline, the ordinary splash of
seawaves brought more water inside the barge.

Respondent ignored the petitioner demand for payment of


150,000.00 for the loss of the shipment plus 100,000.00 as
unrealized profits.
Respondent Pioneer denied the claim of petitioner for the full
amount of 100,000.00 on the ground that its liability depended
upon the total loss of vessel only.
The trial court decided in favor of the plaintiff (petitioner).
The appellate court modified the trial courts decision and absolved
Pioneer from liability after finding that there was a breach of
implied warranty of seaworthiness on the part of the petitioners
and that the loss of the insured cargo was caused by the perils of
the ship and not by the perils of sea. It ruled that the loss is not
covered by the marine insurance policy.
ISSUE:
Whether or not the implied warranty of seaworthiness in marine
insurance attaches to the shipper who is not the shipowner.
HELD:
Section 113 of the Insurance Code provides:
In every marine insurance upon a ship or freight, or freightage, or
upon any thing which is the subject of marine insurance, a warranty
is implied that the ship is seaworthy.
Section 99 of the same Code also provides in part.
Marine insurance includes:
(1) Insurance against loss of or damage to:
(a) Vessels, craft, aircraft, vehicles, goods, freights, cargoes,
merchandise,
From the above-quoted provisions, there can be no mistaking the
fact that the term cargo can be the subject of marine insurance
and that once it is so made, the implied warranty of seaworthiness
immediately attaches to whoever is insuring the cargo whether he
be the shipowner or not.

Page 86 of 154

INSURANCE G01 CASE DIGESTS


Moreover, the fact that the unseaworthiness of the ship was
unknown to the insured is immaterial in ordinary marine insurance
and may not be used by him as a defense in order to recover on the
marine insurance policy.
Since the law provides for an implied warranty of seaworthiness in
every contract of ordinary marine insurance, it becomes the
obligation of a cargo owner to look for a reliable common carrier
which keeps its vessels in seaworthy condition. The shipper of cargo
may have no control over the vessel but he has full control in the
choice of the common carrier that will transport his goods. Or the
cargo owner may enter into a contract of insurance which
specifically provides that the insurer answers not only for the perils
of the sea but also provides for coverage of perils of the ship.
There is no doubt that the term perils of the sea extends only to
losses caused by sea damage, or by the violence of the elements,
and does not embrace all losses happening at sea. They insure
against losses from extraordinary occurrences only, such as stress of
weather, winds and waves, lightning, tempests, rocks and the like.
These are understood to be the perils of the sea referred in the
policy, and not those ordinary perils which every vessel must
encounter. Perils of the sea has been said to include only such
losses as are of extraordinary nature, or arise from some
overwhelming power, which cannot be guarded against by the
ordinary exertion of human skill and prudence. Damage done to a
vessel by perils of the sea includes every species of damages done
to a vessel at sea, as distinguished from the ordinary wear and tear
of the voyage, and distinct from injuries suffered by the vessel in
consequence of her not being seaworthy at the outset of her
voyage (as in this case). It is also the general rule that everything
which happens thru the inherent vice of the thing, or by the act of
the owners, master or shipper, shall not be reputed a peril, if not
otherwise borne in the policy.

On the contention of the petitioners that the trial court found that
the loss was occasioned by the perils of the sea characterized by the
storm and waves which buffeted the vessel, the records show that
the court ruled otherwise. It stated: x x x The other affirmative
defense of defendant Lighterage, That the supposed loss of the logs
was occasioned by force majeure was not supported by the
evidence. At the time Mable 10 sank, there was no typhoon but
ordinary strong wind and waves, a condition which is natural and
normal in the open sea. The evidence shows that the sinking of
Mable 10 was due to improper loading of the logs on one side so
that the barge was tilting on one side and for that it did not navigate
on even keel; that it was no longer seaworthy that was why it
developed leak; that the personnel of the tugboat and the barge
committed a mistake when it turned loose the barge from the
tugboat east of Cabuli point where it was buffeted by storm and
waves, while the tugboat proceeded to west of Cabuli point where
it was protected by the mountain side from the storm and waves
coming from the east direction. x x x
lt must be considered to be settled, furthermore, that a loss which,
in the ordinary course of events, results from the natural and
inevitable action of the sea, from the ordinary wear and tear of the
ship, or from the negligent failure of the ships owner to provide the
vessel with proper equipment to convey the cargo under ordinary
conditions, is not a peril of the sea. Such a loss is rather due to what
has been aptly called the peril of the ship. The insurer undertakes
to insure against perils of the sea and similar perils, not against
perils of the ship. As was well said by Lord Herschell in Wilson, Sons
& Co. v. Owners of Cargo per the Xantho ([1887], 12 A. C., 503, 509),
there must, in order to make the insurer liable, be some casualty,
something which could not be foreseen as one of the necessary
incidents of the adventure. The purpose of the policy is to secure an
indemnity against accidents which may happen, not against events
which must happen.

Page 87 of 154

INSURANCE G01 CASE DIGESTS


Barratry as defined in American Insurance Law is any willful
misconduct on the part of master or crew in pursuance of some
unlawful or fraudulent purpose without the consent of the owners,
and to the prejudice of the owners interest, (Sec. 171, U.S.
Insurance Law, quoted in Vance, Handbook on Law of Insurance,
1951, p. 929.) Barratry necessarily requires a willful and intentional
act in its commission. No honest error of judgment or mere
negligence, unless criminally gross, can be barratry. (See Vance on
Law of Insurance, p. 929 and cases cited therein.)
In the case at bar, there is no finding that the loss was occasioned
by the willful or fraudulent acts of the vessels crew. There was only
simple negligence or lack of skill. Hence, the second assignment of
error must likewise be dismissed.

119. GO TIACO V. UNION INSURANCE OF CANADA, 40 PHIL. 40


(1919)
GO TIACO v. UNION INSURANCE
FACTS

Union Insurance Society of Canton, Ltd., issued a marine


insurance policy upon a cargo of rice belonging to the Go Tiaoco
Brothers, which was transported in the early days of May, 1915, on
the steamship Hondagua from the port of Saigon to Cebu.

On discharging the rice from one of the compartments in


the after hold, upon arrival at Cebu, it was discovered that 1473
sacks had been damaged by sea water. The loss was P3,875.25. The trial court found that the inflow of the sea water during the
voyage was due to a defect in one of the drain pipes of the ship and
concluded that the loss was not covered by the policy of insurance.

The trial court made the ff findings: The drain pipe which
served as a discharge from the water closet passed down through
the compartment where the rice in question was stowed and
thence out to sea through the wall of the compartment, which was
a part of the wall of the ship. The joint or elbow where the pipe
changed its direction was of cast iron; and in course of time it had
become corroded and abraded until a longitudinal opening had
appeared in the pipe about one inch in length. This hole had been in
existence before the voyage was begun, and an attempt had been
made to repair it by filling with cement and bolting over it a strip of
iron. The effect of loading the boat was to submerge the vent, or
orifice, of the pipe until it was about 18 inches or 2 feet below the
level of the sea. As a consequence the sea water rose in the pipe.
Navigation under these conditions resulted in the washing out of
the cement-filling from the action of the sea water, thus permitting
the continued flow of the salt water into the compartment of rice.

Page 88 of 154

INSURANCE G01 CASE DIGESTS

The court found in effect that the opening above described


had resulted in course of time from ordinary wear and tear and not
from the straining of the ship in rough weather on that voyage. The
court also found that the repairs that had been made on the pipe
were slovenly and defective and that, by reason of the condition of
this pipe, the ship was not properly equipped to receive the rice at
the time the voyage was begun. For this reason the court held that
the ship was unseaworthy.

The policy purports to insure the cargo from the following


among other risks: "Perils . . . of the seas, men, of war, fire,
enemies, pirates, rovers, thieves, .jettisons, . . . barratry of the
master and mariners, and of all other perils, losses, and misfortunes
that have or shall come to the hurt, detriment, or damage of the
said goods and merchandise or any part thereof."
ISSUE
WON Union Insurance is liable for the loss of the Go Tiaco Brothers
HELD
NO - the words "all other perils, losses, and misfortunes" are to be
interpreted as covering risks which are of like kind (ejusdem
generis) with the particular risks which are enumerated in the
preceding part of the same clause of the contract. ''According to the
ordinary rules of construction, these words must be interpreted
with reference to the words which immediately precede them. They
were no doubt inserted in order to prevent disputes founded on
nice distinctions. X x x For example, if the expression 'perils of the
seas' is given its widest sense the general words have little or no
effect as applied to that case. If on the other hand that expression is
to receive a limited construction, as apparently it did in Cullen vs.
Butler (5 M. & S., 461), and loss by perils of the seas is to be
confined to loss ex marine tempestatis discrimine, the general

words become most important. X x x" (Thames and Mersey Marine


Insurance Co. vs. Hamilton, Fraser & Co.)
- a loss which, in the ordinary course of events, results from the
natural and inevitable action of the sea, from the ordinary wear and
tear of the ship, or from the negligent failure of the ship's owner to
provide the vessel with proper equipment to convey the cargo
under ordinary conditions, is not a peril of the sea. Such a loss is
rather due to what has been aptly called the "peril of the ship." The
insurer undertakes to insure against perils of the sea and similar
perils, not against perils of the ship. There must, in order to make
the insurer liable, be "some casualty, something which could not be
foreseen as one of the necessary incidents of the adventure. The
purpose of the policy is to secure an indemnity against accidents
which may happen, not against events which must happen."
(Wilson, Sons & Co. vs. Owners of Cargo per the Xantho)
- In the present case the entrance of the sea water into the ship's
hold through the defective pipe already described was not due to
any accident which happened during the voyage, but to the failure
of the ship's owner properly to repair a defect of the existence of
which he was apprised. The loss was therefore more analogous to
that which directly results from simple unseaworthiness than to
that which results from perils of the sea.
- there is no room to doubt the liability of the shipowner for such a
loss as occurred in this case. By parity of reasoning the insurer is not
liable; for, generally speaking, the shipowner excepts the perils of
the sea from his engagement under the bill of lading, while this is
the very peril against which the insurer intends to give protection.
As applied to the present case it results that the owners of the
damaged rice must look to the shipowner for redress and not to the
insurer. The same conclusion must be reached if the question be
discussed with reference to the seaworthiness of the ship. It is
universally accepted that in every contract of insurance upon
Page 89 of 154

INSURANCE G01 CASE DIGESTS


anything which is the subject of marine insurance, a warranty is
implied that the ship shall be seaworthy at the time of the inception
of the voyage. This rule is accepted in our own Insurance Law (Act
No. 2427, sec. 106). It is also well settled that a ship which is
seaworthy for the purpose of insurance upon the ship may yet be
unseaworthy for the purpose of insurance upon the cargo (Act No.
2427, sec. 106).

Disposition Decision of trial court is affirmed

120.

CHOA TIEK SENG V. COURT OF APPEALS, 183 SCRA 223 (1990)

Choa Tiek Seng v Court of Appeals, 183 SCRA 223 (1990)


Facts:

Petitioner imported some lactose crystals from Holland. The


importation involved fifteen (15) metric tons packed in 600
6-ply paper bags with polythelene inner bags, each bag at
25 kilos net. The goods were loaded at the port at
Rotterdam in sea vans on board the vessel "MS Benalder' as
the mother vessel, and thereafter aboard the feeder vessel
"Wesser Broker V-25" of respondent Ben Lines Container,
Ltd. (Ben Lines for short). The goods were insured by the
respondent Filipino Merchants' Insurance Co., Inc.
(insurance company for short) against all risks under the
terms of the insurance cargo policy. Upon arrival at the
port of Manila, the cargo was discharged into the custody of
the arrastre operator respondent E. Razon, Inc. (broker for
short), prior to the delivery to petitioner through his broker.
Of the 600 bags delivered to petitioner, 403 were in bad
order. The surveys showed that the bad order bags suffered
spillage and loss.

Petitioner filed a claim for said loss against respondent


insurance company.
Respondent insurance company rejected the claim alleging
that assuming that spillage took place while the goods were
in transit, petitioner and his agent failed to avert or
minimize the loss by failing to recover spillage from the sea
van, thus violating the terms of the insurance policy sued
upon; and that assuming that the spillage did not occur
while the cargo was in transit, the said 400 bags were
loaded in bad order, and that in any case, the van did not
carry any evidence of spillage.
Hence, petitioner filed the complaint in the RTC of Manila
against respondent insurance company seeking payment. In
its answer, respondent insurance company denied all the
material allegations of the complaint and raised several
special defenses as well as a compulsory counterclaim.
Respondent insurance company filed a third-party
complaint against respondents Ben Lines and broker.
Respondent broker filed its answer to the third-party
complaint denying liability and arguing, among others, that
the petitioner has no valid cause of action against it.
Similarly, Ben Lines filed its answer denying any liability and
a special defense arguing that respondent insurance
company was not the proper party in interest and has no
connection whatsoever with Ben Lines Containers, Ltd. and
that the third-party complaint has prescribed under the
applicable provisions of the Carriage of Goods by Sea Act.
Respondent Ben Lines filed a motion for preliminary hearing
on the affirmative defense of prescription. The trial court
deferred resolution of the aforesaid motion after trial on
the ground that the defense of prescription did not appear
to be indubitable.
After the pre-trial conference and trial on the merits, on
March 31, 1986, the court a quo rendered a judgment
Page 90 of 154

INSURANCE G01 CASE DIGESTS

dismissing the complaint, the counterclaim and the thirdparty complaint with costs against the petitioner.
Hence, the appeal to the Court of Appeals by petitioner
which, in due course, as aforestated, affirmed the judgment
of the trial court.
A motion for reconsideration of said judgment was denied
by the appellate court in a resolution.
Petitioner now filed this petition for review on certiorari in
this Court.

ISSUES:
WON court erred in holding that an "all risks" coverage covers only
losses occasioned by or resulting from "extra and fortuitous events"
despite the clear and unequivocal definition of the term made and
contained in the policy sued upon.
Held: YES. The decision appealed from is hereby REVERSED AND SET
ASIDE. Respondent Filipinas Merchants Insurance Company, Inc. is
liable.
There is no question that the 403 bags in damaged condition
delivered and received by petitioner.
Nevertheless, on the assumption that the cargo suffered damages,
the appellate court ruled: Even assuming that the cargo indeed
sustained damage, still the appellant cannot hold the appellee
insurance company liable on the insurance policy. In the case at bar,
appellant failed to prove that the alleged damage was due to risks
connected with navigation. A distinction should be made between
"perils of the sea" which render the insurer liable on account of the
loss and/or damage brought about thereof and "perils of the ship"
which do not render the insurer liable for any loss or damage. Perils
of the sea or perils of navigation embrace all kinds of marine
casualties, such as shipwreck, foundering, stranding, collision and

every specie of damage done to the ship or goods at sea by the


violent action of the winds or waves. They do not embrace all loses
happening on the sea. A peril whose only connection with the sea is
that it arises aboard ship is not necessarily a peril of the sea; the
peril must be of the sea and not merely one accruing on the sea.
Moreover, the cargo in question was insured in an "against all risk
policy." Insurance "against all risk" has a technical meaning in
marine insurance. Under an "all risk" marine policy, there must be a
general rule be a fortuitous event in order to impose liability on the
insurer; losses occasioned by ordinary circumstances or wear and
tear are not covered, thus, while an "all risk" marine policy purports
to cover losses from casualties at sea, it does not cover losses
occasioned by the ordinary circumstances of a voyage, but only
those resulting from extra and fortuitous events.
It has been held that damage to a cargo by high seas and other
weather is not covered by an "all risk" marine policy, since it is not
fortuitous, particularly where the bad weather occurs at a place
where it could be expected at the time in question.
The Court disagrees.
In Gloren Inc. vs. Filipinas Cia. de Seguros, it was held that an all
risk insurance policy insures against all causes of conceivable loss
or damage, except as otherwise excluded in the policy or due to
fraud or intentional misconduct on the part of the insured. It
covers all losses during the voyage whether arising from a marine
peril or not, including pilferage losses during the war.
In the present case, the "all risks" clause of the policy sued upon
reads as follows:
5. This insurance is against all risks of loss or
damage to the subject matter insured but shall in
Page 91 of 154

INSURANCE G01 CASE DIGESTS


no case be deemed to extend to cover loss,
damage, or expense proximately caused by delay or
inherent vice or nature of the subject matter
insured. Claims recoverable hereunder shall be
payable irrespective of percentage.

121. FILIPINO MERCHANTS INS. V. COURT OF APPEALS, 179 SCRA


638 (1989)
Marine Insurance
Filipino Merchants Insurance Co., Inc v Court of Appeals

The terms of the policy are so clear and require no interpretation.


The insurance policy covers all loss or damage to the cargo except
those caused by delay or inherent vice or nature of the cargo
insured. It is the duty of the respondent insurance company to
establish that said loss or damage falls within the exceptions
provided for by law, otherwise it is liable therefor.
An "all risks" provision of a marine policy creates a special type of
insurance which extends coverage to risks not usually
contemplated and avoids putting upon the insured the burden of
establishing that the loss was due to peril falling within the policy's
coverage. The insurer can avoid coverage upon demonstrating that
a specific provision expressly excludes the loss from coverage.
In this case, the damage caused to the cargo has not been
attributed to any of the exceptions provided for nor is there any
pretension to this effect. Thus, the liability of respondent
insurance company is clear.

FACTS:
1. The consignee of the shipment of fishmeal loaded on board the
vessel SS Bougainville and unloaded at the Port of Manila on or
about December 11, 1976 and seeks to recover from the defendant
insurance company the amount of P51,568.62 representing
damages to the said shipment.

2. The defendant brought a party against third party


defendants Compagnie Maritime Des Chargues Reunis and/or E.
Razon Inc seeking judgment against the third defendants.
3. Evidence shows that the plaintiff insured said shipment with the
defendant insurance company under the said cargo Policy No. M2678 for the sum of P267,653.59 for the 600 metric tons of fishmeal
I new gunny bags of 90 kilos each from Thailand to Manila
against all risks under warehouse to warehouse terms. What was
delivered was only 59.940 metric tons not 600 tons at $395.42 a
ton.
4. The fishmeal in 666 new gunny bags were unloaded from the ship
on December 11, 1976 at Manila unto the arrastre contractor E.
Razon, Inc. and defendant's surveyor ascertained and certified
that in such discharge 105 bags were in bad order condition as
Page 92 of 154

INSURANCE G01 CASE DIGESTS


jointly surveyed by the ship's agent and the arrastre contractor.
The condition of the bad order was proven in the survey report.

inconsistent with the broad protective purpose of "all risks"


insurance.

5. The cargo was also surveyed by the arrastre contractor before the
delivery and a total of 227 bags were in bad order condition.

In the present case, there being no showing that the loss was
caused by any of the excepted perils, the insurer is liable under
the policy.

6. The Filipino Merchants Insurance Company refused to pay


the claim when the consignee made a formal claim which
amounted to P51,568.62.

ISSUE:

There is no evidence presented to show that the condition of the


gunny bags in which the fishmeal was packed was such that they
could not hold their contents in the course of the necessary transit,
much less any evidence that the bags of cargo had burst as the
result of the weakness of the bags themselves. Had there been such
a showing that spillage would have been a certainty, there may
have been good reason to plead that there was no risk covered by
the policy.

Whether or not the insurer is liable under the "all risks policy''?

OTHER TOPIC: INSURABLE INTEREST

HELD:

Anent the issue of insurable interest, we uphold the ruling of


the respondent court that private respondent, as consignee of the
goods in transit under an invoice containing the terms under "C & F
Manila," has insurable interest in said goods.

7. RTC: In favor of the consignee. Defendant was ordered to pay


P51,568.62.
8. CA: Affirmed the decision of the RTC

RATIONALE:
A marine insurance policy providing that the insurance was to
be "against all risks" must be construed as creating a special
insurance and extending to other risks than are usually
contemplated, and covers all losses except such as arise from the
fraud of the insured. The burden of the insured, therefore, is to
prove merely that the goods he transported have been
lost, destroyed or deteriorated. Thereafter, the burden is
shifted to the insurer to prove that the loss was due to
excepted perils. To impose on the insured the burden of
proving the precise cause of the loss or damage would be

Page 93 of 154

INSURANCE G01 CASE DIGESTS

122.

PUROMINES V. COURT OF APPEALS, 220 SCRA 281 (1993)]

Puromines vs Court of Appeals


Facts:
1. Puromines, Inc. (Puromines for brevity) and Makati Agro
Trading, Inc. (not a party in this case) entered into a
contract with private respondents Philipp Brothers Oceanic,
Inc. for the sale of prilled Urea in bulk.
2. On or about May 22, 1988, the vessel M/V "Liliana
Dimitrova" loaded on board at Yuzhny, USSR a shipment of
15,500 metric tons prilled Urea in bulk complete and in
good order and condition for transport to Iloilo and Manila,
to be delivered to petitioner.
3. 3 Bill of Ladings were issued. The shipment covered by Bill
of Lading no. 2 was discharged in Iloilo safely, while those
covered by Bill of Ladings Nos. 1 and 3 arrived in Manila in
bad condition.
4. Damages were valued at P683, 056. 29 including additional
discharging expenses.
5. Petitioner filed a complaint for breach of contract of
carriage against Maritime Factors as ship-agent and owner
of the vessel, and against Philipp Brothers as charterer of
the vessel.
6. Philipp Brothers filed a motion to dismiss on the ground
that the complaint stated no cause of action against them.
Issue:
1. Whether or not petitioner has a cause of action against
Philipp Brothers
Held:
1. NO RULING. There is a valid arbitration clause. Court
ordered for arbitration as to the issue.

2. Petitioner states in its complainants that Philipp Brothers


"was the charterer of the vessel MV 'Liliana Dimitrova'
which transported the shipment from Yuzhny USSR to
Manila."
3. Petitioner further alleged that the caking and hardening,
wetting and melting, and contamination by rust and dirt of
the damaged portions of the shipment were due to the
improper ventilation and inadequate storage facilities of the
vessel; that the wetting of the cargo was attributable to the
failure of the crew to close the hatches before and when it
rained while the shipment was being unloaded in the Port
of Manila
4. Moreover, in its Opposition to the Motion to Dismiss,
petitioner said that "[t]he cause of action of the complaint
arose from breach of contract of carriage by the vessel that
was chartered by defendant Philipp Brothers
5. American jurisprudence defines charter party as a contract
by which an entire ship or some principal part thereof is let
by the owner to another person for a specified time or use.
6. Charter or charter parties are of two kinds. Charter of
demise or bareboat and contracts of affreightment.
7. Under the demise or bareboat charter of the vessel, the
charterer will generally be considered as owner for the
voyage or service stipulated. The charterer mans the vessel
with his own people and becomes, in effect, the owner pro
hac vice, subject to liability to others for damages caused by
negligence.
8. To create a demise the owner of a vessel must completely
and exclusively relinquish possession, anything short of such
a complete transfer is a contract of affreightment (time or
voyage charter party) or not a charter party at all.
9. A contract of affreightment is in which the owner of the
vessel leases part or all of its space to haul goods for others.
It is a contract for a special service to be rendered by the
owner of the vessel and under such contract the general
Page 94 of 154

INSURANCE G01 CASE DIGESTS

10.

11.

12.

13.

owner retains the possession, command and navigation of


the ship, the charterer or freighter merely having use of the
space in the vessel in return for his payment of the charter
hire.
If the charter is a contract of affreightment, which leaves
the general owner in possession of the ship as owner for the
voyage, the rights, responsibilities of ownership rest on the
owner and the charterer is usually free from liability to third
persons in respect of the ship.
If possession is transferred to the charterer by virtue of a
demise, the charterer, and not the owner, is liable as carrier
on the contract of affreightment made by himself or by the
master with third persons, and is answerable for loss,
damage or non-delivery of goods received for
transportation.
An owner who retains possession of the ship, though the
hold is the property of the charterer, remains liable as
carrier and must answer for any breach of duty as to the
care, loading or unloading of the cargo.
Assuming that in the present case, the charter party is a
demise or bareboat charter, then Philipp Brothers is liable
to Puromines, Inc., subject to the terms and conditions of
the sales contract. On the other hand, if the contract
between respondent and the owner of the vessel MV
"Liliana Dimitrova" was merely that of affreightment, then it
cannot be held liable for the damages caused by the breach
of contract of carriage, the evidence of which is the bills of
lading

123.

CALTEX (PHILS.) INC. V. SULPICIO LINES, 315 SCRA 709 (1999)

Caltex vs. Sulpicio Lines (1999)


Facts:
1. Dec. 19, 1987; 8:00 pm: A motor tanker MT Vector (owned
and operated by Vector Shipping Corporation) carried 8,800
barrels of petroleum products of Caltex by virtue of a
charter contract. From Bataan, it headed to Masbate.
2. Dec. 20, 1987 6:30 am: MV Doa Paz passenger and cargo
vessel owned and operated by Sulpicio Lines, left the port of
Tacloban headed for Manila with 1,493
passengers indicated in the Coast Guard Clearance.
3. Dec. 20, 1987: MT Vector collided with MV Doa Paz in the
open sea between Marinduque and Oriental Mindoro,
killing almost all the passengers and crew members of both
ships. Only 24 survived.
4. MV Doa Paz carried an estimated 4,000 passengers most
were not in the passenger manifest.
5. The Board of Marine found that the Vector Shipping
Corporation was at fault and responsible for the collision
with MV Doa Paz.
6. Teresita Caezal and Sotera Caezal, Sebastian Caezals
wife and mother filed a complaint for Damages Arising
from Breach of Contract of Carriage against Sulpicio Lines
for the death of Sebastian Caezal and his 11-year old
daughter Corazon Caezal.
7. Sulpicio Lines filed a 3rd party complaint against Vector
Shipping Corporation and Caltex. It alleged that Caltex
chartered MT Vector with gross and evident bad faith
knowing fully well that MT Vector was improperly manned,
ill-equipped, unseaworthy and a hazard to safe navigation.
8. The RTC dismissed the 3rd party complaint against Caltex.
The CA modified the ruling by including Caltex as one of
those liable for damages.
Page 95 of 154

INSURANCE G01 CASE DIGESTS


Issue: W/N Caltex is liable for damages for the collision between
Vector and Doa Paz
Held: NO. Caltex and Vector entered into a contract of
affreightment, also known as a voyage charter. A charter party is a
contract by which an entire ship, or some principal part thereof, is
let by the owner to another person for a specified time or use; a
contract of affreightment is one by which the owner of a ship or
other vessel lets the whole or part of her to a merchant or other
person for the conveyance of goods, on a particular voyage, in
consideration of the payment of freight.

MT Vector is a common carrier. Charter parties fall into three main


categories: (1) Demise or bareboat, (2) time charter, (3) voyage
charter. Does a charter party agreement turn the common carrier
into a private one? We need to answer this question in order to
shed light on the responsibilities of the parties. In this case, the
charter party agreement did not convert the common carrier into a
private carrier. The parties entered into a voyage charter, which
retains the character of the vessel as a common carrier.

A contract of affreightment may be either time charter, wherein the


leased vessel is leased to the charterer for a fixed period of time,
or voyage charter, wherein the ship is leased for a single voyage. In
both cases, the charter-party provides for the hire of the vessel
only, either for a determinate period of time or for a single or
consecutive voyage, the ship owner to supply the ships store, pay
for the wages of the master of the crew, and defray the expenses
for the maintenance of the ship.

Is Caltex liable for damages under the Civil Code? We rule that it is
not. Sulpicio argues that Caltex negligently shipped its highly
combustible fuel cargo aboard an unseaworthy vessel such as the
MT Vector. The charterer of a vessel has no obligation before
transporting its cargo to ensure that the vessel it chartered
complied with all legal requirements. The duty rests upon the
common carrier simply for being engaged in public service.

Under a demise or bareboat charter on the other hand, the


charterer mans the vessel with his own people and becomes, in
effect, the owner for the voyage or service stipulated, subject to
liability for damages caused by negligence.

The relationship between the parties in this case is governed by


special laws. Because of the implied warranty of seaworthiness,
shippers of goods, when transacting with common carriers, are not
expected to inquire into the vessels seaworthiness, genuineness of
its licenses and compliance with all maritime laws. To demand
more from shippers and hold them liable in case of failure exhibits
nothing but the futility of our maritime laws insofar as the
protection of the public in general is concerned. By the same token,
we cannot expect passengers to inquire every time they board a
common carrier, whether the carrier possesses the necessary
papers or that all the carriers employees are qualified. Such a
practice would be an absurdity in a business where time is always of

If the charter is a contract of affreightment, which leaves the


general owner in possession of the ship as owner for the voyage,
the rights and the responsibilities of ownership rest on the
owner. The charterer is free from liability to third persons in respect
of the ship. Caltex is the charterer. The charterer has no liability for
damages under Philippine Maritime laws.

Page 96 of 154

INSURANCE G01 CASE DIGESTS


the essence. Considering the nature of transportation business,
passengers and shippers alike customarily presume that common
carriers possess all the legal requisites in its operation.

124.

SAN MIGUEL V. HEIRS OF INGUITO, 384 SCRA 87 (2002)

San Miguel vs Heirs of Inguito


GR No. 141716 July 4, 2002
Thus, the nature of the obligation of Caltex demands ordinary
diligence like any other shipper in shipping his cargoes. A cursory
reading of the records convinces us that Caltex had reasons to
believe that MT Vector could legally transport cargo that time of the
year.

Caltex and Vector Shipping Corporation had been doing business


since 1985, or for about two years before the tragic incident
occurred in 1987. Past services rendered showed no reason for
Caltex to observe a higher degree of diligence.

Clearly, as a mere voyage charterer, Caltex had the right to presume


that the ship was seaworthy as even the Philippine Coast Guard
itself was convinced of its seaworthiness. All things considered, we
find no legal basis to hold petitioner liable for damages.

Topic: Marine Insurance - Insurable Interest


Facts:
San Miguel (SMC) entered into a Time Charter Party
Agreement with Julius Ouano, doing business under the
name and style J. Ouano Marine Services. SMC chartered
MV Dona Roberta, owned by Ouano, for a period of 2 years,
for the purpose of transporting SMC's beverage products
from Mandaue City plant to various points in Visayas and
Mindanao.
The agreement stated that:
o Ouano warrants ownership, title and interest over
the vessel
o that the vessel is put at SMC's disposal; SMC hires
the use and service of the vessel
o that the owner warrants that the vessel is
seaworthy and in proper, useful and operational
condition; and that SMC should immediately notify
Ouano in case it finds any defect in the vessel
o that there is no employer-employee relationship
between Ouano (and/or the crew of the vessel) and
SMC; Ouano held SMC free from all claims and
liabilities arising out of the acts of the crew and the
condition of the vessel
o that Ouano undertook to pay all compensation of
all the vessel's crew
o that Ouano shall indemnify SMC for damages and
losses arising from the incompetence and/or
negligence of, and/or the failure to observe the
Page 97 of 154

INSURANCE G01 CASE DIGESTS

required extra-ordinary diligence by the crew;


Ouano would automatically be liable for
shortlanded shipment, where the value shall be
withheld from Ouano's collectibles, and wrong
levels, where the value shall likewise be withheld,
but SMC shall reimburse Ouano if SMC's laboratory
shall make a determination that the bottles were
never opened after it left the plant
SMC issued sailing orders to the master of the vessel,
Captain Inguito, stating where to sail to, when it is expected
to depart and arrive, to maintain communications, and to
observe weather condition (exercise utmost precautionary
measures)
Inguito obtained the necessary sailing clearance from the
Philippine Coast Guard. The loading was completed on
schedule, but the vessel did not leave in accordance with
the orders.
November 12, 1990:
o 4am - a typhoon was spotted near Samar moving
towards the general direction of Eastern Visayas
o 6am - the vessel left Mandaue
o 7am - while still abeam Cawit Island off Cebu, SMC
Radio Operator Moreno contacted Inguito and
advised him to take shelter; Inguito replied that
they would proceed since the typhoon was far away
from them and that the winds were in their favor
o 4pm - Moreno reiterated to Inguito the advice and
pointed out that it would be difficult to take shelter
after passing Balicasag Island because they were
approaching open sea; Inguito refused to heed
o 8pm - vessel was 38 miles southeast of Balicasag
o 10pm - vessel was 25 miles approaching Sulauan
Point; moments later power went out in Moreno's
office

11:40pm - power resumed, Moreno made a series


of calls to the vessel but he failed to get in touch
with anyone
November 13, 1990:
o 1:15am - Inguito called Moreno over the radio and
requested that he contact Rico Ouano, son of the
owner, because they needed a helicopter to rescue
them; vessel was 20 miles west of Sulauan Point.
Inguiti requested for a helicopter to rescue them;
the Chief Engineer informed Rico that they can no
longer stop the water from coming into the vessel
because the crew members were feeling dizzy from
the petroleum fumes
o 2:30am - the vessel sank; only 5 of the 25 officers
and crew on board survived
Ouano, in lieu of the Inguito (who died in the sea tragedy),
filed a Marine Protest
the heirs of Inguito filed a complaint for tort against SMC
and Ouano with the RTC of Lapu-Lapu City
Ouano filed a cross-claim: the proximate cause of the loss of
the vessel and of its officers and crew was the fault and
negligence of SMC, which had complete control and
disposal of the vessel as charterer and which issued a sailing
order despite being forewarned of the typhoon; he prayed
for indemnification of the cost of the vessel and unrealized
rentals and earning
SMC: it was Ouano who had control, supervision and
responsibilities over the navigation of the vessel; Ouano
never initiated contact with the vessel despite knowledge of
the typhoon; the proximate cause was Ouano's breach of
his obligation to provide SMC with a seaworthy vessel duly
manned by competent crew members; demanded the value
of the cargo lost in sea
RTC: the proximate cause was attributable to SMC; liable for
the loss of earnings of those who died in the sea tragedy,
Page 98 of 154

INSURANCE G01 CASE DIGESTS

moral and exemplary damages for heirs of each deceased


crew member; liable for the value of the total loss of the
vessel and unrealized rental earnings
SMC and Ouano appealed to the CA. CA - modified the
decision: SMC and Ouano are jointly liable to the heirs of
the deceased; the claims of SMC and Ouano against each
other are dismissed

Issue:
Who should be held liable for the loss
note: i don't know how this involves insurable interest >.< it only
talks about how the owner shall indemnify the charterer
Ruling:
Ouano, the owner of the vessel, is liable to the heirs of the
deceased, and to SMC for the loss of their goods
a charter party is a contract by virtue of which the owner or
the agent of a vessel binds himself to transport
merchandise or persons for a fixed price. It has also been
defined as a contract by virtue of which the owner or the
agent of the vessel leases for a certain price the whole or a
portion of the vessel for the transportation of goods or
persons from one port to another
2 types of charter parties: (1) bareboat or demise: the
charterer mans the vessel with his own people; (2) contract
of affreightment: the owner of the vessel leases part or all
of its space to haul goods for others. It is a contract for
special service to be rendered by the owner of the vessel.
Under such contract the ship owner retains the possession,
command and navigation of the ship, the charterer or
freighter merely having use of the space in the vessel in
return for his payment of the charter hire.
2 types of contract of affreightment: time charter and
voyage charter

If the charter is a contract of affreightment, which leaves


the general owner in possession of the ship as owner for the
voyage, the rights and the responsibilities of ownership rest
on the owner. The charterer is free from liability to third
persons in respect of the ship
the charter between SMC and Ouano was a contract of
affreighment, as evidenced by the the provisions of their
agreement
Ouano was the employer, and had command and control
over the vessel and the crew
SMC should free from liability for any loss or damage
sustained during the voyage, unless it can be shown that
the loss was due to its fault or negligence, which the
evidence does not show (attempts of SMC's radio operator
to advise Inguito to take shelter and efforts to keep in
contact)
Absent any showing of fault or negligence from SMC, Ouano
had the clear duty to ensure the safe carriage and arrival of
goods transported on board its vessels. Ouano also
expressly warranted that the vessel was seaworthy.
seaworthiness: the sufficiency of the vessel in materials,
construction, equipment, officers, men, and outfit for the
trade or service in which it is employed; includes the fitness
of a ship for its fuel and provisions supply, the quality of its
officers and crew, and its adaptability for the time of voyage
proposed
The CA found that the proximate cause of the sinking was
the negligence of Inguito
Ouano is vicariously liable for the negligent acts of his
employee, Captain Inguito. Under Articles 2176 and 2180 of
the Civil Code, owners and managers are responsible for
damages caused by the negligence of a servant or an
employee, the master or employer is presumed to be
negligent either in the selection or in the supervision of that
employee. This presumption may be overcome only by
Page 99 of 154

INSURANCE G01 CASE DIGESTS

satisfactorily showing that the employer exercised the care


and the diligence of a good father of a family in the
selection and the supervision of its employee.
Ouano failed to present proof that he exercised the due
diligence in the selection and supervision of the captain.
Thus, he is vicariously liable for the loss of lives and
property occasioned by the lack of care and negligence of
his employee

**as for insurable interest, Section 100, 103, 105 and 106 provide
for who has an insurable interest in this case
Section 100. The owner of a ship has in all cases an insurable
interest in it, even when it has been chartered by one who
covenants to pay him its value in case of loss; Provided, That in this
case the insurer shall be liable for only that part of the loss which
the insured cannot recover from the charterer
Sec. 103. The owner of a ship has an insurable interest in expected
freightage which according to the ordinary and probable course of
things he would have earned but for the intervention of a peril
insured against or other peril incident to the voyage. (freightage:
the benefit which is to accrue to the owner from the use of the
vessel in the voyage contemplated, ir the benefit derived from the
employment of the ship)

125.

CALTEX (PHILS.) INC. V. SULPICIO LINES, 315 SCRA 709 (1999)

Caltex vs Sulpicio Lines


Facts:
MV Vector and Caltex entered into a charter contract (contract of
affreightment). On December 20, 1987, motor tanker MV Vector,
carrying petroleum products of Caltex, collided in the open sea with
passenger ship MV Doa Paz, causing the death of all but 25 of the
latters passengers. Among those who died were Sebastian Canezal
and his daughter Corazon Canezal. On March 22, 1988, the board of
marine inquiry found that Vector Shipping Corporation was at fault.
On February 13, 1989, Teresita Caezal and Sotera E. Caezal,
Sebastian Caezals wife and mother respectively, filed with the
Regional Trial Court of Manila a complaint for damages arising from
breach of contract of carriage against Sulpicio Lines. Sulpicio filed a
third-party complaint against Vector and Caltex. The trial court
dismissed the complaint against Caltex, but the Court of Appeals
included the same in the liability. Hence, Caltex filed this petition.

Sec. 105. One who has an interest in the thing from which profits
are expected to proceed has an insurable interest in the profits.

Issue:

Sec. 106. The charterer of a ship has an insurable interest in it, to


the extent that he is liable to be damnified by its loss.

Whether or not Caltex, as the charterer of a sea vessel, is liable for


damages resulting from a collision between the chartered vessel
and a passenger ship?

Ruling:
Page 100 of 154

INSURANCE G01 CASE DIGESTS


The Supreme Court ruled in the negative. The Supreme Court ruled
that MT Vector is a common carrier. The charter party agreement
did not convert the common carrier into a private carrier. The
parties entered into a voyage charter, which retains the character of
the vessel as a common carrier. It is imperative that a public carrier
shall remain as such, notwithstanding the charter of the whole or
portion of a vessel by one or more persons, provided the charter is
limited to the ship only, as in the case of a time-charter or voyage
charter. It is only when the charter includes both the vessel and its
crew, as in a bareboat or demise that a common carrier becomes
private, at least insofar as the particular voyage covering the
charter-party is concerned. Indubitably, a ship-owner in a time or
voyage charter retains possession and control of the ship, although
her holds may, for the moment, be the property of the charterer. A
common carrier is a person or corporation whose regular business is
to carry passengers or property for all persons who may choose to
employ and to remunerate him. 16 MT Vector fits the definition of a
common carrier under Article 1732 of the Civil Code.

being engaged in "public service." The relationship between the


parties in this case is governed by special laws. Because of the
implied warranty of seaworthiness, shippers of goods, when
transacting with common carriers, are not expected to inquire into
the vessels seaworthiness, genuineness of its licenses and
compliance with all maritime laws. To demand more from shippers
and hold them liable in case of failure exhibits nothing but the
futility of our maritime laws insofar as the protection of the public
in general is concerned. Such a practice would be an absurdity in a
business where time is always of the essence. Considering the
nature of transportation business, passengers and shippers alike
customarily presume that common carriers possess all the legal
requisites in its operation.

The public must of necessity rely on the care and skill of common
carriers in the vigilance over the goods and safety of the passengers,
especially because with the modern development of science and
invention, transportation has become more rapid, more
complicated and somehow more hazardous. For these reasons, a
passenger or a shipper of goods is under no obligation to conduct
an inspection of the ship and its crew, the carrier being obliged by
law to impliedly warrant its seaworthiness.
The charterer of a vessel has no obligation before transporting its
cargo to ensure that the vessel it chartered complied with all legal
requirements. The duty rests upon the common carrier simply for
Page 101 of 154

INSURANCE G01 CASE DIGESTS


126.

SAN MIGUEL V. HEIRS OF INGUITO, 384 SCRA 87 (2002)

G.R. No. 141716. July 4, 2002]


SAN MIGUEL CORPORATION, petitioner, vs. HEIRS OF SABINIANO
INGUITO, and JULIUS OUANO, respondents.

SMC Radio Operator Moreno contacted Captain Inguito 3 times


through the radio and advised him to take shelter but Capt. Inquito
did not heed the advice and continued to sail.

FACTS:

At 1:15 a.m., November 13, 1990, Captain Inguito called


Moreno over the radio and requested him to contact Rico Ouano,
son of Julius Ouano, because they needed a helicopter to rescue
them.

San Miguel Corporation entered into a Time Charter Party


Agreement with Julius Ouano, owner of Ouano Marine Services.
Under the terms of the agreement, SMC will charter the M/V Doa
Roberta owned by Julius Ouano for a period of two years, to
transport SMCs beverage products from its Mandaue City plant to
various points in Visayas and Mindanao.

At 2:30 a.m. of November 13, 1990, the M/V Doa Roberta


sank. Out of the 25 officers and crew on board the vessel, only five
survived.
Shipowner Julius Ouano, in lieu of the captain who perished in
the sea tragedy, filed a Marine Protest.

Pertinent portions of the Time Charter Party Agreement state:


1.
OWNER [i.e., Ouano] warrants his ownership, title and interest
over the vessel DOA ROBERTA;
4.
OWNER warrants that the vessel is seaworthy;
9.
There shall be no employer-employee relations between the
OWNER and/or its vessels crew on one hand and the CHARTERER
on the other;
10. The OWNER shall undertake to pay all compensation of all the
vessels crew;
11. The OWNER shall be responsible to and shall indemnify the
CHARTERER for damages and losses arising from the incompetence
and/or negligence of its crew.

The heirs of the deceased captain and crew, as well as the


survivors filed a complaint for tort against San Miguel Corporation
and Julius Ouano.
Julius Ouano alleged that the proximate cause of the loss of the
vessel and its officers and crew was the fault and negligence of SMC
which issued the sailing order for its departure despite being
forewarned of the impending typhoon.
SMC argued that the proximate cause of the sinking was
Ouanos breach of his obligation to provide SMC with a seaworthy
vessel duly manned by competent crew members.

During the term of the charter, SMC issued sailing orders to MV


Doa Roberta Captain Inguito to sail on November 12, 1990.

RTC --- proximate cause of the loss of the M/V Doa Roberta was
attributable to SMC.

Meanwhile, at 4:00 a.m. of November 12, 1990, typhoon


Ruping was spotted.

Both SMC and Ouano appealed to the Court of Appeals. SMC


argued that as mere charterer, it did not have control of the vessel
and that the proximate cause of the loss of the vessel and its cargo
Page 102 of 154

INSURANCE G01 CASE DIGESTS


was the negligence of the ship captain. For his part, Ouano
complained of the reduced damages awarded to him by the trial
court.
CA --- SMC and Ouano solidarily liable to plaintiffs heirs and
survivors, except to the heirs of Capt. Sabiniano Inguito

message. Neither Ouano nor his son was available during the entire
time that the vessel set out and encountered foul weather.
Considering that the charter was a contract of affreightment, the
shipowner had the clear duty to ensure the safe carriage and arrival
of goods transported on board its vessels. More specifically, Ouano
expressly warranted in the Time Charter Party that his vessel was
seaworthy.

ISSUE: WHETHER THE WARRANT BY THE SHIPOWNER OF THE


SEAWORTHINESS OF THE M/V DOA ROBERTA WAS VIOLATED.

WHAT IT MEANS FOR A VESSEL TO BE SEAWORTHY

HELD: YES.

For a vessel to be seaworthy, it must be adequately equipped


for the voyage and manned with a sufficient number of competent
officers and crew.[32]

CHARTER PARTY AGREEMENT MAKES SHIPOWNER LIABLE TO SMC

DEFINITION OF SEAWORTHINESS

Under the foregoing definitions, as well as the clear terms of


the Charter Party Agreement between the parties, the charterer,
SMC, should be free from liability for any loss or damage sustained
during the voyage,[31] unless it be shown that the same was due to
its fault or negligence.

Seaworthiness is defined as the sufficiency of the vessel in


materials, construction, equipment, officers, men, and outfit, for
the trade or service in which it is employed.[33] It includes the fitness
of a ship for a particular voyage with reference to its physical and
mechanical condition, the extent of its fuel and provisions supply,
the quality of its officers and crew, and its adaptability for the time
of voyage proposed.[34]

The evidence does not show that SMC or its employees were
amiss in their duties.

SHIPOWNER WAS NEGLIGENT

PROXIMATE CAUSE OF THE SINKING WAS NEGLIGENCE OF CAPTAIN


INGUITO

In contrast to the care exercised by Moreno, Rico Ouano tried


to communicate with the captain only after receiving the S.O.S.

It appears that the proximate cause of the sinking of the vessel


was the gross failure of the captain of the vessel to observe due
Page 103 of 154

INSURANCE G01 CASE DIGESTS


care and to heed SMCs advices to take shelter. He was fully
apprised of typhoon Ruping and its strength. Due diligence
dictates that at any time before the vessel was in distress, he should
have taken shelter in order to safeguard the vessel and its crew.
It is very clear that Captain Sabiniano Inguito had sufficient
time within which to secure his men and the vessel. But he waited
until the vessel was already in distress. In fact, there was an incident
when a sailing order was issued by SMC to Inguito but he decided
not to proceed with the voyage because of a tropical storm.[35]

OUANO VICARIOUSLY LIABLE UNDER ARTS. 2176 AND 2180 OF THE


CIVIL CODE
Under Articles 2176 and 2180 of the Civil Code, owners and
managers are responsible for damages caused by the negligence of
a servant or an employee, the master or employer is presumed to
be negligent either in the selection or in the supervision of that
employee. This presumption may be overcome only by satisfactorily
showing that the employer exercised the care and the diligence of a
good father of a family in the selection and the supervision of its
employee.[38]
Ouano miserably failed to overcome the presumption of his
negligence.

previous occasion, SMC issued a sailing order to the captain of the


M/V Doa Roberta, but the vessel cancelled its voyage due to
typhoon.[39] Likewise, it appears from the records that SMC issued
the sailing order on November 11, 1990, before typhoon Ruping
was first spotted at 4:00 a.m. of November 12, 1990.[40]

ONLY SHIPOWNER OUANO IS LIABLE FOR THE LOSSES


Consequently, Ouano should answer for the loss of lives and
damages suffered by the heirs of the officers and crew members
who perished on board the M/V Doa Roberta, except Captain
Sabiniano Inguito. The award of damages granted by the Court of
Appeals is affirmed only against Ouano, who should also indemnify
SMC for the cost of the lost cargo, in the total amount of
P10,278,542.40.[41]

(NOTE: The Time Charter Agreement between SMC and Ouano in


this case was a contract of of affreightment which means that the
shipowner or the captain of the vessel had command and control of
the vessel at all times. This is in contrast to a bareboat or demise
charter wherein it is the charterer who mans the vessel with his
own people and therefore had control of the vessel.)

SMC CANNOT BE LIABLE FOR THE LOSSES


However, we cannot sustain the appellate courts finding that
SMC was likewise liable for the losses. The contention that it was
the issuance of the sailing order by SMC which was the proximate
cause of the sinking is untenable. The fact that there was an
approaching typhoon is of no moment. It appears that on one
Page 104 of 154

INSURANCE G01 CASE DIGESTS

The shipment was insured with petitioner Philippine


American General Insurance Co., Inc. (PHILAMGEN for
brevity), under Marine Open Policy No. 100367-PAG.

"MV Asilda" left the port of Zamboanga in fine weather at


eight o'clock in the evening of the same day. At around
eight forty-five the following morning, 7 July 1983, the
vessel sank in the waters of Zamboanga del Norte bringing
down her entire cargo with her including the subject 7,500
cases of 1-liter Coca-Cola softdrink bottles.

On 15 July 1983 the consignee Coca-Cola Bottlers


Philippines, Inc., Cebu plant, filed a claim with respondent
FELMAN for recovery of damages it sustained as a result of
the loss of its softdrink bottles that sank with "MV Asilda."
Respondent denied the claim thus prompting the consignee
to file an insurance claim with PHILAMGEN which paid its
claim of P755,250.00.

Claiming its right of subrogation PHILAMGEN sought


recourse against respondent FELMAN which disclaimed any
liability for the loss. Consequently, on 29 November 1983
PHILAMGEN sued the shipowner for sum of money and
damages.

127.
PHILIPPINE AMERICAN GENERAL INSURANCE V. CA, 273 SCRA
262 (1997)
G.R. No. 116940
June 11, 1997
THE PHILIPPINE AMERICAN GENERAL INSURANCE COMPANY, INC.,
petitioner,
vs.
COURT OF APPEALS and FELMAN SHIPPING LINES, respondents.
Nature:
This case deals with the liability, if any, of a shipowner for loss of
cargo due to its failure to observe the extraordinary diligence
required by Art. 1733 of the Civil Code as well as the right of the
insurer to be subrogated to the rights of the insured upon payment
of the insurance claim.

Facts:

On 6 July 1983 Coca-Cola Bottlers Philippines, Inc., loaded


on board "MV Asilda," a vessel owned and operated by
respondent Felman Shipping Lines (FELMAN for brevity),
7,500 cases of 1-liter Coca-Cola softdrink bottles to be
transported from Zamboanga City to Cebu City for
consignee Coca-Cola Bottlers Philippines, Inc., Cebu.

Issue:
Whether or not the limited liability under Art. 587 of the Code of
Commerce should apply.

Page 105 of 154

INSURANCE G01 CASE DIGESTS


from 29 November 1983, the date of judicial demand,

Held:

pursuant to Arts. 2212 and 2213 of the Civil Code.

No, Art. 587 of the Code of Commerce is not applicable to


the case at bar. Simply put, the ship agent is liable for the
negligent acts of the captain in the care of goods loaded on
the vessel. This liability however can be limited through
abandonment of the vessel, its equipment and freightage as
provided in Art. 587. Nonetheless, there are exceptional
circumstances wherein the ship agent could still be held
answerable despite the abandonment, as where the loss or
injury was due to the fault of the shipowner and the
captain.
The international rule is to the effect that the right of
abandonment of vessels, as a legal limitation of a
shipowner's liability, does not apply to cases where the
injury or average was occasioned by the shipowner's own
fault.

It must be stressed at this point that Art. 587 speaks only of


situations where the fault or negligence is committed solely
by the captain. Where the shipowner is likewise to be
blamed, Art. 587 will not apply, and such situation will be
covered by the provisions of the Civil Code on common
carrier.

WHEREFORE, the petition is GRANTED. Respondent


FELMAN SHIPPING LINES is ordered to pay petitioner
PHILIPPINE AMERICAN GENERAL INSURANCE CO., INC.,
Seven Hundred Fifty-five Thousand Two Hundred and Fifty
Pesos (P755,250.00) plus legal interest thereon counted

128.

DELSAN TRANSPORT V. CA, 369 SCRA 24 (2001)

Delsan Transport vs. Court of Appeals


G.R. No. 127897; November 15, 2001
Facts:
Caltex Philippines entered into a contract of affreightment with
the petitioner, Delsan Transport Lines, Inc. for a period of one
year whereby the petitioner agreed to transport Caltex industrial
fuel oil from Batangas refinery to different parts of the country.
Under the contract, petitioner took on board its vessel, M/T
Maysun industrial oil of Caltex to be delivered to the latters Oil
Terminal in Zamboanga City. The shipment was insured with the
private respondent, American Home Assurance Corporation
On August 14, 1986, MT Maysun set sail for Zamboanga City but
unfortunately the vessel in the early morning of August 16, 1986
near Panay Gulf. The shipment was insured with the private
respondent, American Home Assurance Corporation.
Subsequently, private respondent paid Caltex the sum of
Php.5,096,635.57.
Exercising its right of subrogation under Art. 2207, NCC, the
private respondent demanded from the petitioner the same
amount paid to Caltex. Due to its failure to collect from the
petitioner, private respondent filed a complaint with the RTC of
Page 106 of 154

INSURANCE G01 CASE DIGESTS


Makati City but the trial court dismissed the complaint, finding
the vessel to be seaworthy and that the incident was due to a
force majeure, thus exempting the petitioner from liability.
However, the decision of the trial court was reversed by the
Court of Appeals, giving credence to the report of PAGASA that
the weather was normal and that it was impossible for the vessel
to sink.
In the absence of any explanation as to what may have caused
the sinking of the vessel coupled with the finding that the same
was improperly manned, the appellate court ruled that the
petitioner is liable on its obligation as common carrier to herein
private respondent insurance company as subrogee of Caltex.

Issue: Whether or not the payment made by American Home


Assurance to Caltex for the insured value of the lost cargo
amounted to an admission that the vessel was seaworthy, thus
precluding any action of recovery against Delsan Transport?

Held: NO. Instant Petition is DENIED.


Ratio:
The payment made by the private respondent for the insured
value of the lost cargo operates as waiver of its (private
respondent) right to enforce the term of the implied warranty
against Caltex under the marine insurance policy. However, the
same cannot be validly interpreted as an automatic admission of
the vessels seaworthiness by the private respondent as to

foreclose recourse against the petitioner for any liability under


its contractual obligation as a common carrier.
The fact of payment grants the private respondent subrogatory
right which enables it to exercise legal remedies that would
otherwise be available to Caltex as owner of the lost cargo
against the petitioner common carrier.
In order to escape liability for the loss of its cargo of industrial
fuel oil belonging to Caltex, petitioner attributes the sinking of
MT Maysun to fortuitous even or force majeure. But the tale of
strong winds and big waves by Captain and the Chief Mate of the
said vessel however, was effectively rebutted and belied by the
weather report from PAGASA. Thus, as the appellate court
correctly ruled, petitioners vessel, MT Maysun, sank with its
entire cargo for the reason that it was not seaworthy. There was
no squall or bad weather or extremely poor sea condition in the
vicinity when the said vessel sank.
Neither may petitioner escape liability by presenting in
evidence certificates that tend to show that at the time of drydocking and inspection by the Philippine Coast Guard, the
vessel MT Maysun, was fit for voyage. These pieces of evidence
do not necessarily take into account the actual condition of the
vessel at the time of the commencement of the voyage. As
correctly observed by the Court of Appeals:
At the time of dry-docking and inspection, the ship may
have appeared fit. The certificates issued, however, do not
negate the presumption of unseaworthiness triggered by
an unexplained sinking. Of certificates issued in this
regard, authorities are likewise clear as to their probative
value, (thus):
Page 107 of 154

INSURANCE G01 CASE DIGESTS


failure to rebut the presumption of fault or negligence as
common carrier occasioned by the unexplained sinking of
its vessel, MT Maysun, while in transit.

Seaworthiness relates to a vessels actual


condition. Neither the granting of
classification or the issuance of certificates
established seaworthiness. (2-A Benedict on
Admiralty, 7-3, Sec. 62).
And also:
Authorities are clear that diligence in
securing certificates of seaworthiness does
not satisfy the vessel owners obligation.
Also securing the approval of the shipper of
the cargo, or his surveyor, of the condition of
the vessel or her stowage does not establish
due diligence if the vessel was in fact
unseaworthy, for the cargo owner has no
obligation in relation to seaworthiness.
(Ibid.)
Additionally, the exoneration of MT Maysuns officers and crew
by the Board of Marine Inquiry merely concerns their respective
administrative liabilities. It does not in any way operate to
absolve the petitioner common carrier from its civil liabilities. It
does not in any way operate to absolve the petitioner common
carrier from its civil liability arising from its failure to observe
extraordinary diligence in the vigilance over the goods it was
transporting and for the negligent acts or omissions of its
employees, the determination of which properly belongs to the
courts.

129.
ORIENTAL ASSURANCE V. COURT OF APPEALS, 200 SCRA 459
(1991)
Oriental Assurance vs. Court of Appeals
G.R. No. 94052 August 9, 1991
FACTS:

1. Panama Sawmill shipped 1208 pieces of apitog logs to Manila and


insured the logs with Oriental for the value of Php 1 million. Two
barges were loaded with 610 and 598 logs. At sea, typhoons
ravaged one of the barges, resulting in the loss of 497 of 598 of the
logs.
2. The Insurance contract provided for indemnity under the
following conditions:
Warranted that this Insurance is against TOTAL LOSS ONLY.
Subject to the following clauses:
Civil Code Article 1250 Waiver clause
Typhoon warranty clause

In the case at bar, petitioner is liable for the insured value of


the lost cargo of industrial fuel oil belonging to Caltex for its

Omnibus clause.

Page 108 of 154

INSURANCE G01 CASE DIGESTS


3. Oriental didnt give an indemnity because there wasnt total loss
of the shipment.
4. The sawmill filed a civil case against Oriental and the court
ordered it to pay 410,000 as value for the missing logs. The CA
affirmed the lower court judgment but reduced the legal interest.
Hence this appeal by Oriental.

ISSUE:
WON Oriental Assurance can be held liable under its marine
insurance policy based on the theory of a divisible contract of
insurance and, consequently, a constructive total loss? NO.
HELD:
Perla v CA- The terms of the contract constitute the measure of the
insurer liability and compliance therewith is a condition precedent
to the insured's right to recovery from the insurer.
Whether a contract is entire or severable is a question of intention
to be determined by the language employed by the parties. The
policy in question shows that the subject matter insured was the
entire shipment of 2,000 cubic meters of apitong logs. The fact that
the logs were loaded on two different barges did not make the
contract several and divisible as to the items insured. The logs on
the two barges were not separately valued or separately insured.
Only one premium was paid for the entire shipment, making for
only one cause or consideration. The insurance contract must,
therefore, be considered indivisible.

Also, the insurer's liability was for "total loss only" as stipulated. A
total loss may be either actual or constructive. An actual total loss
under Sec 130 of the Insurance Code is caused by:
(a) A total destruction of the thing insured;
(b) The irretrievable loss of the thing by sinking, or by being broken
up;
(c) Any damage to the thing which renders it valueless to the owner
for the purpose for which he held it; or
(d) Any other event which effectively deprives the owner of the
possession, at the port of destination, of the thing insured.

A constructive total loss, gives to a person insured a right to


abandon and it means:
SECTION 139. A person insured by a contract of marine insurance
may abandon the thing insured, or any particular portion thereof
separately valued by the policy, or otherwise separately insured,
and recover for a total loss thereof, when the cause of the loss is a
peril injured against,
(a) If more than three-fourths thereof in value is actually lost, or
would have to be expended to recover it from the peril;
(b) If it is injured to such an extent as to reduce its value more than
three-fourths

Page 109 of 154

INSURANCE G01 CASE DIGESTS


The appellate court considered the cargo in one barge as separate
from the other and ruled that 497 of 598 was more than of the
amount lost, showing a constructive total loss.

130.
PHILIPPINE HOME ASSURANCE V. COURT OF APPEALS, 257
SCRA 468 (1996)
G.R. No. 106999, June 26, 1996
FACTS:

The SC, however, said that although the logs were placed in two
barges, they were not separately valued by the policy, nor
separately insured. Of the entirety of 1,208, pieces of logs, only 497
pieces thereof were lost or 41.45% of the entire shipment. Since the
cost of those 497 pieces does not exceed 75% of the value of all
1,208 pieces of logs, the shipment can not be said to have sustained
a constructive total loss under Section 139(a) of the Insurance Code.

Eastern Shipping Lines, Inc. (ESLI) loaded on board SS


Eastern Explorer in Japan shipment of carriage to Mania and
Cebu consigned to the ff: (1) William Lines, Inc. - engine
parts; (2) Orca's Company - ammonium chloride; (3) Pan
Oriental Match Company - bags of Glue; (4) Ding Velayo garments.
While the vessel was off Okinawa, a small flame was
detected on the acetylene cylinder
located in the
accommodation area near the engine room on the main
deck level. The acetylene cylinder suddenly exploded
causing death and severe injuries to the crew and instantly
setting fire to the whole vessel, which forced the crew to
abandon the ship.
The cargoes which were saved were loaded to another
vessel for delivery to their original ports of destination. ESLI
charged the consignees several amounts corresponding to
additional freight and salvage charges.
The charges were all paid by Philippine Home Assurance
Corporation (PHAC) under protest for and in behalf of the
consignees. Thereafter, as subrogee of the consignees, filed
a complaint against ELSI to recover the sum paid under
protest on the ground that the same were actually damage
directly brought by the fault and negligence of ESLI.
RTC & CA: Dismissed the complaint and ruled in favor of
ESLI.

ISSUE:
Page 110 of 154

INSURANCE G01 CASE DIGESTS


(1) Who among the carrier, consignee or insurer of the goods is
liable for the additional charges or expenses incurred? Carrier.
(2)Whether or not the respondent court committed an error in
concluding that the expenses incurred in saving the cargo are
considered general average? Yes.

Consequently, respondent ESLI's claim for the contribution


from the consignees at the time of the occurrence of the
average turns to naught.
The cargo consignees cannot be made liable to respondent carrier
thus respondent carrier must refund petitioner the amount it paid
under protest for additional freight and salvage charges in behalf of
the consignees.

RATIO:
(1)

Fire may not be considered a natural disaster or calamity


since it almost always arise from some act of men. In this
case, it is not disputed that a small flame was detected on
the acetylene cylinder and that by reason thereof the same
exploded despite efforts to extinguish the fire. Moreover,
the acetylene cylinder, obviously full loaded, was stored in
the accommodation area near the engine room and not in a
storage area considerably far and safe distance from the
engine room. Therefore, there is a strong evidence that the
acetylene cylinder caught fire because of the fault and
negligence of respondent ESLI, its captain and its crew.

As a rule, general or gross averages include all damages and


expenses which are deliberately caused in order to save the
vessel, its cargo, or both at the same time, from a real and
known risk. While this case may technically fall within the
purview of the said provision, the formalities prescribed
under Art. 813 and 814 of the Code of Commerce in order
to incur the expenses and cause the damage corresponding
to gross average were not complied with.

(2)

Page 111 of 154

INSURANCE G01 CASE DIGESTS

FIRE INSURANCE
131. PHILIPPINE HOME ASSURANCE V. COURT OF APPEALS, 257 SCRA
468 (1996)
FIRE INSURANCE
Phil Home Assurance v. CA

Phil Home paid under protest, and as subrogee of the


consignee, filed a complaint to recover the sum paid under protest
on the ground that the damages were the fault, negligence of ESLI
ISSUE

FACTS

WON the burning of the ship is a natural disaster or


calamity which would absolve ESLI from liability? NO

ESLI (respondent) loaded to a ship certain goods like engine


parts, mamonium chloride, and glue 300. .

HELD

While the vessel was off Okinawa, Japan, a small flame was
detected on the acetylene cylinder located in the accommodation
area near the engine room on the main deck level. As the crew was
trying to extinguish the fire, the acetylene cylinder suddenly
exploded sending a flash of flame throughout the accommodation
area, thus causing death and severe injuries to the crew and
instantly setting fire to the whole superstructure of the vessel. The
incident forced the master and the crew to abandon the ship.
After the fire was extinguished, the cargoes which were
saved were loaded to another vessel for delivery to their original
ports of destination.
ESLI charged the consignees additional charges or expenses
incurred by the owner of the ship in the salvage operations and in
the transshipment of the goods via a different carrier.

In our jurisprudence, fire may not be considered a natural


disaster or calamity since it almost always arises from some act of
man or by human means.
It cannot be an act of God unless caused by lightning or a
natural disaster or casualty not attributable to human agency.
In the case at bar, it is not disputed that a small flame was
detected on the acetylene cylinder and that by reason thereof, the
same exploded despite efforts to extinguish the fire. Neither is
there any doubt that the acetylene cylinder, obviously fully loaded,
was stored in the accommodation area near the engine room and
not in a storage area considerably far, and in a safe distance, from
the engine room. Moreover, there was no showing, and none was
alleged by the parties, that the fire was caused by a natural disaster
or calamity not attributable to human agency. On the contrary,
there is strong evidence indicating that the acetylene cylinder
caught fire because of the fault and negligence of respondent ESLI,
its captain and its crew.
Page 112 of 154

INSURANCE G01 CASE DIGESTS

the fact that the acetylene cylinder was checked, tested and
examined and subsequently certified as having complied with the
safety measures and standards by qualified experts before it was
loaded in the vessel only shows to a great extent that negligence
was present in the handling of the acetylene cylinder after it was
loaded and while it was on board the ship. Indeed, had the
respondent and its agents not been negligent in storing the
acetylene cylinder near the engine room, then the same would not
have leaked and exploded during the voyage
There is no merit in the finding of the trial court to which
respondent court erroneously agreed that the fire was not the
fault or negligence of respondent but a natural disaster or
calamity. The records are simply wanting in this regard.

WHEREFORE, the judgment appealed from is hereby REVERSED and


SET ASIDE. Respondent Eastern Shipping Lines, Inc. is ORDERED to
return to petitioner Philippine Home Assurance Corporation the
amount it paid under protest in behalf of the consignees herein.

132. BACHRACH V. BRITISH AMERICAN ASSURANCE , 17 PHIL. 35


(1910)
E. M. BACHRACH vs. BRITISH AMERICAN ASSURANCE
COMPANY
G.R. No. L-5715 December 20, 1910
JOHNSON, J.:
FACTS:
1. E. M. Bachrach insured against loss or damage by fire,
goods, belonging to its general furniture store, such as iron
and brass bedsteads, toilet tables, chairs, ice boxes,
bureaus, washstands, mirrors, and sea-grass furniture from
British American Assurance Company through it's agent.
2. In the policy, Co insurance was allowed which should be
declared in the event of loss or claim. Also, such policy
provided Permission granted for the use of gasoline not to
exceed 10 gallons for the "Calalac" automobile, but only
whilst contained in the reservoir of the car.
3. Fire broke out. British American Assurance Company
denied the claim of Bachrach alleging the ff:
a. That the plaintiff maintained a paint and varnish shop in
the said building where the goods which were insured were
stored.
b. That the plaintiff transferred his interest in and to the
property covered by the policy to H. W. Peabody & Co. to
secure certain indebtedness due and owing to said
company, and also that the plaintiff had transferred his
interest in certain of the goods covered by the said policy to
one Macke, to secure certain obligations assumed by the
Page 113 of 154

INSURANCE G01 CASE DIGESTS


said Macke for and on behalf of the insured. That the
sanction of the said defendant had not been obtained by
the plaintiff, as required by the said policy.
c. That the plaintiff, immediately preceding the outbreak of
the alleged fire, willfully placed a gasoline can containing 10
gallons of gasoline in the upper story of said building in
close proximity to a portion of said goods, wares, and
merchandise, which can was so placed by the plaintiff as to
permit the gasoline to run on the floor of said second story,
and after so placing said gasoline, he, the plaintiff, placed in
close proximity to said escaping gasoline a lighted lamp
containing alcohol, thereby greatly increasing the risk of
fire.
4. The lower court ruled in favor of Bachrach making British
American Assurance liable.
ISSUE:
WoN British American Assurance liable of such claim.
HELD:YES.
RATIO DECIDENDI:
- Keeping of inflammable oils on the premises, though
prohibited by the policy, does not void it if such keeping is
incidental to the business. It may be added also that there
was no provision in the policy prohibiting the keeping of
paints and varnishes upon the premises where the insured
property was stored. If the company intended to rely upon
a condition of that character, it ought to have been plainly
expressed in the policy.

by the defendant to the plaintiff, it will be noted that there


is no provision in said policy prohibiting the plaintiff from
placing a mortgage upon the property insured. It is claimed
that the execution of a chattel mortgage on the insured
property violated what is known as the "alienation clause,"
which is now found in most policies, and which is expressed
in the policies involved in cases 6496 and 6497 by a
purchase imposing forfeiture if the interest in the property
pass from the insured. (Cases 6496 and 6497, in which are
involved other action against other insurance companies for
the same loss as in the present action.)
*alienation clause - forfeiture if the interest in the property
pass from the insured
- there is no alienation within the meaning of the insurance
law until the mortgage acquires a right to take possession
by default under the terms of the mortgage. No such right is
claimed to have accrued in the case at bar, and the
alienation clause is therefore inapplicable.
- This clause has been the subject of a vast number of
judicial decisions (13 Am. & Eng. Encyc. of Law, 2d ed., pp.
239 et seq.), and it is held by the great weight of authority
that the interest in property insured does not pass by the
mere execution of a chattel mortgage and that while a
chattel mortgage is a conditional sale, there is no alienation
within the meaning of the insurance law until the mortgage
acquires a right to take possession by default under the
terms of the mortgage. No such right is claimed to have
accrued in the case at bar, and the alienation clause is
therefore inapplicable.

- Furthermore, upon reading the policy of insurance issued


Page 114 of 154

INSURANCE G01 CASE DIGESTS


o
133. TAN CHUCO V. YORKSHIRE F IRE AND LIFE INSURANCE , 14 PHIL.
346 (1909)
Tan Chuco vs Yorkshire Fire and Life Insurance Company
Facts:

o
Tan Chuco files a claim under an open fire insurance policy
for the alleged loss by fire of certain stock of goods insured
by Yorkshire.
CFI: Evidence did not sustain Yorkshires allegation that Tan
Chuco or his agents had intentionally and fraudulently set
the building on fire
o But was of the opinion that the Tan Chuco failed to
establish the value of the goods he alleges were
destroyed by the fire.
o He submitted fabricated written evidence and false
testimony in support of his claim that the insured
goods actually destroyed were worth more than the
total amount of the insurance thereon.
o CFI was of the opinion that the submitted inventory
was not genuine and was fraudulently prepared.
o Tan Chucos representatives and employees who
were in the building when the fire took place, not
only made no effort to extinguish the fire, or to save
the goods from destruction, but also failed to save
any of the books or papers connected with the
business of which he was in charge ofthose could
have corroborated with the data in the alleged
inventory
o The inventory submitted was dated January 1, not
of custom to Tan Chuco who were of Chinese
decent.

No explanation was offered which would account


for the remarkable conduct of Tan Chucos manager
in preparing an inventory two months after his
employer had left for China and then instead of
forwarding such inventory to his principal by mail,
entrusted it for transmission to a friend who had
not even left for China when the fire took place.
Indication that Tan Chuco had been experiencing
adverse business conditions before the fire

Issue: Whether or not Tan Chuco may claim under the fire
insurance policy?

Held: NO.
We think that the action of the trial court in rejecting the proof
offered by Tan Chuco as to the amount of the loss must be
sustained.

The contract of fire insurance being a contract of indemnity, Tan


Chuco is only entitled to recover the amount of actual loss sustained
by him. There being no express valuation in the policy, the
judgment was properly entered against him for lack of satisfactory
proof of the amount of loss.
Rule: In the absence of express valuation in a fire insurance policy,
the insured is only entitled to recover the amount of actual loss
sustained and the burden is upon him to establish such amount.

Page 115 of 154

INSURANCE G01 CASE DIGESTS


134. MALAYAN INSURANCE V. CRUZ-ARNALDO, 154 SCRA 672
(1987)

G.R. No. L-67835 October 12, 1987


Lessons Applicable: Authority to Receive Payment/Effect of
Payment (Insurance)
Laws Applicable: Article 64, Article 65, Section 77, Section 306 of the
Insurance Code

FACTS:

June 7, 1981: Malayan insurance co., inc. (MICO) issued


to Coronacion Pinca, Fire Insurance Policy forher
property effective July 22, 1981, until July 22, 1982

October 15,1981: MICO allegedly cancelled the policy for non-

Under Section 416 of the Insurance Code, the period for appeal
is thirty days from notice of the decision of the Insurance
Commission. The petitioner filed its motion for reconsideration
on April 25, 1981, or fifteen days such notice, and the
reglementary period began to run again after June 13, 1981,
date of its receipt of notice of the denial of the said motion for
reconsideration. As the herein petition was filed on July 2, 1981,
or nineteen days later, there is no question that it is tardy by

four days.
Insurance Commission: favored Pinca
MICO appealed
ISSUE: W/N MICO should be liable because its agent Adora was
authorized to receive it

HELD: YES. petition is DENIED

payment, of the premium and sent the corresponding notice to


Pinca
December 24, 1981: payment of the premium for Pinca was
received by Domingo Adora, agent of MICO
January 15, 1982: Adora remitted this payment to
MICO,together with other payments
January 18, 1982: Pinca's property was completely burned
February 5, 1982: Pinca's payment was returned by MICO to
Adora on the ground that her policy had been cancelled earlier
but Adora refused to accept it and instead demanded for
payment

SEC. 77. An insurer is entitled to payment of the premium as


soon as the thing is exposed to the peril insured against.
Notwithstanding any agreement to the contrary, no policy or
contract of insurance issued by an insurance company is valid
and binding unless and until the premium thereof has been
paid, except in the case of a life or an industrial life
policy whenever the grace period provision applies.
SEC. 306. xxx xxx xxx

Any insurance company which delivers to an insurance agant


or insurance broker a policy or contract of insurance shall be
demmed to have authorized such agent or broker to receive on its
Page 116 of 154

INSURANCE G01 CASE DIGESTS


behalf payment of any premium which is due on such policy or
contract of insurance at the time of its issuance or delivery or which
becomes due thereon.

Payment to an agent having authority to receive or collect


payment is equivalent to payment to the principal himself; such
payment is complete when the money delivered is into the
agent's hands and is a discharge of the indebtedness owing to
the principal.

SEC. 64. No policy of insurance other than life shall be


cancelled by the insurer except upon prior notice thereof to the
insured, and no notice of cancellation shall be effective unless it
is based on the occurrence, after the effective date of the
policy, of one or more of the following:

(a) non-payment of premium;


(b) conviction of a crime arising out of acts increasing the hazard
insured against;
(c) discovery of fraud or material misrepresentation;
(d) discovery of willful, or reckless acts or commissions increasing
the hazard insured against;

As for the method of cancellation, Section 65 provides as follows:

A valid cancellation must, therefore, require concurrence of the


following conditions:
(1) There must be prior notice of cancellation to the insured;
(2) The notice must be based on the occurrence, after the
effective date of the policy, of one or more of the grounds
mentioned;
(3) The notice must be (a) in writing, (b) mailed, or delivered to
the named insured, (c) at the address shown in the policy;
(4) It must state (a) which of the grounds mentioned in Section 64
is relied upon and (b) that upon written request of the insured, the
insurer will furnish the facts on which the cancellation is based.

(e) physical changes in the property insured which result in the


property becoming uninsurable;or
(f) a determination by the Commissioner that the continuation of
the policy would violate or would place the insurer in violation of
this Code.

SEC. 65. All notices of cancellation mentioned in the preceding


section shall be in writing, mailed or delivered to the named
insured at the address shown in the policy, and shall state (a)
which of the grounds set forth in section sixty-four is relied
upon and (b) that, upon written request of the named insured,
the insurer will furnish the facts on which the cancellation is
based.

All MICO's offers to show that the cancellation was


communicated to the insured is its employee's testimony that
the said cancellation was sent "by mail through our mailing
section." without more
It stands to reason that if Pinca had really received the said
notice, she would not have made payment on the original policy
on December 24, 1981. Instead, she would have asked for a
Page 117 of 154

INSURANCE G01 CASE DIGESTS

new insurance, effective on that date and until one year later,
and so taken advantage of the extended period.
Incidentally, Adora had not been informed of the cancellation
either and saw no reason not to accept the said payment
Although Pinca's payment was remitted to MICO's by its agent
on January 15, 1982, MICO sought to return it to Adora only on
February 5, 1982, after it presumably had learned of the
occurrence of the loss insured against on January 18, 1982
make the motives of MICO highly suspicious

Page 118 of 154

INSURANCE G01 CASE DIGESTS

CASUALTY INSURANCE
135.

GUINGON V. DEL MONTE, 20 SCRA 1043 (1967)


Guingon v. Del Monte
20 SCRA 1043 (1967)
Casualty Insurance

FACTS:
1. Julio Aguilar owner and operator of several jeepneys
insured them with Capital Insurance & Surety Co., Inc.
2. February 20, 1961: Along the intersection of Juan Luna and
Moro streets, City of Manila, the jeepneys operated by
Aguilar driven by Iluminado del Monte and Gervacio
Guingon bumped and Guingon died some days after
3. Iluminado del Monte was charged with homicide thru
reckless imprudence and was penalized 4 months
imprisonment
4. The heirs of Gervacio Guingon filed an action for damages
praying that P82,771.80 be paid to them jointly and
severally by the driver del Monte, owner and operator
Aguilar, and the Capital Insurance & Surety Co., Inc.
5. CFI: Iluminado del Monte and Julio Aguilar jointly and
severally to pay plaintiffs the sum of P8,572.95 as damages
for the death of their father, plus P1,000.00 for attorney's
fees plus costs
6. Capital Insurance and Surety Co., Inc. is hereby sentenced
to pay P5,000 plus P500 as attorney's fees and costs to be
applied in partial satisfaction of the judgment rendered
against Iluminado del Monte and Julio Aguilar in this case
ISSUE:

2. W/N the heirs can sue the insurer and insured jointly? - YES

RATIO: Affirmed in toto.


1. YES

policy: the insurer agreed to indemnify the insured "against all


sums . . . which the Insured shall become legally liable to pay in
respect of: a. death of or bodily injury to any person . . . ." indemnity against liability
TEST: Where the contract provides for indemnity
against liability to third persons, then third persons to whom
the insured is liable, CAN sue the insurer. Where the contract is
for indemnity against actual loss or payment, then third persons
CANNOT proceed against the insurer, the contract being solely
to reimburse the insured for liability actually discharged by him
thru payment to third persons, said third persons' recourse
being thus limited to the insured alone.

2. YES

policy: expressly disallows suing the insurer as a co-defendant


of the insured in a suit to determine the latter's liability
no action close: suit and final judgment be first obtained against
the insured; that only "thereafter" can the person injured
recover on the policy
Sec. 5 of Rule 2 on "Joinder of causes of action" and Sec. 6 of
Rule 3 on "Permissive joinder of parties" cannot be
superseded, at least with respect to third persons not a party to
the contract, as herein, by a "no action" clause in the contract
of insurance.

1. W/N there a stipulation pour autriu to enable that will enable


the heirs to sue against Capital Insurance and Surety Co., Inc.? - YES
Page 119 of 154

INSURANCE G01 CASE DIGESTS


136.

DE LA CRUZ V. CAPITAL INSURANCE , 17 SCRA 559 (1966)

INSURANCE: CASUALTY INSURANCE

HELD: YES

SIMON DE LA CRUZ vs CAPITAL INSURANCE and SURETY Co., Inc.


FACTS:

Eduardo de la Cruz, a mucker in Itogon-Suyoc Mines, Inc.


was the holder of an accident insurance policy underwritten
by Capital Insurance for the period of November 13, 1956 November 12, 1957.
Itugon sponsored a boxing contest as part of their
celebration of New Year. Eduardo participated as a nonprofessional boxer. In the course of his bout with another
person, another non-professional, of the same height,
weight, and size, Eduardo slipped and was hit by his
opponent on the left part of the back of his head. This
caused Eduardo to fall with his head hitting the rope of the
ring. He died the following day. The cause of his death was
hemmorhage, intracranial, left.
The father, Simon, was named beneficiary and filed a claim
with the insurance company. Insurer denied. CFI (Specific
Performance) ruled in favour of Simon.
Insurer contends that while the death of the insured was
due to head injury, said injury was sustained by his
voluntary participation in the contest. He cannot be
considered to have met his death by accidental means. In
other words what is required to be accidental is the means
that caused the death and not the death itself.

ISSUE: WON Eduardos death is an accident within the meaning of


the policy despite the fact that he voluntarily participated in the
boxing contest.

The terms "accident" and "accidental", as used in insurance


contracts, have not acquired any technical meaning, and are
construed by the courts in their ordinary and common
acceptation. Thus, the terms have been taken to mean that
which happen by chance or fortuitously, without intention
and design, and which is unexpected, unusual, and
unforeseen. An accident is an event that takes place
without one's foresight or expectation an event that
proceeds from an unknown cause, or is an unusual effect of
a known cause and, therefore, not expected.
Even if we take appellant's theory, the death of the insured
in the case at bar would still be entitled to indemnification
under the policy. The generally accepted rule is that, death
or injury does not result from accident or accidental means
within
the
terms
of
an
accident-policy if it is the natural result of the insured's
voluntary act, unaccompanied by anything unforeseen
except the death or injury. There is no accident when a
deliberate act is performed unless some additional,
unexpected, independent, and unforeseen happening
occurs which produces or brings about the result of injury
or death. In other words, where the death or injury is not
the natural or probable result of the insured's voluntary
act, or if something unforeseen occurs in the doing of the
act which produces the injury, the resulting death is within
the protection of policies insuring against death or injury
from accident.
In the present case, while the participation of the insured in
the boxing contest is voluntary, the injury was sustained
when he slid, giving occasion to the infliction by his
opponent of the blow that threw him to the ropes of the
ring. Without this unfortunate incident, that is, the
unintentional slipping of the deceased, perhaps he could
Page 120 of 154

INSURANCE G01 CASE DIGESTS

not have received that blow in the head and would not have
died. The fact that boxing is attended with some risks of
external injuries does not make any injuries received in the
course of the game not accidental. In boxing as in other
equally physically rigorous sports, such as basketball or
baseball, death is not ordinarily anticipated to result. If,
therefore, it ever does, the injury or death can only be
accidental or produced by some unforeseen happening or
event as what occurred in this case.
As to the policy provision enumerating sports events which
are excluded from coverage, death or disablement resulting
from engagement in boxing contests was not declared
outside of the protection of the insurance contract. Failure
of the defendant insurance company to include death
resulting from a boxing match or other sports among the
prohibitive risks leads inevitably to the conclusion that it did
not intend to limit or exempt itself from liability for such
death.

137. PAN MALAYAN V. COURT OF APPEALS, 184 SCRA 54 (1990)


PAN MALAYAN INSURANCE vs CA
GR 81026, 184 SCRA 54; April 3, 1990
Topic: Casualty Insurance: Accident and health insurance
Facts:
Pan Malayan insured a Lancer car registered in the name of
Canlubang Automotive Resources. On May 26, 1985 the car was hit
by a pick-up truck due to the carelessness, recklessness and
imprudence of its driver. The insured car suffered damages, whose
repairs Pan Malayan paid for. Thus it was subrogated in the rights of
the owner against the pick-up drivers employer, the private
respondent Erlinda Fabie. Despite demands, Fabie refused and
failed to pay.
Pan Malayan alleged that the damage to the insured car
was covered under the own damage coverage of the policy, and
that its driver was authorized at the time of the accident. The
respondents argued that payment under the "own damage" clause
of the insurance policy precluded subrogation under Article 2207 of
the Civil Code, since indemnification there was made on the
assumption that there was no wrongdoer or no third party at fault.
Pan Malayan alleged in its complaint that, pursuant to a
motor vehicle insurance policy, it had indemnified Canlubang for
the damage to the insured car resulting from a traffic accident
allegedly caused by the negligence of the driver of private
respondent, Erlinda Fabie. It thus claims that its cause of action
against private respondents was anchored upon Article 2207 of the
Civil Code.
The RTC dismissed Pan Malayans complaint. The CA
affirmed.
Issue:

Page 121 of 154

INSURANCE G01 CASE DIGESTS


1. Was the insurers indemnity under the own damage
clause of its policy an admission that its insured/ client
caused the damage?
2. Given this admission, was the insurer subrogated?
Held:
1. NO
2. YES
Rationale: (Only #1)
It must be emphasized that the lower court's ruling that the "own
damage" coverage under the policy impliesdamage to the insured
car caused by the assured itself, instead of third parties, proceeds
from an incorrect comprehension of the phrase "own damage" as
used by the insurer. When PANMALAY utilized the phrase "own
damage" a phrase which, incidentally, is not found in the
insurance policy to define the basis for its settlement of
CANLUBANG's claim under the policy, it simply meant that it had
assumed to reimburse the costs for repairing the damage to the
insured vehicle. It is in this sense that the so-called "own damage"
coverage under Section III of the insurance policy is differentiated
from Sections I and IV-1 which refer to "Third Party Liability"
coverage (liabilities arising from the death of, or bodily injuries
suffered by, third parties) and from Section IV-2 which refer to
"Property Damage" coverage (liabilities arising from damage caused
by the insured vehicle to the properties of third parties).
Neither is there merit in the Court of Appeals' ruling that
the coverage of insured risks under Section III-1 of the
policy does not include to the insured vehicle arising from
collision or overturning due to the negligent acts of the
third party. Not only does it stem from an erroneous
interpretation of the provisions of the section, but it also
violates a fundamental rule on the interpretation of
property insurance contracts.

1. It is a basic rule in the interpretation of contracts that the


terms of a contract are to be construed according to the
sense and meaning of the terms which the parties
thereto have used. In the case of property insurance
policies, the evident intention of the contracting
parties, i.e., the insurer and the assured, determine the
import of the various terms and provisions embodied in the
policy. It is only when the terms of the policy are
ambiguous, equivocal or uncertain, such that the parties
themselves disagree about the meaning of particular
provisions, that the courts will intervene. In such an event,
the policy will be construed by the courts liberally in favor
of the assured and strictly against the insurer.
Section III-1 of the insurance policy which refers to the conditions
under which the insurer PANMALAY is liable to indemnify the
assured CANLUBANG against damage to or loss of the insured
vehicle, reads as follows:
SECTION III LOSS OR DAMAGE
1. The Company will, subject to the Limits of
Liability, indemnify the Insured against loss of or
damage to the Scheduled Vehicle and its
accessories and spare parts whilst thereon:
(a) by accidental collision or
overturning, or collision or
overturning consequent upon
mechanical breakdown or
consequent upon wear and tear;
(b) by fire, external explosion, self
ignition or lightning or burglary,
housebreaking or theft;
Page 122 of 154

INSURANCE G01 CASE DIGESTS


(c) by malicious act;
(d) whilst in transit (including the
processes of loading and unloading)
incidental to such transit by road,
rail, inland, waterway, lift or
elevator.
It cannot be said that the meaning given by PANMALAY and
CANLUBANG to the phrase "by accidental collision or overturning"
found in the first paint of sub-paragraph (a) is untenable. Although
the terms "accident" or "accidental" as used in insurance contracts
have not acquired a technical meaning, the Court has on several
occasions defined these terms to mean that which takes place
"without one's foresight or expectation, an event that proceeds
from an unknown cause, or is an unusual effect of a known cause
and, therefore, not expected". Certainly, it cannot be inferred from
jurisprudence that these terms, without qualification, exclude
events resulting in damage or loss due to the fault, recklessness or
negligence of third parties. The concept "accident" is not necessarily
synonymous with the concept of "no fault". It may be utilized simply
to distinguish intentional or malicious acts from negligent or
careless acts of man.
Moreover, a perusal of the provisions of the insurance policy reveals
that damage to, or loss of, the insured vehicle due to negligent or
careless acts of third parties is not listed under the general and
specific exceptions to the coverage of insured risks which are
enumerated in detail in the insurance policy itself.

138.

SUN INSURANCE V. COURT OF APPEALS, 211 SCRA 554 (1990)

SUN INSURANCE OFFICE, LTD., petitioner, vs.


THE HON. COURT OF APPEALS and NERISSA LIM, respondents.
CRUZ, J.:
July 17, 1992
Facts
1. Sun life issued a personal accident policy to Felix Lim.
2. 2 months later, he died. His wife as the beneficiary sought
payment of the policy but was rejected.
a. However, despite the rejection of the claim, Sun life
found that there was neither suicide nor an
accident.
3. The eyewitness to his death, his secretary, testified that he
was playing with his handgun from which he had previously
removed his magazine. He pointed it to his temple and
then, an explosion occurred. Lim fell on the floor dead.
4. RTC ruled in favor of the wife. Sun life should pay the policy
plus damages.
5. CA affirmed.
Issue
Is the death of Lim an accident in which the beneficiary may claim
payment of the policy? YES
(To restate: Does Lims death fall under the exceptions in which the
insurer would not be liable? NO)
Page 123 of 154

INSURANCE G01 CASE DIGESTS


Ruling:

The Court believes that Lims death was an ACCIDENT.


o The term, when used in an insurance contract, is to
be construed and considered according to the
ordinary understanding and common usage and
speech of people generally.
o An accident is an event which happens without any
human agency or, if happening through human
agency, an event which, under the circumstances, is
unusual to and not expected by the person to
whom it happens. It has also been defined as an
injury which happens by reason of some violence
or CASUALTY to the injured without his design,
consent, or voluntary co-operation.
o As the secretary testified, Lim had removed the
magazine from the gun and believed it was no
longer dangerous. He expressly assured her that the
gun was not loaded. It is submitted that Lim did not
willfully expose himself to needless peril when he
pointed the gun to his temple because the fact is
that he thought it was not unsafe to do so. The act
was precisely intended to assure Nalagon that the
gun was indeed harmless.
o Lim did not know that the gun he placed to his head
was loaded.
Lim was unquestionably negligent and that negligence cost
him his own life. But it should not prevent his widow from
recovering from the insurance policy he obtained precisely
against accident. There is nothing in the policy that relieves
the insurer of the responsibility to pay the indemnity

agreed upon if the insured is shown to have contributed to


his own accident. Indeed, most accidents are caused by
negligence. There are only four exceptions expressly made
in the contract to relieve the insurer from liability, and none
of these exceptions is applicable in the case at bar. **
** Exceptions
The Company shall not be liable in respect of
1. bodily injury
a) sustained
i) while the Insured Person is engaging in (or practicing
for or taking part in training peculiar to) any of the
Excluded Activities.
ii) by any person before such person attains the Lower
Age Limit or after the expiry of the Period of Insurance
during which such person attains the Upper Age Limit.
b) consequent upon
i) the Insured Person committing or attempting to
commit suicide or wilfully exposing himself to needless
peril except in an attempt to save human life.
ii) war, invasion, act of foreign enemy, hostilities
(whether war be declared or not) civil war, rebellion,
revolution, insurrection, or military or usurped power.
2. bodily injury or Death Disablement or Medical
Expenses consequent upon or contributed to by the
Insured Person
a) having taken a drug unless the Insured proves that the
drug was taken in accordance with proper medical
prescription and directions and not for treatment of drug
addiction.
b) suffering from pre-existing physical or mental defect
or infirmity which had not been declared to and accepted
in writing by the Company.
3. Death Disablement or Medical Expenses consequent
upon or contributed to by the Insured Person being
pregnant or suffering from sickness or disease not
resulting from bodily injury or suffering from bodily
injury due to a gradually operating cause.

Page 124 of 154

INSURANCE G01 CASE DIGESTS


4. Risks of Murder and Assault.

The award of damages is deleted since there is no evidence


to show that there was any wrongful act done by the
insurer, Sun Life.

SC affirms CA.
139.

BIAGTAN V. INSULAR LIFE, 144 SCRA 58 (1972)

Biagtan vs Insular Life (1972)


FACTS
Juan Biagtan was insured with Insular for P5k with a supplementary
contract Accidental Death Benefit clause for another P5k if "the
death of the Insured resulted directly from bodily injury effected
solely through external and violent means sustained in an accident .
. . and independently of all other causes." The clause, however,
expressly provided that it would not apply where death resulted
from an injury "intentionally inflicted by a third party."
One night, robbers entered their house. Juan was stabbed 9 times
with a knife and then died. The robbers were convicted of robbery
with homicide.
The family now claims for the additional P5k from Insular, under the
Accidental Death Benefit clause. Insular refused saying that the
death resulted from injuries intentionally inflicted by 3rd parties
and was therefore not covered. The family sued.
CFI ruled in favor of the family.

Were the injuries sustained by the deceased intentionally


afflicted, thus not covered by the accidental death benefit
clause?
HELD
Yes. Whatever the true intent of the robbers may be, the act itself
of inflicting the injuries was intentional. The exception clause does
not speak of the purpose of the one causing the injury, only of the
fact that such injuries have been "intentionally" inflicted, to
distinguish them from injuries which, although received at the
hands of a third party, are purely accidental. Examples would be
when a gun which discharges while being cleaned and kills a
bystander; or a hunter who shoots at his prey and hits a person
instead; or an athlete in a competitive game involving physical
effort who collides with an opponent and fatally injures him as a
result.
In Calanoc vs. CA, where a shot was fired and it turned out
afterwards that the watchman was hit in the abdomen, the wound
causing his death, the Court held that it could not be said that the
killing was intentional for there was the possibility that the
malefactor had fired the shot to scare the people around for his
own protection and not necessarily to kill or hit the victim.
It is not the same with the case at bar, for while a single shot fired
from a distance, and by a person who was not even seen aiming at
the victim, could indeed have been fired without the intent to kill or
injure, nine wounds inflicted with bladed weapons at close range
cannot conceivably be considered as innocent insofar as such intent
is concerned.

ISSUE
Page 125 of 154

INSURANCE G01 CASE DIGESTS


Insular won and the family cannot claim the additional 5k under the
ADB clause because the injury was intentionally caused by a person,
i.e. not accidental.

Justice Teehankee dissents, saying that Calanoc v. CA is controlling


because the insurance company was unable to prove that the killing
was intentional. Burden of proof is with the insurance company.
Insurance, being contracts of adhesion, must be construed strictly
against insurance company in cases of ambiguity. The
supplementary contract enumerated exceptions. The only exception
which is not susceptible of classification is that provided in
paragraph 5(e), the very exception which is currently the subject of
contention, which would also except injuries "inflicted intentionally
by a third party, either with or without provocation on the part of
the insured, and whether or not the attack or the defense by the
third party was caused by a violation of the law by the insured." The
ambiguous clause conflicts with all the other four exceptions in the
same paragraph 5 particularly that immediately preceding it in item
(d) which excepts injuries received where the insured has violated
the law or provoked the injury, while this clause, construed as the
insurance company now claims, would seemingly except also all
other injuries, intentionally inflicted by a third party, regardless of
any violation of law or provocation by the insured, and defeat the
very purpose of the policy of giving the insured double indemnity in
case of accidental death by "external and violent means" in the
very language of the policy.' It is obvious from the very classification
of the exceptions and applying the rule of noscitus a sociis, that the
double-indemnity policy covers the insured against accidental
death, whether caused by fault, negligence or intent of a third party
which is unforeseen and unexpected by the insured. All the

associated words and concepts in the policy plainly exclude the


accidental death from the coverage of the policy only where the
injuries are self-inflicted or attended by some proscribed act of the
insured or are incurred in some expressly excluded calamity such as
riot, war or atomic explosion.- The untenability of insurer's claim
that the insured's death fell within the exception is further
heightened by the stipulated fact that two other insurance
companies which likewise covered the insured for much larger sums
under similar accidental death benefit clauses promptly paid the
benefits thereof to plaintiffs beneficiaries.

140.

CALANOC V. COURT OF APPEALS, 98 PHIL. 79 (1955)

Calanoc v CA
Facts:
The circumstances surrounding the death of Melencio Basilio show
that when he was killed at about seven oclock in the night of
January 25, 1951, he was on duty as watchman of the Manila Auto
Supply at the corner of Avenida Rizal and Zurbaran; that it turned
out that Atty. Antonio Ojeda who had his residence at the corner of
Zurbaran and Oroquieta, a block away from Basilios station, had
come home that night and found that his house was well-lighted,
but with the windows closed; that getting suspicious that there
were culprits in his house, Atty. Ojeda retreated to look for a
policeman and finding Basilio in khaki uniform, asked him to
accompany him to the house with the latter refusing on the ground
that he was not a policeman, but suggesting that Atty. Ojeda should
ask the traffic policeman on duty at the corner of Rizal Avenue and
Zurbaran; that Atty. Ojeda went to the traffic policeman at said
Page 126 of 154

INSURANCE G01 CASE DIGESTS


corner and reported the matter, asking the policeman to come
along with him, to which the policeman agreed; that on the way to
the Ojeda residence, the policeman and Atty. Ojeda passed by
Basilio and somehow or other invited the latter to come along; that
as the tree approached the Ojeda residence and stood in front of
the main gate which was covered with galvanized iron, the fence
itself being partly concrete and partly adobe stone, a shot was fired;
that immediately after the shot, Atty. Ojeda and the policeman
sought cover; that the policeman, at the request of Atty. Ojeda, left
the premises to look for reinforcement; that it turned out
afterwards that the special watchman Melencio Basilio was hit in
the abdomen, the wound causing his instantaneous death; that the
shot must have come from inside the yard of Atty. Ojeda, the bullet
passing through a hole waist-high in the galvanized iron gate; that
upon inquiry Atty. Ojeda found out that the savings of his children in
the amount of P30 in coins kept in his aparador contained in
stockings were taken away, the aparador having been ransacked;
that a month thereafter the corresponding investigation conducted
by the police authorities led to the arrest and prosecution of four
persons in Criminal Case No. 15104 of the Court of First Instance of
Manila for Robbery in an Inhabited House and in Band with
Murder.

Issue: Whether or not the Philippine American Life Insurance Co.


liable to the petitioner for the amount covered by the supplemental
contract?

Held: Yes.

Ratio:
The circumstances of Basilios death cannot be taken as purely
intentional on the part of Basilio to expose himself to the danger.
There is no proof that his death was the result of intentional killing
because there is the possibility that the malefactor had fired the
shot merely to scare away the people around.
While as a general rule "the parties may limit the coverage of the
policy to certain particular accidents and risks or causes of loss, and
may expressly except other risks or causes of loss therefrom",
however, it is to be desired that the terms and phraseology of the
exception clause be clearly expressed so as to be within the easy
grasp and understanding of the insured, for if the terms are
doubtful or obscure the same must of necessity be interpreted or
resolved against the one who has caused the obscurity. (Article
1377, new Civil Code) And so it has been generally held that the
"terms in an insurance policy, which are ambiguous, equivocal, or
uncertain . . . are to be construed strictly and most strongly against
the insurer, and liberally in favor of the insured so as to effect the
dominant purpose of indemnity or payment to the insured,
especially where a forfeiture is involved", and the reason for this
rule is that he "insured usually has no voice in the selection or
arrangement of the words employed and that the language of the
contract is selected with great care and deliberation by experts and
legal advisers employed by, and acting exclusively in the interest of,
the insurance company."
Insurance is, in its nature, complex and difficult for the layman to
understand. Policies are prepared by experts who know and can
Page 127 of 154

INSURANCE G01 CASE DIGESTS


anticipate the bearings and possible complications of every
contingency. So long as insurance companies insist upon the use of
ambiguous, intricate and technical provisions, which conceal rather
than frankly disclose, their own intentions, the courts must, in
fairness to those who purchase insurance, construe every ambiguity
in favor of the insured.

141. PHIL AM CARE HEALTH SYSTEMS V. COURT OF APPEALS, 379


SCRA 356 (2002)
PHILAMCARE HEALTH SYSTEMS, INC. V CA (TRINOS)
379 SCRA 357

An insurer should not be allowed, by the use of obscure phrases and


exceptions, to defeat the very purpose for which the policy was
procured.

YNARES-SANTIAGO; March 18, 2002

Applying this to the situation, the ambiguous or obscure terms in


the insurance policy are to be construed strictly against the insurer
and liberally in favor of the insured party. The reason is to ensure
the protection of the insured since these insurance contracts are
usually arranged and employed by experts and legal advisers acting
exclusively in the interest of the insurance company. As long as
insurance companies insist upon the use of ambiguous, intricate
and technical provisions, which conceal their own intentions, the
courts must, in fairness to those who purchase insurance, construe
every ambiguity in favor of the insured.

NATURE
Petition for review of CA decision

FACTS
- Ernani TRINOS, deceased husband of respondent Julita, applied for
a health care coverage with Philamcare Health Systems, Inc. In the
standard application form, he answered no to the question: Have
you or any of your family members ever consulted or been treated
for high blood pressure, heart trouble, diabetes, cancer, liver
disease, asthma or peptic ulcer? (If Yes, give details).
- The application was approved for period of one year; upon
termination, it was extended for another 2 years. Amount of
coverage was increased to a maximum sum of P75T per disability.
- During this period, Ernani suffered a HEART ATTACK and was
confined at the Manila Medical Center (MMC) for one month. While
her husband was in the hospital, Julita tried to claim the
hospitalization benefits.
Page 128 of 154

INSURANCE G01 CASE DIGESTS


- Petitioner treated the Health Care Agreement (HCA) as void since
there was a concealment regarding Ernanis medical history.
Doctors at the MMC allegedly discovered at the time of his
confinement, he was hypertensive, diabetic and asthmatic. Julita
then paid the hospitalization expenses herself, amounting to about
P76T.
- After her husband died, Julita instituted action for damages
against Philamcare and its Pres. After trial, the lower court ruled in
her favor and ordered Philamcare to reimburse medical and
hospital coverage amounting to P76T plus interest, until fully paid;
pay moral damages of P10T; pay exemplary damages of P10T; attys
fees of P20T.
- CA affirmed the decision of the trial court but deleted all awards
for damages and absolved petitioner Reverente.

(4) It is not an insurance company, governed by Insurance


Commission, but a Health Maintenance Organization under the
authority of DOH.
(5) Trinos concealed a material fact in his application.
(6) Julita was not the legal wife since at the time of their marriage,
the deceased was previously married to another woman who was
still alive.*

ISSUES
1. WON a health care agreement is an insurance contract (If so,
incontestability clause under the Insurance Code is applicable)
2. WON the HCA can be invalidated on the basis of alleged
concealment

Petitioners Claims
(1) Agreement grants living benefits such as medical check-ups
and hospitalization which a member may immediately enjoy so long
as he is alive upon effectivity of the agreement until its expiration.
(2) Only medical and hospitalization benefits are given under the
agreement without any indemnification, unlike in an insurance
contract where the insured is indemnified for his loss.
(3) HCAs are only for a period of one year; therefore,
incontestability clause does not apply, as it requires effectivity
period of at least 2 yrs.

HELD

YES

Ratio Every person has an insurable interest in the life and health
of himself1. The health care agreement was in the nature of non-life
1 Sec.10. Every person has an insurable interest in the life and health:

(1) of himself, of his spouse and of his children;

Page 129 of 154

INSURANCE G01 CASE DIGESTS


insurance, which is primarily a contract of indemnity. Once the
member incurs hospital, medical or any other expense arising from
sickness, injury or other stipulated contingent, the health care
provider must pay for the same to the extent agreed upon under
the contract.

Reasoning
- A contract of insurance2 is an agreement whereby one undertakes
for a consideration to indemnify another against loss, damage or
liability arising from an unknown or contingent event.
- An insurance contract exists where the following elements concur:
(a) The insured has an insurable interest;
(b) The insured is subject to a risk of loss by the happening of the
peril;
(c) The insurer assumes the risk;
(d) Such assumption of risk is part of a general scheme to distribute
actual losses among a large group of persons bearing a similar risk;
and

(2) of any person on whom he depends wholly or in part for education or support, or in whom he has a pecuniary interest;
(3) of any person under a legal obligation to him for the payment of money, respecting property or service, of which death or illness might
delay or prevent the performance; and
(4) of any person upon whose life any estate or interest vested in him depends.

(e) In consideration of the insurers promise, the insured pays a


premium.

2. NO
Ratio Where matters of opinion or judgment are called for,
answers made in good faith and without intent to deceive will not
avoid a policy even though they are untrue; since in such case the
insurer is not justified in relying upon such statement, but is
obligated to make further inquiry.
Reasoning
- The fraudulent intent on the part of the insured must be
established to warrant rescission of the insurance contract. The
right to rescind should be exercised previous to the commencement
of an action on the contract. No rescission was made. Besides, the
cancellation of health care agreements as in insurance policies
requires:
(a) Prior notice of cancellation to insured;
(b) Notice must be based on the occurrence after effective date of
the policy of one or more of the grounds mentioned;
(c) Must be in writing, mailed or delivered to the insured at the
address shown in the policy;
(d) Must state the grounds relied upon provided in Section 64 of the
Insurance Code and upon request of insured, to furnish facts on
which cancellation is based.

2 Section 2 (1) of the Insurance Code

Page 130 of 154

INSURANCE G01 CASE DIGESTS


- These conditions have not been met. When the terms of insurance
contract contain limitations on liability, courts should construe them
in such a way as to preclude insurer from non-compliance of
obligation. Being a contract of adhesion, terms of an insurance
contract are to be construed strictly against the party which
prepared it the insurer.
- Also, Philamcare had 12 months from the date of issuance of the
Agreement within which to contest the membership of the patient
if he had previous ailment of asthma, and six months from the
issuance of the agreement if the patient was sick of diabetes or
hypertension.
* The health care agreement is in the nature of a contract of
indemnity. Hence, payment should be made to the party who
incurred the expenses. It is clear that respondent paid all the
hospital and medical bills; thus, she is entitled to reimbursement.

Disposition Petition DENIED.

142. FORTUNE INSURANCE V. COURT OF APPEALS, 244 SCRA 308


(1996)

FORTUNE VS CA (1995)
G.R. No. 115278 May 23, 1995
Petitioner: FORTUNE INSURANCE AND SURETY CO., INC. (Fortune)
Respondent: PRODUCERS BANK OF THE PHILIPPINES (PBP)
FACTS:
> PBP filed against Fortune a complaint for recovery of the sum of
P725,000.00 under the policy issued by Fortune. The money was
allegedly lost during a robbery of Producer's armored vehicle while
it was in transit to transfer the money from its Pasay City Branch to
its head office in Makati along Taft Avenue.
>The armored car was driven by Benjamin Magalong escorted by
Security Guard Saturnino Atig.
>Driver Magalong was assigned by PRC Management Systems with
the PBP by virtue of an Agreement and Atiga was assigned by
Unicorn Security Services, Inc. by virtue of a contract of Security
Service.
>After an investigation conducted by the Pasay police authorities,
the driver Magalong and guard Atiga were charged, together with
Edelmer Bantigue, Reynaldo Aquino and John Doe, with violation of
P.D. 532 (Anti-Highway Robbery Law) before the Fiscal of Pasay City.
>Demands were made by PBP but Fortune refused to pay as the loss
is excluded from the coverage of the insurance policy which is
Page 131 of 154

INSURANCE G01 CASE DIGESTS


stipulated under "General Exceptions" Section (b) which reads as
follows:

HELD:
YES.

GENERAL EXCEPTIONS
The company shall not be liable under this policy in
report of
xxx xxx xxx
(b) any loss caused by any dishonest, fraudulent or
criminal act of the insured or any officer, employee,
partner, director, trustee or authorized
representative of the Insured whether acting alone
or in conjunction with others. . . .
8. The plaintiff opposes the contention of the
defendant and contends that Atiga and Magalong
are not its "officer, employee, . . . trustee or
authorized representative . . . at the time of the
robbery.

Fortune is exempt from liability under the general exceptions clause


of the insurance policy.
>It should be noted that the insurance policy entered into by the
parties is a theft or robbery insurance policy which is a form of
casualty insurance (Section 174 of the Insurance Code). Other than
what is mentioned in the provision, the rights and obligations of the
parties must be determined by the terms of their contract, taking
into consideration its purpose and always in accordance with the
general principles of insurance law.
>The purpose of the exception is to guard against liability should the
theft be committed by one having unrestricted access to the
property. In such cases, the terms specifying the excluded classes
are to be given their meaning as understood in common
speech. The terms "service" and "employment" are generally
associated with the idea of selection, control, and compensation.

>RTC & CA: held that there should be recovery. The trial court ruled
that Magalong and Atiga were not employees or representatives of
Producers. The wages and salaries of both Magalong and Atiga are
presumably paid by their respective firms, which alone wields the
power to dismiss them. Neither is the Court prepared to accept the
proposition that driver Magalong and guard Atiga were the
"authorized representatives" of plaintiff.

>A contract of insurance is a contract of adhesion, thus any


ambiguity therein should be resolved against the insurer, or it
should be construed liberally in favor of the insured and strictly
against the insurer. Limitations of liability should be regarded with
extreme jealousy and must be construed
in such a way, as to preclude the insurer from non-compliance with
its obligation.

ISSUE:

>If the terms of the contract are clear and unambiguous, there is no
room for construction and such terms cannot be enlarged or
diminished by judicial construction.

W/N the recovery in the policy is precluded under the general


exceptions clause?

Page 132 of 154

INSURANCE G01 CASE DIGESTS


>An insurance contract is a contract of indemnity. It is settled that
the terms of the policy constitute the measure of the insurer's
liability. In the absence of statutory prohibition to the contrary,
insurance companies have the same rights as individuals to limit
their liability and to impose whatever conditions they deem best
upon their obligations not inconsistent with public policy.

companions. In short, for these particular tasks, the three acted as


agents of Producers. A "representative" is defined as one who
represents or stands in the place of another; one who represents
others or another in a special capacity, as an agent, and is
interchangeable with "agent."

>It was clear that Fortunes intention is to exclude and exempt from
protection and coverage losses arising from dishonest, fraudulent,
or criminal acts of persons granted or having unrestricted access to
Producers' money or payroll. When it used then the term
"employee," it must have had in mind any person who qualifies as
such as generally and universally understood, or jurisprudentially
established in the light of the four standards in the determination of
the employer-employee relationship, or as statutorily declared even
in a limited sense as in the case of Article 106 of the Labor Code
which considers the employees under a "labor-only" contract as
employees of the party employing them and not of the party who
supplied them to the employer.

143. PERLA COMPANIA SEGUROS V. RAMOLETE , 203 SCRA 487


(1991)

>Fortune claims that Producers' contracts with PRC Management


Systems and Unicorn Security Services are "labor-only" contracts.
But even granting for the sake of argument that these contracts
were not "labor-only" contracts, and PRC Management Systems and
Unicorn Security Services were truly independent contractors, we
are satisfied that Magalong and Atiga were, in respect of the
transfer of Producer's money from its Pasay City branch to its head
office in Makati, its "authorized representatives" who served as
such with its teller Maribeth Alampay.
>Producers entrusted the three with the specific duty to safely
transfer the money to its head office, with Alampay to be
responsible for its custody in transit; Magalong to drive the armored
vehicle which would carry the money; and Atiga to provide the
needed security for the money, the vehicle, and his two other

PERLA COMPANIA DE SEGUROS, INC., petitioner,


vs.
HON. JOSE R. RAMOLETE, PRIMITIVA Y. PALMES, HONORATO
BORBON, SR., OFFICE OF THE PROVINCIAL SHERIFF, PROVINCE OF
CEBU, respondents.
FACTS:

A Cimarron PUJ owned and registered in the name of Nelia


Enriquez, and driven by Cosme Casas, collided with a private
jeep owned by the late Calixto Palmes (husband of private
respondent Primitiva Palmes) who was then driving the
private jeep.
The impact of the collision was such that the private jeep
was flung away to a distance of about thirty (30) feet and
then fell on its right side pinning down Calixto Palmes. He
died as a result of cardio-respiratory arrest due to a crushed
chest. It also caused physical injuries on the part of
Adeudatus Borbon who was then only two (2) years old.
Private respondents Primitiva Palmes (widow of Calixto
Palmes) and Honorato Borbon, Sr. (father of minor
Adeudatus Borbon) filed a complaint with the CFI Judge Jose

Page 133 of 154

INSURANCE G01 CASE DIGESTS


R. Ramolete ordered the Borbon claim excluded from the
complaint it being distinct and separate from that of Palmes
CFI: in favor of Palmes
Judgment became final and executory, writ of execution
issued but returned unsatisfied
The judgment debtor Nelia Enriquez was summoned. She
declared under oath that the Cimarron PUJ registered in her
name was covered by a third-party liability insurance policy
issued by petitioner Perla.
Palmes filed a motion for garnishment. The Judge issued an
Order directing the Provincial Sheriff or his deputy to
garnish the third-party liability insurance policy.
Perla filed a MR alleging that the writ was void on the
ground that it (Perla) was not a party to the case and that
jurisdiction over its person had never been acquired by the
trial court
Thus certiorari and prohibition alleging grave abuse of discretion on
the part of respondent Judge Ramolete in ordering garnishment of
the third-party liability insurance contract issued by petitioner Perla
in favor of the judgment debtor, Nelia Enriquez.

clearly had an interest in the proceeds of the third-party liability


insurance contract. In a third-party liability insurance contract, the
insurer assumes the obligation of paying the injured third party to
whom the insured is liable.
The insurer becomes liable as soon as the liability of the insured to
the injured third person attaches. Prior payment by the insured to
the injured third person is not necessary in order that the obligation
of the insurer may arise. From the moment that the insured became
liable to the third person, the insured acquired an interest in the
insurance contract, which interest may be garnished like any other
credit.

ISSUE: Can Perla be made liable? - YES


RATIO:
In the present case, there can be no doubt, therefore, that the trial
court actually acquired jurisdiction over petitioner Perla when it was
served with the writ of garnishment of the third-party liability
insurance policy it had issued in favor of judgment debtor Nelia
Enriquez. Perla cannot successfully evade liability thereon by such a
contention.
Every interest which the judgment debtor may have in property
may be subjected to execution. judgment debtor Nelia Enriquez
Page 134 of 154

INSURANCE G01 CASE DIGESTS


144.

SHAFER V. JUDGE OF RTC, 167 SCRA 386 (1986)

Shafer v Hon Judge of RTC


Facts:

Petitioner Sherman Shafer obtained a private car policy


over his Ford Laser car from Makati Insurance Company,
Inc., for third party liability (TPL).> During the effectivity of
the policy, an information for reckless imprudence resulting
in damage to property and serious physical injuries was filed
against petitioner. The information reads as follows: the
above-named accused, being then the driver and in actual
physical control of a Ford Laser car did then and there
wilfully, unlawfully and criminally drive, operate and
manage the said Ford Laser car in a careless, reckless and
imprudent manner without exercising reasonable caution,
diligence and due care to avoid accident to persons and
damage to property and in disregard of existing traffic rules
and regulations, causing by such carelessness, recklessness
and imprudence the said Ford Laser car to hit and bump a
Volkswagen car and as a result thereof one Jovencio
Poblete, Sr. who was on board of the said Volkswagen car
sustained physical injurieswhich injuries causing deformity
on the face."
The owner of the damaged Volkswagen car filed a separate
civil action against petitioner for damages, while Jovencio
Poblete, Sr., who was a passenger in the Volkswagen car
when allegedly hit and bumped by the car driven by
petitioner, did not reserve his right to file a separate civil
action for damages. Instead, in the course of the trial in the
criminal case, Poblete, Sr. testified on his claim for damages
for the serious physical injuries which he claimed to have
sustained as a result of the accident.

Upon motion, petitioner was granted leave by the former


presiding judge of the trail court to file a third party
complaint against the herein private respondent, Makati
Insurance Company, Inc. Said insurance company,
however, moved to vacate the order granting leave to
petitioner to file a third party complaint against it and/or
to dismiss the same.
The court a quo issued an order dismissing the third party
complaint on the ground that it was premature, based on
the premise that unless the accused (herein petitioner) is
found guilty and sentenced to pay the offended party
(Poblete Sr.) indemnity or damages, the third party
complaint is without cause of action. The court further
stated that the better procedure is for the accused
(petitioner) to wait for the outcome of the criminal aspect
of the case to determine whether or not the accused, also
the third party plaintiff, has a cause of action against the
third party defendant for the enforcement of its third party
liability (TPL) under the insurance contract. Petitioner
moved for reconsideration of said order, but the motion
was denied; hence, this petition.

ISSUE: WON the court a quo erred in dismissing petitioner's third


party complaint on the ground that petitioner had no cause of
action yet against the insurance company (third party defendant).
HELD: YES
There is no need on the part of the insured to wait for the decision
of the trial court finding him guilty of reckless imprudence. The
occurrence of the injury to the third party immediately gave rise to
the liability of the insurer under its policy
Compulsory Motor Vehicle Liability Insurance (third party liability,
or TPL) is primarily intended to provide compensation for the
Page 135 of 154

INSURANCE G01 CASE DIGESTS


death or bodily injuries suffered by innocent third parties or
passengers as a result of a negligent operation and use of motor
vehicles. The victims and/or their dependents are assured of
immediate financial assistance, regardless of the financial capacity
of motor vehicle owners.
The liability of the insurance company under the Compulsory Motor
Vehicle Liability Insurance is for loss or damage. Where an insurance
policy insures directly against liability, the insurer's liability accrues
immediately upon the occurrence of the injury or event upon which
the liability depends, and does not depend on the recovery of
judgment by the injured party against the insured.
The injured for whom the contract of insurance is intended can sue
directly the insurer. The general purpose of statutes enabling an
injured person to proceed directly against the insurer is to protect
injured persons against the insolvency of the insured who causes
such injury, and to give such injured person a certain beneficial
interest in the proceeds of the policy, and statutes are to be liberally
construed so that their intended purpose may be accomplished. It
has even been held that such a provision creates a contractual
relation which inures to the benefit of any and every person who
may be negligently injured by the named insured as if such injured
person were specifically named in the policy.
In the event that the injured fails or refuses to include the insurer as
party defendant in his claim for indemnity against the insured, the
latter is not prevented by law to avail of the procedural rules
intended to avoid multiplicity of suits. Not even a "no action" clause
under the policy-which requires that a final judgment be first
obtained against the insured and that only thereafter can the
person insured recover on the policy can prevail over the Rules of
Court provisions aimed at avoiding multiplicity of suits.

A third party complaint is a device allowed by the rules of


procedure by which the defendant can bring into the original suit a
party against whom he will have a claim for indemnity or
remuneration as a result of a liability established against him in
the original suit. Third party complaints are allowed to minimize the
number of lawsuits and avoid the necessity of bringing two (2) or
more actions involving the same subject matter. They are
predicated on the need for expediency and the avoidance of
unnecessary lawsuits. If it appears probable that a second action
will result if the plaintiff prevails, and that this result can be avoided
by allowing the third party complaint to remain, then the motion to
dismiss the third party complaint should be denied.
Respondent insurance company's contention that the third party
complaint involves extraneous matter which will only clutter,
complicate and delay the criminal case is without merit. An offense
causes two (2) classes of injuries the first is the social injury
produced by the criminal act which is sought to be repaired thru the
imposition of the corresponding penalty, and the second is the
personal injury caused to the victim of the crime, which injury is
sought to be compensated thru indemnity, which is civil in nature.
In the instant case, the civil aspect of the offense charged, i.e.,
serious physical injuries allegedly suffered by Jovencio Poblete, Sr.,
was impliedly instituted with the criminal case. Petitioner may thus
raise all defenses available to him insofar as the criminal and civil
aspects of the case are concerned. The claim of petitioner for
payment of indemnity to the injured third party, under the
insurance policy, for the alleged bodily injuries caused to said third
party, arose from the offense charged in the criminal case, from
which the injured (Jovencio Poblete, Sr.) has sought to recover civil
damages. Hence, such claim of petitioner against the insurance
company cannot be regarded as not related to the criminal action.

Page 136 of 154

INSURANCE G01 CASE DIGESTS

145. MALAYAN INSURANCE V. CA, VALLEJOS ET AL .,, 165 SCRA 136


(1998)

CASUALTY INSURANCE
144. MALAYAN INSURANCE CO., INC.,, vs.

4. PANTRANCO claimed that the jeep of Sio Choy was operated at


an excessive speed and bumped the PANTRANCO bus. It prayed that
it be absolved from any and all liability. Defendant Sio Choy and the
Insurance Company denied the liability, claiming that the fault in
the accident was solely imputable to the PANTRANCO.

THE HON. COURT OF APPEALS, MARTIN C. VALLEJOS,


5. SIO CHOY'S CONTENTION
FACTS
1. Petitioner Malayan issued in favor of the respondent
Sio Choy Private Comprehensive Car Comprehensive covering a
Willys Jeep.
The insurance coverage was for own damage not exceeding
P600.00 and third party liability in the amount of P20,000

2. During the effectivity of the insurance policy, the insured jeep


while being driven by Juan Campollo, an employee of the
respondent San Leon Rice Mill collided with passenger bus
belonging to the respondent PANTRANCO at the national
highway causing damage to the insured vehicle and injuries to
the driver and Respondent Martin Vallejos

3. RESPONDENT VALLEJOS filed an action for damages afaist Sio Roy


Malayan and PANTRANCO before the RTC.

alleged that he had actually paid the Matrin Vallejos, the


amount of P5,000.00 for the hospitalization and other expenses
He alleged that Malayan had issued in his favor a private
car comprehensive car, wherein the insurance company obligated
itself to indemnify Sio Choy, as insured for the damge to his motor
vehicle as well as the liability to third persin arrising to any vehicular
accident
(which the policy is in full force and effect when the vehicular
accident complained of occurred.
He prayed that he be reimbursed by the insurance company for
the amount that he may be ordered to pay
File a third party complaint against San Leon, since the driver
was its employee
6. RTC ruled in favor of the Martin Vallejos and against Sio
Choy and Malayan Insurance and third party defendant San Leon
Rice
Page 137 of 154

INSURANCE G01 CASE DIGESTS


CA affirmed the decision. They are jointly and severally liable for
the damages of Martin Vallejos. However, San Leon Rice Mill has
no obligation to indemnify or reimburse the petitioner
insurance company since it is not a privy to the contract of
insurance between Sio Choy and the insurance company.

ISSUES:
(1) Whether the trial court, as upheld by the Court of Appeals,
was correct in holding petitioner and respondents Sio Choy and
San Leon Rice Mill, Inc. "solidarily liable" to respondent Vallejos; and

(2) Whether petitioner is entitled to be reimbursed by respondent


San Leon Rice Mill, Inc. for whatever amount petitioner has been
adjudged to pay respondent Vallejos on its insurance policy.

RATIONALE:
In order to determine the alleged liability of respondent San Leon
Rice Mill to Malayan, it is important to determine first the nature or
basis of the liability of Malayan to Respondent Vallejos, as
compared to that of respondents Sio and San Leon Rice Mill.
1. No."ONLY" RESPONDENTS SIO CHOY AND SAN LEON RICE
ARE SOLIDARILY LIABLE TO VALLEJOS. MALAYAN IS NOT INCLUDED.

It must be observed that Respondent Sio Choy is made liable to


Vallejos as the owner of the ill- fated Will Jeep pursuant to Article
2184 of the Civil Code.
Art. 2184. In motor vehicle mishaps, the owner is solidarily
liable with his driver, if the
former, who was in the vehicle, could have, by the use of due
diligence, prevented the misfortune it is disputably presumed that
a driver was negligent, x x x
The basis of liability of Respondent San Leon to Vallejos, the former
being the employer of the driver of the Willys Jeep at the time
of the motor vehicle mishap is Article 2180 of the Civil Code
Art. 2180. The obligation imposed by article 2176 is
demandable not only for one's own acts or omissions, but also
for those of persons for whom one is responsible.
Employers shall be liable for the damages caused by their
employees and household helpers acting within the scope of their
assigned tasks, even though the former are not engaged ill any
business or industry. Sio Choy and San Leon Rice Mill are the
principal tortfeasors who are primarily liable to Vallejos. The law
states that the responsibility of two or more persons who are liable
for quasi-delict is solidarily.
ON THE OTHER HAND, the basis of petitioner's liability is its
insurance contract with respondent Sio Choy. If petitioner is
adjudged to pay respondent Vallejos in the amount of not more
than P20,000.00, this is on account of its being the insurer of
respondent Sio Choy under the third party liability clause included
in the private car comprehensive policy existing between petitioner
Page 138 of 154

INSURANCE G01 CASE DIGESTS


and respondent Sio Choy at the time of the complained vehicular
accident

146. HEIRS OF G.Y. POE V. MALAYAN INSURANCE , 584 SCRA 152


(2009)

2. Yes. Malayan has the right to be reimbursed. There is a right of


Subrogation. Petitioner, upon paying respondent Vallejos the
amount of riot exceeding P20,000.00, shall become the subrogee
of the insured, the respondent Sio Choy; as such, it is subrogated to
whatever rights the latter has against respondent San Leon Rice
Mill, Inc. Article 1217 of the Civil Code gives to a solidary debtor
who has paid the entire obligation the right to be reimbursed by his
co-debtors for the share which corresponds to each.

Heirs of GY Poe vs Malayan Insurance

In accordance with Article 1217, petitioner, upon payment


to respondent Vallejos and thereby becoming the subrogee of
solidary debtor Sio Choy, is entitled to reimbursement from
respondent San Leon Rice Mill, Inc.
SC hold that only respondents Sio Choy and San Leon Rice Mill, Inc.
are solidarily liable to the respondent Martin
C. Vallejos for the amount of P29,103.00. Vallejos may enforce the
entire obligation on only one of said solidary debtors. If Sio Choy as
solidary debtor is made to pay for the entire obligation
(P29,103.00) and petitioner, as insurer of Sio Choy, is
compelled to pay P20,000.00 of said entire obligation, petitioner
would be entitled, as subrogee of Sio Choy as against San
Leon Rice Mills, Inc., to be reimbursed by the latter in the amount
of P14,551.50 (which is 1/2 of P29,103.00 )

Facts:
1. George Y. Poe (George) while waiting for a ride to work in
front of Capital Garments Corporation, Ortigas Avenue
Extension, Barangay Dolores, Taytay, Rizal, was run over by
a ten-wheeler Isuzu hauler truck
2. The truck was owned by Rhoda Santos (Rhoda), and then
being driven by Willie Labrador (Willie).
3. To seek redress for Georges untimely death, his heirs and
herein petitioners, namely, his widow Emercelinda, and
their children Flerida and Fernando, filed with the RTC a
Complaint for damages
4. Malayan Insurance was named respondent in this case since
they are the insurer of Rhoda Santos over the subject truck.
5. Under the insurance policy of the said truck Malayan binds
itself, among others, to be liable for damages as well as any
bodily injury to third persons which may be caused by the
operation of the insured vehicle.
6. In their answer, Malayan insurance argued that their
liability will only attach if there is a judicial pronouncement
that the insured Rhoda and her driver are liable.
7. They of course argued that the accident was caused by the
negligence of George.
8. RTC Ruled that Rhoda and Malayan are liable solidarily.
9. CA Set aside the RTC decision.
Issue:
1. Whether or not Malayan Insurance should be held solidarily
liable with Rhoda.
Held:
Page 139 of 154

INSURANCE G01 CASE DIGESTS


1. Court rundle that it is up to petitioner to hold them either
solidary or joint. Nevertheless, Malayan IS LIABLE
2. Respondent MICI does not deny that it is the insurer of the
truck. Nevertheless, it asserts that its liability is limited, and
it should not be held solidarily liable with Rhoda for all the
damages awarded to petitioners.
3. It is settled that where the insurance contract provides for
indemnity against liability to third persons, the liability of
the insurer is direct and such third persons can directly sue
the insurer.
4. The direct liability of the insurer under indemnity contracts
against third party liability does not mean, however, that
the insurer can be held solidarily liable with the insured
and/or the other parties found at fault, since they are being
held liable under different obligations.
5. The liability of the insured carrier or vehicle owner is based
on tort, in accordance with the provisions of the Civil Code
6. The liability of the insurer arises from contract, particularly,
the insurance policy.
7. The third-party liability of the insurer is only up to the
extent of the insurance policy and that required by law; and
it cannot be held solidarily liable for anything beyond that
amount.
8. In the case of Vda de Maglana vs Colsolacion if indeed
Malayan have limited liability under the insurance policy
petitioners would have had the option either (1) to claim
the amount awarded to them from respondent Malayan, up
to the extent of the insurance coverage, and the balance
from Rhoda; or (2) to enforce the entire judgment against
Rhoda, subject to reimbursement from respondent MICI to
the extent of the insurance coverage.
9. However, Malayan failed to present the insurance contract
to prove their limited liability.
10. As such there is no more difference in the amounts of
damages which petitioners can recover from Rhoda or

respondent Malayan; petitioners can recover the said


amounts in full from either of them, thus, making their
liabilities solidary or joint and several.

147.

VDA. DE MAGLANA VS. CONSOLACION, 212 SCRA 268 (1992)

Vda. De Magdalena vs. Consolacion and AFISCO Insurance Corp


(1992)
Facts:
1. Lope Maglana was an employee of the Bureau of Customs
whose work station was at Lasa, Davao City. On December
20, 1978, early morning, Lope Maglana was on his way to
his work, driving a motorcycle, he met an accident that
resulted to his death. He died on the spot.
2. The PUJ jeep that bumped the deceased was driven by
Pepito Into, operated and owned by defendant Destrajo.
3. From the investigation conducted by the traffic investigator,
the PUJ jeep was overtaking another passenger jeep that
was going towards the city poblacion. While overtaking, the
PUJ jeep of defendant Destrajo running abreast with the
overtaken jeep, bumped the motorcycle driven by the
deceased who was going towards the direction of Lasa,
Davao City. The point of impact was on the lane of the
motorcycle and the deceased was thrown from the road
and met his untimely death.
4. Maglanas heirs filed an action for damages and attorneys
fees against Destrajo and AFISCO Insurance Corporation. An
information for homicide thru reckless imprudence was also
filed against Pepito Into.
5. The CFI ruled that Destrajo had not exercised sufficient
diligence as the operator of the jeepney, ordering him to
pay the heirs for damages. AFISCO Insurance Company was
ordered to reimburse Destrajo whatever amounts the
Page 140 of 154

INSURANCE G01 CASE DIGESTS


latter shall have paid only up to the extent of its insurance
coverage.
6. Petitioner heirs filed an MR, contending that AFISCO should
not merely be held secondarily liable because the Insurance
Code provides that the insurer's liability is "direct and
primary and/or jointly and severally with the operator of
the vehicle, although only up to the extent of the insurance
coverage." Hence, they argued that the P20,000.00
coverage of the insurance policy issued by AFISCO, should
have been awarded in their favor.
7. AFISCO argued that since the Insurance Code does not
expressly provide for a solidary obligation, the presumption
is that the obligation is joint.
8. Petitioners reassert their position that the insurance
company is directly and solidarily liable with the negligent
operator up to the extent of its insurance coverage.
Issue: W/N the insurance company is directly and solidarily liable
with the negligent operator up to the extent of its insurance
coverage
Held: Directly liable with the operator, YES. Solidarily liable with the
operator, NO.
The particular provision of the insurance policy on which petitioners
base their claim is as follows: Sec. 1 LIABILITY TO THE PUBLIC
1. The Company will, subject to the Limits of Liability, pay all sums
necessary to discharge liability of the insured in respect of
(a) death of or bodily injury to any THIRD PARTY
....
3. In the event of the death of any person entitled to indemnity
under this Policy, the Company will, in respect of the liability
incurred to such person indemnify his personal representatives in
terms of, and subject to the terms and conditions hereof.

The above-quoted provision leads to no other conclusion but that


AFISCO can be held directly liable by petitioners.
As this Court ruled in Shafer vs. Judge, RTC of Olongapo City "where
an insurance policy insures directly against liability, the insurer's
liability accrues immediately upon the occurrence of the injury or
even upon which the liability depends, and does not depend on the
recovery of judgment by the injured party against the insured."
The underlying reason behind the third party liability (TPL) of the
Compulsory Motor Vehicle Liability Insurance is "to protect injured
persons against the insolvency of the insured who causes such
injury, and to give such injured person a certain beneficial interest
in the proceeds of the policy."
Since petitioners had received from AFISCO the sum of P5,000.00
under the no-fault clause, AFISCO's liability is now limited to
P15,000.00.
However, we cannot agree that AFISCO is likewise solidarily liable
with Destrajo.
In Malayan Insurance Co., Inc. v. Court of Appeals, this Court had
the opportunity to resolve the issue as to the nature of the liability
of the insurer and the insured vis-a-vis the third party injured in an
accident. We categorically ruled thus: While it is true that where the
insurance contract provides for indemnity against liability to third
persons, such third persons can directly sue the insurer,
however, the direct liability of the insurer under indemnity contracts
against third party liability does not mean that the insurer can be
held solidarily liable with the insured and/or the other parties found
at fault. The liability of the insurer is based on contract; that of the
insured is based on tort.

Page 141 of 154

INSURANCE G01 CASE DIGESTS


The Court then proceeded to distinguish the extent of the liability
and manner of enforcing the same in ordinary contracts from that
of insurance contracts. While in solidary obligations, the creditor
may enforce the entire obligation against one of the solidary
debtors, in an insurance contract, the insurer undertakes for a
consideration to indemnify the insured against loss, damage or
liability arising from an unknown or contingent event. To make the
insurer solidarily liable with the latters entire obligation beyond the
sum limited in the insurance contract would result in an evident
breach of the concept of solidary obligation.
Similarly, petitioners herein cannot validly claim that AFISCO, whose
liability under the insurance policy is P20,000.00, can be held
solidarily liable with Destrajo for the total amount of P53,901.70 in
accordance with the decision of the lower court.
Since under both the law and the insurance policy, AFISCO's liability
is only up to P20,000.00, the second paragraph of the dispositive
portion of the decision in question may have unwittingly sown
confusion among the petitioners and their counsel. What should
have been clearly stressed as to leave no room for doubt was the
liability of AFISCO under the explicit terms of the insurance contract.
In fine, we conclude that the liability of AFISCO based on the
insurance contract is direct, but not solidary with that of Destrajo
which is based on Article 2180 of the Civil Code.
As such, petitioners have the option either to claim the P15,000
from AFISCO and the balance from Destrajo or enforce the entire
judgment from Destrajo subject to reimbursement from AFISCO to
the extent of the insurance coverage.

148.

GSIS V. COURT OF APPEALS, 308 SCRA 559 (1999)

GSIS vs CA
GR No. 101439 June 21, 1999
Facts:
National Food Authority (NFA, formerly National Grains
Authority) owned a Chevrolet truck which was insured
against liabilities for death of and injuries to third persons
with GSIS (third-person liability).
The truck, driven by Corbeta, collided with a public utility
vehicle, a Toyota Tamaraw, owned and operated by Victor
Uy, under the name of "Victory Line." The tamaraw was a
total wreck and all the collision victims were its passengers.
5 passengers dies, while 10 sustained bodily injuries.
3 cases were filed with CFU of Agusan del Norte and Butuan
City:
1. quasi-delict, damages and attorney's fees by Uy;
2. damages by Taer, an injured passenger, against Uy
and insurer Mabuhay Insurance (MIGC), to which
Uy filed a cross-claim against MIGC and third-party
complaint against Corbeta and NFA;
3. actions by heirs and victims of the deceased for
damages due to quasi-delict against NFA and
Corbeta, against GSIS as insurer of the truck, against
Uy for breach of contract of carriage, against MIGC
as insurer of the tamaraw.
RTC: Corbeta's negligence was the proximate cause of the
collision; the truck which crossed over to the other lane was
speeding because after the collision, its left front wheel was
detached and fell into a ravine.
RTC: first case - Uy was awarded 109,100 for damages; 2nd
case - dismissed against Uy, ordered MIGC, Corbeta and
NFA to pay Taer; 3rd case - damages are also awarded to
the heirs and victims.
Page 142 of 154

INSURANCE G01 CASE DIGESTS

CA affirmed the ruling of RTC

Issue:
1. Whether GSIS should be held solidarily liable with the negligent
insured/owner-operator of the Chevrolet truck for damages
awarded to private respondents which are beyond the limitations of
the insurance policy and the Insurance Memorandum Circular No. 578
2. Whether the private respondents have no cause of action against
the petitioner, allegedly for failure of the victims to file an insurance
claim within six (6) months from the date of the accident
Ruling:
1. The liability of GSIS is direct, but not solidary with that of the
owner and negligent driver, and its liability is only to the extent of
the insurance policy and those required by law.
Compulsory Motor Vehicle Liability Insurance (third party
liability, or TPL) is primarily intended to provide
compensation for the death or bodily injuries suffered by
innocent third parties or passengers as a result of a
negligent operation and use of motor vehicles. The victims
and/or their dependents are assured of immediate financial
assistance, regardless of the financial capacity of motor
vehicle owners.
The injured for whom the contract of insurance is intended
can sue directly the insurer. The general purpose of statutes
enabling an injured person to proceed directly against the
insurer is to protect injured persons against the insolvency
of the insured who causes such injury, and to give such
injured person a certain beneficial interest in the proceeds
of the policy, and statutes are to be liberally construed so
that their intended purpose may be accomplished. It has
even been held that such a provision creates a contractual
relation which inures to the benefit of any and every person

who may be negligently injured by the named insured as if


such injured person were specifically named in the policy.
However, although the victim may proceed directly against
the insurer for indemnity, the third party liability is only up
to the extent of the insurance policy and those required by
law. While it is true that where the insurance contract
provides for indemnity against liability to third persons, and
such third persons can directly sue the insurer, the direct
liability of the insurer under indemnity contracts against
third party liability does not mean that the insurer can be
held liable in solidum with the insured and/or the other
parties found at fault. For the liability of the insurer is based
on contract; that of the insured carrier or vehicle owner is
based on tort. The liability of GSIS based on the insurance
contract is direct, but not solidary with that of the NFA. The
latters liability is based separately on Article 2180 of the
Civil Code.
As therein provided, the maximum indemnity for death was
twelve thousand (P12,000.00) pesos per victim. The
schedules for medical expenses were also provided by said
IMC, specifically in paragraphs (C) to (G). Consequently,
heirs of the victims who died in the May 9, 1979 vehicular
incident, could proceed (1) against GSIS for the indemnity of
P12,000 for each dead victim, and against NFA and
Guillermo Corbeta for any other damages or expenses
claimed; or (2) against NFA and Corbeta to pay them all
their claims in full.

2. Private respondents have a cause of action.


Evidence shows that a notice of loss was sent to GSIS on
September 7, 1979, almost 4 months after the incident
(May 9, 1979). Likewise, GSIS did not deny receipt of notice
of claim during the trial, and it would be too late to state
otherwise.
Page 143 of 154

INSURANCE G01 CASE DIGESTS

A delay in reporting the loss by the insured and/or by the


beneficiaries must be promptly raised by the insurer in
objecting to the claims.
The insurer failed to object to the presentation of the proof
of loss before the trial court. The defense of delay should
have been promptly interposed. Moreover, the petitioner
merely waited for the victims or beneficiaries to file their
complaint. As matters stand now, the defense of laches or
prescription is deemed waived because of petitioners
failure to raise it not only before but also during the
hearing.

149. FINMAN GEN. ASSURANCE V. COURT OF APPEALS, 213 SCRA


493 (1992)

rendered a favorable response for the respondent. The appellate


court ruled likewise.
Petitioner filed this petition alleging grave abuse of
discretion on the part of the appellate court in applying the
principle of "expresso unius exclusio alterius" in a personal accident
insurance policy, since death resulting from murder and/or assault
are impliedly excluded in said insurance policy considering that the
cause of death of the insured was not accidental but rather a
deliberate and intentional act of the assailant. Therefore, said death
was committed with deliberate intent which, by the very nature of a
personal accident insurance policy, cannot be indemnified.
ISSUE: Whether or not the insurer is liable for the payment of the
insurance premiums?
HELD:

FINMAN GENERAL ASSURANCE CORPORATION vs.THE


HONORABLE COURT OF APPEALS
FACTS:
On October 22, 1986, deceased, Carlie Surposa was insured
with petitioner Finman General Assurance Corporation with his
parents, spouses Julia and Carlos Surposa, and brothers Christopher,
Charles, Chester and Clifton, all surnamed, Surposa, as beneficiaries.
While said insurance policy was in full force and effect, the insured,
Carlie Surposa, died on October 18, 1988 as a result of a stab wound
inflicted by one of the three (3) unidentified men. Private
respondent and the other beneficiaries of said insurance policy filed
a written notice of claim with the petitioner insurance company
which denied said claim contending that murder and assault are not
within the scope of the coverage of the insurance policy. Private
respondent filed a complaint with the Insurance Commission which

Yes, the insurer is still liable.


Contracts of insurance are to be construed liberally in favor
of the insured and strictly against the insurer. Thus ambiguity in the
words of an insurance contract should be interpreted in favor of its
beneficiary. The terms "accident" and "accidental" as used in
insurance contracts have not acquired any technical meaning, and
are construed by the courts in their ordinary and common
acceptation. Thus, the terms have been taken to mean that which
happen by chance or fortuitously, without intention and design, and
which is unexpected, unusual, and unforeseen. Where the death or
injury is not the natural or probable result of the insured's voluntary
act, or if something unforeseen occurs in the doing of the act which
produces the injury, the resulting death is within the protection of
the policies insuring against death or injury from accident. In the
case at bar, it cannot be pretended that Carlie Surposa died in the
course of an assault or murder as a result of his voluntary act
Page 144 of 154

INSURANCE G01 CASE DIGESTS


considering the very nature of these crimes. Neither can it be said
that where was a capricious desire on the part of the accused to
expose his life to danger considering that he was just going home
after attending a festival.
Furthermore, the personal accident insurance policy
involved herein specifically enumerated only ten (10) circumstances
wherein no liability attaches to petitioner insurance company for
any injury, disability or loss suffered by the insured as a result of any
of the stimulated causes. The principle of " expresso unius exclusio
alterius" the mention of one thing implies the exclusion of
another thing is therefore applicable in the instant case since
murder and assault, not having been expressly included in the
enumeration of the circumstances that would negate liability in said
insurance policy cannot be considered by implication to discharge
the petitioner insurance company from liability for, any injury,
disability or loss suffered by the insured. Thus, the failure of the
petitioner insurance company to include death resulting from
murder or assault among the prohibited risks leads inevitably to the
conclusion that it did not intend to limit or exempt itself from
liability for such death.

Page 145 of 154

INSURANCE G01 CASE DIGESTS

SURETYSHIP
150. PHILIPPINE PRYCE ASSURANCE V. COURT OF APPEALS, 230 SCRA
164 (1994)
G.R. No. 107062 February 21, 1994
PHILIPPINE PRYCE ASSURANCE CORPORATION, petitioner,
vs.
THE COURT OF APPEALS, (Fourteenth Division) and GEGROCO,
INC., respondents.
NOCON, J.:
FACTS:
--Petitioner, Interworld Assurance Corporation (now Philippine
Pryce Assurance Corporation), was sued for collection of sum of
money by respondent
Gegroco, Inc.
--The complaint alleged that Pryce issued two surety bonds in behalf
of its principal Sagum General Merchandise for P500,000.00 and
P1,000,000.00
respectively.
--Pryce admitted having executed the said bonds, but denied
liability because allegedly
1) the checks which were to pay for the premiums bounced
and were dishonored hence there is no contract to speak of
between Pryce and its
supposed principal Sagum; and
2) the bonds were merely to guarantee payment of its
principal's obligation, thus, there is a benefit of excussion (a right
under Art. 2066 which only
a guarantor may invoke against the creditor
wherein the guarantor will point out to the creditor all the
debtors properties in the
Philippines sufficient to cover amount of debt).

(NOTE: Article 2047. By guaranty, a person, called the guarantor,


binds himself to the creditor to fulfill the obligation of the
principal debtor in case
the latter should fail to do so. If a person binds
himself solidarily with the principal debtor, the provisions
of Section 4, Chapter 3,
Title I of this Book (Obligations & Contracts) shall be
observed. In such case, the contract is called a suretyship.)
--After the issues had been joined, the case was set for pre-trial
conference.
--Pryce failed to appear during the pre-trial that was reset 3 times
because of the non-appearance of Pryce and/or its counsel during
the scheduled pre-trials.
--Pryce also failed to pay the docket fees for the Third-Party
Complaint it filed against his principal Sagum.
--Pryce was considered as in default and respondent was allowed to
present evidence ex-parte.
RTC---ruled in favor of respondent Gregoco.
CA---affirmed the RTC ruling.
ISSUE: Whether or not there was a contract of suretyship between
Pryce and its principal Sagum which makes Pryce solidarily liable to
Gregoco
HELD: YES.
We do not find any reversible error in the conclusion reached by the
court a quo.
The Insurance Code states that:
Sec. 177. The surety is entitled to payment of the
premium as soon as the contract of suretyship or
bond is perfected and delivered to the obligor. No
Page 146 of 154

INSURANCE G01 CASE DIGESTS


contract of suretyship or bonding shall be valid and
binding unless and until the premium therefor has
been paid, except where the obligee has accepted
the bond, in which case the bond becomes valid
and enforceable irrespective of whether or not the
premium has been paid by the obligor to the
surety. . . . (emphasis added)
The above provision outrightly negates petitioner's first defense. In
a desperate attempt to escape liability, petitioner further asserts
that the above provision is not applicable because the respondent
allegedly had not accepted the surety bond, hence could not have
delivered the goods to Sagum Enterprises. This statement clearly
intends to muddle the facts as found by the trial court and which
are on record.
In the first place, petitioner, in its answer, admitted to have issued
the bonds subject matter of the original action.19 Secondly, the
testimony of Mr. Leonardo T. Guzman, witness for the respondent,
reveals the following:
Q. What are the conditions and
terms of sales you extended to
Sagum General Merchandise?
A. First, we required him to submit
to us Surety Bond to guaranty
payment of the spare parts to be
purchased. Then we sell to them on
90 days credit. Also, we required
them to issue post-dated checks.
Q. Did Sagum General merchandise
comply with your surety bond
requirement?
A. Yes. They submitted to us and
which we have accepted two surety
bonds.

Q Will you please present to us the


aforesaid surety bonds?
A. Interworld Assurance Corp.
Surety Bond No. 0029 for P500,000
dated July 24, 1987 and Interworld
Assurance Corp. Surety Bond No.
0037 for P1,000.000 dated October
7, 1987. 20
Likewise attached to the record are exhibits consisting of delivery
invoices addressed to Sagum General Merchandise proving that
parts were purchased, delivered and received.
On the other hand, petitioner's defense that it did not have
authority to issue a Surety Bond when it did is an admission of fraud
committed against respondent. No person can claim benefit from
the wrong he himself committed. A representation made is
rendered conclusive upon the person making it and cannot be
denied or disproved as against the person relying thereon. 22
Petition is DISMISSED for lack of merit.
SO ORDERED.

Page 147 of 154

INSURANCE G01 CASE DIGESTS


151.

AFP GENERAL INSURANCE V. MOLINA, 556 SCRA 630 (2008)

AFP General Insurance Corporation vs. Molina


556 SCRA 630
2008
Nature:
This is a petition for review on certiorari of the Decision
dated August 20, 2001 of the Court of Appeals in CA-G.R. SP No.
58763, which dismissed herein petitioners special civil action for
certiorari. Before the appellate court, petitioner AFP General
Insurance Corporation (AFPGIC) sought to reverse the Resolution
dated October 5, 1999 of the National Labor Relations Commission
(NLRC) in NLRC NCR CA-011705-96 for having been issued with
grave abuse of discretion.
Facts:
The private respondents are the complainants in a case for
illegal dismissal, docketed as NLRC NCR Case No. 02-00672-90, filed
against Radon Security & Allied Services Agency and/or Raquel Aquias
and Ever Emporium, Inc. In his Decision dated August 20, 1996, the
Labor Arbiter ruled that the private respondents were illegally
dismissed and ordered Radon Security to pay them separation pay,
backwages, and other monetary claims.
Radon Security appealed the Labor Arbiters decision to
public respondent NLRC and posted a supersedeas bond, issued by
herein petitioner AFPGIC as surety.

On April 6, 1998, the NLRC affirmed with modification the decision


of the Labor Arbiter. The NLRC found the herein private
respondents constructively dismissed and ordered Radon Security
to pay them their separation pay, in lieu of reinstatement with
backwages, as well as their monetary benefits limited to three
years, plus attorneys fees equivalent to 10% of the entire amount,
with Radon Security and Ever Emporium, Inc. adjudged jointly and
severally liable.

Radon Security duly moved for reconsideration, but this was


denied by the NLRC in its Resolution dated June 22, 1998. Radon
Security then filed a Petition for Certiorari docketed as G.R. No.
134891 with this Court, but we dismissed this petition in our
Resolution of August 31, 1998.
In dismissing the appeal of AFPGIC, the NLRC pointed out
that AFPGICs theory that the bond cannot anymore be proceeded
against for failure of Radon Security to pay the premium is
untenable, considering that the bond is effective until the finality of
the decision. The NLRC stressed that a contrary ruling would allow
respondents to simply stop paying the premium to frustrate
satisfaction of the money judgment.
AFPGIC then moved for reconsideration, but the NLRC
denied the motion in its Resolution dated February 29, 2000.
AFPGIC then filed a special civil action for certiorari,
docketed as CA-G.R. SP No. 58763, with the Court of Appeals, on the

Page 148 of 154

INSURANCE G01 CASE DIGESTS


ground that the NLRC committed a grave abuse of discretion in
affirming the Order dated March 30, 1999 of the Labor Arbiter.
Issue:

152. REPARATIONS COM. V. UNIVERSAL DEEP SEA FISHING 83 SCRA


764 (1978)
Reparations Commission vs. Universal Deep-Sea Fishing Corp.
Nos. L-21901 and L-21996; June 27, 1

WON THE COURT OF APPEALS SERIOUSLY ERRED IN SUSTAINING


THE PUBLIC RESPONDENT NLRC ALTHOUGH THE LATTER GRAVELY
ABUSED ITS DISCRETION WHEN IT ARBITRARILY IGNORED THE FACT
THAT SUBJECT APPEAL BOND WAS ALREADY CANCELLED FOR NONPAYMENT OF PREMIUM AND THUS IT COULD NOT BE SUBJECT OF
EXECUTION OR GARNISHMENT.
Held:
The filing of a cash or surety bond is a jurisdictional requirement in
an appeal involving monetary award, and the bond shall be in effect
until the final disposition of the case. A surety bond, once accepted
by the obligee (the employee to whom money benefits were due),
becomes valid and enforceable, irrespective of whether or not the
premiums thereon have been paid by the obligor (the employer
liable for payment).

Universal Deep-Sea Fishing was awarded 6 trawl boats by the


Reparations Commission as end-user of reparations goods. The six
fishing boats were delivered to Universal two a time. In each
transaction, Universal executed a performance bond with Manila
Surety as the Surety in favor of the Reparations Commission. A
corresponding indemnity agreement was executed to indemnify the
surety company for any damage, loss, charges, etc., which it may
sustain or incur as a consequence of having become surety upon the
performance bond.

Facts:
Universal Deep-Sea Fishing Corporation was awarded six (6)
trawl boats by the Reparations Commission as end-user of
reparations goods. These fishing boats, christened the M/S
UNIFISH 1, M/S UNIFISH 2, M/S UNIFISH 3, M/S UNIFISH 4, M/S
UNIFISH 5, and M/S UNIFISH 6, were delivered to Universal two
at a time, f.o.b. Japanese port.
M/S UNIFISH 1 and M/S UNIFISH 2, with an aggregate purchase
price of P536,428.44, were delivered to Universal on November
20, 1958 and the Contract of Conditional Purchase and Sale of
Reparations Goods executed by the parties on February 12, 1960
Page 149 of 154

INSURANCE G01 CASE DIGESTS


provided among others, that the first installment representing
10% of the amount or P53,642.84 shall be paid within 24 months
from the date of complete delivery thereof. The due date of the
first installment was on May 8, 1961 and the term is for the
payment of purchase price in ten (10) equal yearly installments
beginning on May 8, 1962.
To guarantee the faithful compliance with the obligations under
said contract, a performance bond in the amount of P53,640.00,
with Universal as principal and the Manila Surety & Fidelity Co.,
Inc., as surety, was executed in favor of the Reparations
Commission. A Corresponding indemnity agreement was
executed to indemnify the surety company for any damage, loss
charges, etc., which it may sustain or incur as a consequence of
having become a surety upon the performance bond.
M/S UNIFISH 3 and M/S UNIFISH 4, with a total purchase price of
P687,777.76 were delivered to Universal on April 20, 1959 and
the Contract of Conditional Purchase and Sale of Reparation
Goods, dated November 25, 1959, provided that the first
installment representing 10% of the amount or P68,777.77 shall
be paid within 24 months from the date of complete delivery
thereof. The due date of the first installment was on July 1961
and the term is for the payment of ten (10) equal yearly
installments beginning July, 1962.
A performance bond in the amount of P68,777.77, issued by the
Manila Surety & Fidelity Co., Inc., was also submitted to
guarantee the faithful compliance with the obligations set forth
in the contract, and indemnity agreement was executed in favor
of the surety company in consideration of the said bond.

The delivery of the M/S UNIFISH 5 and M/S UNIFISH 6 is covered


by a contract for the Utilization of Reparations Goods executed
by the parties on February 12, 1960. The first installment (10% of
F.O.B. Cost), P54,500.00, was due October 17, 1961. The term is
for the payment of ten (1) equal yearly installments beginning
October 17, 1962. A performance bond in judgment, amount of
P54,500.00 issued by judgment, Manila Surety & Fidelity Co.,
Inc., was submitted, and an indemnity agreement was executed
by Universal in favor of judgment, surety company.
On August 10, 1962, the Reparations Commission filed an action
against Universal and Manila Surety to recover various amounts
of money due under these contracts. Universal claimed that the
amounts of money sought to be collected are not yet due and
demandable.
Manila Surety also contended that the action is premature, but
set up a cross-claim against Universal for reimbursement of
whatever amount of money it may have to pay by the plaintiff by
reason of the complaint, including interest, and for the collection
of accumulated and unpaid premiums on the bonds with interest
thereon. It also filed a third-party complaint against Pablo S.
Sarmiento, one of the indemnitors in the indemnity agreements.
The latter denied personal liability claiming that he signed the
agreements in question as his capacity as acting general manage
of Universal.
The Trial Court rendered judgment ordering Universal to pay the
Reparations Commission the amount they were seeking to
recover. Manila Surety & Fidelity was also ordered to pay, jointly
and severally with Universal, P53,643.00, P68,777.77, and
P54.058.00 in favor of the plaintiff.
Page 150 of 154

INSURANCE G01 CASE DIGESTS


Universal and Pablo Sarmiento were also ordered to pay, jointly
and severally, Manila Surety, P53,643.00 and P68,777.77 with
12% interest per year from Aug. 10, 1962 until fully paid plus
P2,000.00 as attorneys fees. Universal was likewise ordered to
pay Manila Surety P54,508.00 with 12% interest per year from
August 10, 1962 until fully paid.

Issue: Whether the trial court erred in not awarding in favor of


Manila Surety the premiums on the performance bonds executed by
Universal in favor of the Reparations Commission.

Held: YES. Universal is further ordered to pay the Manila Surety


P7,251.42 for the premiums and documentary stamps on the
performance bonds.

Ratio:
The payment of premiums on the bonds to the surety company

had been expressly undertaken by UNIVERSAL in the indemnity


agreements executed by it in favor of judgment, surety
company. The premium is judgment, consideration for furnishing
judgment, bonds and judgment, obligation to pay judgment,
same subsists for as long as judgment, liability of judgment,
surety shall exist.

153.

ARRANZ V. MANILA FIDELITY, 101 PHIL. 272 (1957)

MELECIO ARRANZ vs. MANILA FIDELITY AND SURETY CO., INC


G.R. No. L-9674

April 29, 1957

FACTS:
1. Manila Fidelity & Surety Co., executed and delivered to the
Manila Ylang Ylang Distillery a surety bond, understood to
pay jointly and severally with Arranz as principal, the sum of
P90,000. The surety bond executed by Arranz and the
Manila Fidelity contains the following stipulation:
The surety hereunder waives notice of
default and expressly agrees that it shall not be
necessary for the Manila Ylang Ylang Distillery, Ltd.
to proceed against the Principal upon his default or
to exhaust the property of said Principal, before
proceeding against the surety, the Surety's liability
under this bond being a primary one and shall be
eligible and demandable immediately upon
occurrence of such default.
2. To secure the surety against loss arising from the surety
bond, plaintiff executed a second mortgaged over the
properties which were transferred by the Manila Ylang
Ylang Distillery to Arranza. When the first installment of
P50K became due the surety, Manila Fidelity, did not have
funds to pay the same, and neither did it have funds to pay
the second installment of P40K which became due.
3. Complaint was filed by the Manila Ylang Ylang Distillery and
a supplemental complaint was later filed to include the
second installment of P40K already due. Manila Fidelity had
no funds with which to pay either the P50K or the P40K due
under the agreement and the only amount it was able to
raise was P20K. And that was paid to Manila Ylang Ylang
Distillery on account.
Page 151 of 154

INSURANCE G01 CASE DIGESTS


4. Manila Fidelity had no money with which to respond for the
obligation. Arranz was constrained to mortgage some of his
properties to PNB to secure a loan to pay their obligation.
PNB, however, required Manila Fidelity to release the
second mortgage it obtained from Arranz. Arranz paid the
property mortgaged from Manila Fidelity except one in
order to release the property from the encumbrance.
5. Arranz then filed a complaint to recover the premiums it
paid during the period when the property was under
mortgage. He argued that the amounts were never due and
owing to the defendant surety and that he paid it against
his will in order to be able to save the properties from loss
and obtain the credit accommodation from the Philippine
National Bank.
6.
7. The TC ruled against Arranz and dismissed the case. Thus,
this appeal.
ISSUE:
WON Arranz is obligated to pay the premium
notwithstanding the failure of Manila Fidelity to pay the
indebtedness secured by it? YES, he is still obligated to pay.
HELD:

Now, therefore, if the above abounded Principal


shall pay promptly said installments and interest thereon
and shall in all respects do and fully observe all and singular
the covenants, agreements and conditions as provided for
in the aforesaid agreement of November 21, 1949, Annexes
"A" and "B" respectively, to the true intent and meaning
thereof, this obligation shall be null and void, otherwise, it
shall remain in full force and effect. (p. 16, R.O.A..)

As the loan and interest remained unpaid the surety continued to


be bound to the creditor-obligee, and as a corollary its right to
collect the premium on the bond also continued.

Plaintiff-appellant, therefore, cannot excuse himself from the


payment of the premium on the bond upon the failure or refusal of
the surety to pay the loan and the interest. Even if, therefore, the
payment of the premium were against his will, still plaintiffappellant has no cause of action for the return thereof, because the
surety was entitled thereto.

The premium is the consideration for furnishing the bond or


the guaranty. While the liability of the surety to the obligee subsists
the premium is collectible from the principal. Under the terms of
the contract of suretyship the surety's obligation is that the
principal pay the loan and the interest thereon, and that the surety
shall be relieved of his obligation when the loan or obligation
secured is paid.

Page 152 of 154

INSURANCE G01 CASE DIGESTS

154. CAPITAL INSURANCE V. RONQUILLO TRADING, 123 SCRA 526


(1983)

Capital Surety & Insurance Co. Inc. vs. Ronquillo Trading


G.R. No. L-36488
July 25, 1983
Facts:
Capital Surety and Insurance Co., Inc., thru its general agent,
executed and issued a surety bond in the amount of $14,800.00 or
its peso equivalent in behalf of Ronquillo
Trading and in favor of S.S. Eurygenes, its master, and/or its agents,
Delgado Shipping Agencies.
The bond was a guarantee for any additional freight which may be
determined to be due on a cargo of 258 surplus army vehicles
consigned from Pusan, Korea to the
Ronquillo Trading on board the S.S. Eurygenes and booked on said
vessel by the Philippine Merchants Steamship Company, Inc.
In consideration for the issuance by the appellant of the aforesaid
surety bond the
appellees executed an indemnity agreement whereby among other
things, they jointly and severally promised to pay the appellant the
sum of P1,827.00 in advance as premium and documentary stamps
for each period of twelve months while the surety bond was in
effect.

On April 30, 1963 or about five (5) days before the expiration of the
liability on the bond,
P.D. Marchessini and Co., Ltd. and Delgado Shipping Agencies, Inc.,
filed a case in the Court of First Instance of Manila against the
Philippine Merchants Steamship Co., Inc.,
Jose L. Bautista, doing business under the name and style of
"Ronquillo Trading", and the herein appellant Capital Insurance &
Surety Co., Inc. for the sum of $14,800.00 or its equivalent in
Philippine currency, for the loss they allegedly suffered as a direct
consequence of the failure of the defendants to load the stipulated
quantity of 406 U.S. surplus army vehicles.
The appellant was made party defendant because of the bond it
posted in behalf of the appellees. Upon the expiration of the 12
months life of the bond, the appellant made a formal demand for
the payment of the renewal premiums and cost of documentary
stamps for another year in the amount of P1,827.00.
The appellees refused to pay, contending that the liability of the
appellant under the surety bond accrued during the period of
twelve months the said bond was originally in force and before its
expiration and that the defendants-appellees were under no
obligation to renew the surety bond.
The appellant, therefore, filed a complaint to recover the sum of P
l,827.00 against the appellees in the City Court of Manila wherein
said court rendered judgment absolving the appellees from the
complaint. The appellant appealed the judgment to the Court of
First Instance of Manila where the decision of the city court was
affirmed and the complaint dismissed. Its motion for
Page 153 of 154

INSURANCE G01 CASE DIGESTS


reconsideration having been denied, appellant filed the instant
appeal.

or accrued liability so as not to let that liability lapse or expire and


thereby bar enforcement.

Issue:

It must be noted that in the surety bond it was stipulated that the
"liability of surety on this bond would expire on May 5, 1963 and
said bond would be cancelled 15 days after its expiration, unless
surety was notified of any existing obligations thereunder."

Wether or not a surety's liability under the bond has accrued, during
the period of twelve months the bond was originally in force and
before its expiration and that herein appellees were under no
obligation to pay the premiums and costs of documentary stamps
for the succeeding period it was in effect?
Held:
Yes.
Ratio:
The bond was given to secure payment by appellees of such
additional freight as would already be due on the cargo when it
actually arrived in Manila. The bond was not executed to secure
obligation or liability which was still to arise after its twelve month
life. While it was true that the lower court held that the bond was
still in effect after its expiry date, the effectivity was not due to a
renewal made by the appellees but because the surety bond
provided that "the liability of the surety will not expire if, as in this
case, it was notified of an existing obligation thereunder".
The meaning of the bond's still being in effect was that, the suit on
the bond instituted by the obligees prior to the expiration of the
"liability" thereunder was only for the purpose of enforcing that
liability and amounted to notice to appellant of an already existing

Under this stipulation the bond expired on the stated date and the
phrase "unless surety was notified of any existing obligations
thereunder" refers to obligations incurred during the term of the
bond.
Under the Indemnity Agreement, the appellees "agreed to pay the
COMPANY the sum of ONE THOUSAND EIGHT HUNDRED ONLY
(P1,800.00) Pesos, Philippine Currency, in advance as premium
thereof for every twelve (12) months or fraction thereof, while this
bond or any renewal or substitution thereof was in effect."
Obviously, the duration of the bond was for "every twelve (12)
months or fraction thereof, while this bond or any renewal or
substitution was in effect." Since the appellees opted not to renew
the contract they cannot be obliged to pay the premiums.
More specifically, where a contract of surety is terminated under its
terms, the liability of the principal for premiums after such
termination ceases notwithstanding the pendency of a lawsuit to
enforce a liability that accrued during its stipulated lifetime. The
appeal was dismissed for lack of merit. The decision of the court a
quo was affirmed.

Page 154 of 154

You might also like